Capstone: Gastrointestinal (NCLEX)

अब Quizwiz के साथ अपने होमवर्क और परीक्षाओं को एस करें!

a 38 y/o client has chronic cirrhosis and is jaundiced. to decrease pruritus from the jaundice, the nurse would: 1. use wool blankets 2. keep room temp above 75.8 3. keep the room temp below 70.8 4. use lots of soap in the bathwater

3 a decrease in sweating decreases pruritus

"Which of the following types of gastritis is associated with Helicobacter pylori and duodenal ulcers? 1. Erosive (hemorrhagic) gastritis 2. Fundic gland gastritis (type A) 3. Antral gland gastritis (type B) 4.Aspiring-induced gastric ulcer

3 - Erosive (hemorrhagic) gastritis can be caused by ingestion of substances that irritate the gastric mucosa. Fundic gland gastritis (type A) is associated with diffuse severe mucosal atrophy and the presence of pernicious anemia. Antral gland gastritis (type B) is the most common form of gastritis, and is associated with Helicobacter pylori and duodenal ulcers

the nurse is caring for a pt who had an open cholecystectomy 24 hrs ago. which of the follwing actions should the nurse take to assist the pt to maintain an effective breathing pattern? select all that apply a. place in a supine position b. provide analgesics for pain c. encourage coughing and deep breathing d. monitor bowel sounds e. assist with splinting during coughing f. maintain bedrest for 48 hrs after

BCE pain relief allows comfortable breathing, preventing shallow respiration's and guarding, splinting makes coughing more comfortable, encouraging coughing and deep breathing to keep lung clear

A client is admitted with a bowel obstruction. The client has nausea, vomiting, and crampy abdominal pain. The physician has written orders for the client to be up ad lib, to have narcotics for pain, to have a nasogastric tube inserted if needed, and for I.V. Ringer's Lactate and hyperalimentation fluids. The nurse should do the following in order of priority from first to last: 1. Assist with ambulation to promote peristalsis 2. Administer Ringer's Lactate 3. Insert a nasogastric tube. 4. Start and infusion of hyperalimentation fluids.

1,2,3,4 The nurse should first help the client ambulate to try to induce peristalsis; this may be effective and require the least amount of invasive procedures. I.V. fluid therapy can be done to correct fluid and electrolyte imbalances (sodium and potassium), and normal saline or Ringer's Lactate to correct interstitial fluid deficit. Nasogastric (NG) decompression of G.I. tract to reduce gastric secretions and nasointestinal tubes may also be used. Hyperalimentation can be used to correct protein deficiency from chronic obstruction, paralytic ileus, or infection.

Which of the following dietary measures would be useful in preventing Esophageal reflux? 1. Eating small, frequent meals 2. increasing fluid intake 3. avoiding air swallowing with meals 4. Adding a bedtime snack to the dietary plan

1. Esophageal reflux worsens when the stomach is over-distended with food. Therefore, an important measure is to eat small,frequent meals.

a 38 y/o woman was admitted with a medical dx of cholecystitis. the nurse would expect the hx of her present illness to include intolerance to which nutrient? 1. carbohydrates 2. fat 3. protein 4. vitamin c

2 inflammation of the gallbladder, presence of gallstones, or both interfere with movement of the bile from the liver or gallbladder into the small intestine, where it promotes the digestion of fats

Which of the following statements about nasoenteric tubes is correct? 1. The tube cannot be attached to suction. 2. The tube contains a soft rubber bag filled with mercury. 3. The tube is taped securely to the client's cheek after insertion. 4. The tube can have its placement determined only by auscultation.

2. A nasoenteric tube has a small balloon at its tip that is weighted with mercury. The weight of the mercury helps advance the tube by gravity through the intestine. Nasoenteric tubes are attached to suction. A nasoenteric tube is not taped in position until it has reached the obstruction. Because the tube has a radiopaque strip, its progress through the intestinal tract can be followed by fluoroscopy.

The client with ulcerative colitis is following orders for bed rest with bathroom privileges. When evaluating the effectiveness of this level of activity, the nurse should determine if the client has: 1. Conserved energy. 2. Reduced intestinal peristalsis. 3. Obtained needed rest. 4. Minimized stress.

2. Although modified bed rest does help conserve energy and promotes comfort, its primary purpose in this case is to help reduce the hypermotility of the colon. Remaining on bed rest does not by itself reduce stress, and if the client is having stress, the nurse can plan with the client to use strategies that will help the client manage the stress.

A client newly diagnosed with ulcerative colitis who has been placed on steroids asks the nurse why steroids are prescribed. The nurse shuld tell the client? 1. "Ulcerative colitis can be cured by the use of steroids." 2. "Steroids are used in severe flare-ups because they can decrease the incidence of bleeding." 3. "Long-term use of steroids will prolong periods of remission." 4.. "The side effects of steroids outweigh their benefits to clients with ulcerative colitis."

2. Steroids are effective in management of the acute symptoms of ulcerative colitis. Steroids do not cure ulcerative colitis, which is a chronic disease. Long-term use is not effective in prolonging the remission and is not advocated. Clients should be assessed carefully for side effects related to steroid therapy, but the benefits of short-term steroid therapy usually outweigh the potential adverse effects.

Which of the following instructions should the nurse include in the teaching plan for a client who is experiencing gastroesophageal reflux disease (GERD)? 1. Limit caffeine intake to two cups of coffee per day 2. Do not lie down for 2 hours after eating 3. Follow a low-protein diet 4. Take medications with milk to decrease irritation

2. The nurse should instruct the client to not lie down for about 2 hours after eating to prevent reflux.

The nurse is teaching the patient a client with a peptic ulcer discharge instructions. The client asks the nurse which type of analgesic he may take. Which of the following responses by the nurse would be most accurate? 1. Aspirin 2. Acetaminophen 3. Naproxen 4. Ibuprofen

2.Acetaminophen is recommended for pain relief because it does no promote irritation of the mucosa. Aspirin, and nonsteroidal anti- inflammatory drugs suchs as naproxen and ibuprofen, may cause irritation of the mucosa and subsequent bleeding

The client admitted to rule out pancreatic islet tumors complains of feeling weak,shaky, and sweaty. Which should be the first intervention implemented by the nurse? 1. Start an IV with D5W. 2. Notify the health-care provider. 3. Perform a bedside glucose check. 4. Give the client some orange juice.

3

The nurse is preparing to teach a client with a peptic ulcer about the diet that should be followed after discharge. The nurse should explain that the diet will most likely consist of which of the following? 1. Bland foods. 2. High-protein foods. 3. Any foods that are tolerated. 4. Large amounts of milk.

3. Diet therapy for ulcer disease is a controversial issue. There is no scientific evidence that diet therapy promotes healing. Most clients are instructed to follow a diet that they can tolerate. There is no need for the client to ingest only a bland or high-protein diet. Milk may be included in the diet, but it is not recommended in excessive amounts.

The client has been taking magnesium hydroxide (milk of magnesia) to control hiatal hernia symptoms. The nurse should assess the client for which of the following conditions most commonly associated with the ongoing use of magnesium based antacids? 1. anorexia 2. weight gain 3. diarrhea 4. constipation

3. The magnesium salts in magnesium hydroxide are related to those found in laxatives and may cause diarrhea.

A client has an appendectomy and develops peritonitis. The nurse should asses the client for an elevated temperature and which additional clinical indication commonly associated with peritonitis? 1. hyperactivity 2. extreme hunger 3. urinary retention 4. local muscular rigidity

4

The nurse should instruct the client to avoid which of the following drugs while taking metoclopramide hydrochloride (Reglan)? 1. Antacids 2. Antihypertensives 3. Anticoagulants 4. Alcohol

4. This drug can cause sedation. Alcohol and other CNS depressants add to this sedation. A client taking this drug should be cautioned to avoid driving or performing other hazardous activities for a few hours after taking the drug.

A 51-year-old woman with Crohn's disease who is taking infliximab (Remicade) calls the nurse in the outpatient clinic about new symptoms. Which symptom is most important to communicate to the health care provider? a. Fever b. Nausea c. Joint pain d. Headache

A

Which of the following would confirm a diagnosis of appendicitis? a. The pain is localized at a position halfway between the umbilicus and the right iliac crest. b. Mr. Liu describes the pain as occurring 2 hours after eating c. The pain subsides after eating d. The pain is in the left lower quadrant"

A

When preparing a patient for a capsule endoscopy study, what should the nurse do? A) Ensure the patient understands the required bowel preparation. B) Have the patient return to the procedure room for removal of the capsule. C) Teach the patient to maintain a clear liquid diet throughout the procedure. D) Explain to the patient that conscious sedation will be used during placement of the capsule.

A A capsule endoscopy study involves the patient performing a bowel prep to cleanse the bowel before swallowing the capsule. The patient will be on a clear liquid diet for 1 to 2 days before the procedure and will remain NPO for 4 to 6 hours after swallowing the capsule. The capsule is disposable and will pass naturally with the bowel movement, although the monitoring device will need to be removed.

The nurse admitting a patient with acute diverticulitis explains that the initial plan of care is to a. administer IV fluids. b. give stool softeners and enemas. c. order a diet high in fiber and fluids. d. prepare the patient for colonoscopy.

A A patient with acute diverticulitis will be NPO and given parenteral fluids

A pt with a history of peptic ulcer disease has presented to the emergency department with complaints of severe abdominal pain and a rigid, boardlike abdomen, prompting the health care team to suspect a perforated ulcer. Which of the following actions should the nurse anticipate? A) Providing IV fluids and inserting a nasogastric tube B) Administering oral bicarbonate and testing the patient's gastric pH level C) Performing a fecal occult blood test and administering IV calcium gluconate D) Starting parenteral nutrition and placing the patient in a high-Fowler's position

A A perforated peptic ulcer requires IV replacement of fluid losses and continued gastric aspiration by NG tube. Nothing is given by mouth and gastric pH testing is not a priority. Calcium gluconate is not a medication directly relevant to the patient's suspected diagnosis and parenteral nutrition is not a priority in the short term.

The nurse is preparing a client for a CT (computed tomography) scanning of the abdomen with contrast. What question should be asked before the examination? A. "Are you allergic to iodine or seafood?" B. "Have you had anything to eat or drink within the past 12 hours?" C. "Have you finished drinking all the required fluid?" D. "Can you tolerate being tilted from side to side during the procedure?"

A Allergies to iodine or seafood can mean a cross-allergic reaction to the contrast dye used for CT scans. Clients reporting such allergies are scheduled for CT without contrast to avoid anaphylactic reactions.

When providing discharge teaching for the patient after a laparoscopic cholecystectomy, what information should the nurse include? A. A lower-fat diet may be better tolerated for several weeks. B. Do not return to work or normal activities for 3 weeks. C. Bile-colored drainage will probably drain from the incision. D. Keep the bandages on and the puncture site dry until it heals.

A Although the usual diet can be resumed, a low-fat diet is usually better tolerated for several weeks following surgery. Normal activities can be gradually resumed as the patient tolerates. Bile-colored drainage or pus, redness, swelling, severe pain, and fever may all indicate infection. The bandage may be removed the day after surgery, and the patient can shower.

Which nursing action will be included in the plan of care for a 27-year-old male patient with bowel irregularity and a new diagnosis of irritable bowel syndrome (IBS)? a. Encourage the patient to express concerns and ask questions about IBS. b. Suggest that the patient increase the intake of milk and other dairy products. c. Educate the patient about the use of alosetron (Lotronex) to reduce symptoms. d. Teach the patient to avoid using nonsteroidal antiinflammatory drugs (NSAIDs).

A Because psychologic and emotional factors can affect the symptoms for IBS, encouraging the patient to discuss emotions and ask questions is an important intervention.

The nurse will anticipate preparing a 71-year-old female patient who is vomiting "coffee-ground" emesis for a. endoscopy. b. angiography. c. barium studies. d. gastric analysis.

A Endoscopy is the primary tool for visualization and diagnosis of upper gastrointestinal (GI) bleeding. Angiography is used only when endoscopy cannot be done because it is more invasive and has more possible complications. Barium studies are helpful in determining the presence of gastric lesions, but not whether the lesions are actively bleeding. Gastric analysis testing may help with determining the cause of gastric irritation, but it is not used for acute GI bleeding.

A nurse is preparing to hang a fat emulsion (lipids) and observes some visible fat globules at the top of the solution. The nurse ensure to do which of the following actions? A. Take another bottle of solution. B. Runs the bottle solution under a warm water. C. Rolls the bottle solution gently. D. Shake the bottle solution vigorously.

A Fat emulsions are used as dietary supplements for patients who are unable to get enough fat in their diet, usually because of certain illnesses or recent surgery. The nurse should examine the bottle of fat emulsion for separation of emulsion into layers or fat globules or the accumulation of froth. The nurse should not hang a fat emulsion if any of these observed and should return the solution to the pharmacy.

A nurse is reviewing nutrition teaching for a client who has cholecystitis. Which of the following food choices can trigger cholecystitis? A. Brownie with nuts B. Bowl of mixed fruit C. Grilled turkey D. Baked potato

A Foods that are high in fat, such as a brownie with nuts, can cause cholecystitis.

A 58-year-old woman has just returned to the nursing unit after an esophagogastroduodenoscopy (EGD). Which action by unlicensed assistive personnel (UAP) requires that the registered nurse (RN) intervene? a. Offering the patient a drink of water b. Positioning the patient on the right side c. Checking the vital signs every 30 minutes d. Swabbing the patient's mouth with cold water

A Immediately after EGD, the patient will have a decreased gag reflex and is at risk for aspiration. Assessment for return of the gag reflex should be done by the RN. The other actions by the UAP are appropriate.

The nurse is caring for a patient in the emergency department with complaints of acute abdominal pain, nausea, and vomiting. When the nurse palpates the patient's left lower abdominal quadrant, the patient complains of pain in the right lower quadrant. The nurse will document this as which of the following diagnostic signs of appendicitis? a. Rovsing sign b. referred pain c. Chvostek's sign d. rebound tenderness

A In patients with suspected appendicitis, Rovsing sign may be elicited by palpation of the left lower quadrant, causing pain to be felt in the right lower quadrant.

A 51-year-old male patient has a new diagnosis of Crohn's disease after having frequent diarrhea and a weight loss of 10 pounds (4.5 kg) over 2 months. The nurse will plan to teach about a. medication use. b. fluid restriction. c. enteral nutrition. d. activity restrictions.

A Medications are used to induce and maintain remission in patients with inflammatory bowel disease (IBD)

The nurse is planning care for a 48-year-old woman with acute severe pancreatitis. The highest priority patient outcome is a. maintaining normal respiratory function. b. expressing satisfaction with pain control. c. developing no ongoing pancreatic disease. d. having adequate fluid and electrolyte balance.

A Respiratory failure can occur as a complication of acute pancreatitis, and maintenance of adequate respiratory function is the priority goal. The other outcomes would also be appropriate for the patient.

The hospital administrator had undergone percutaneous transhepatic cholangiography. which assessment finding indicates complication after the operation? A. Fever and chills B. Hypertension C. Bradycardia D. Nausea and diarrhea

A Septicemia is a common complication after a percutaneous transhepatic cholangiography. Evidence of fever and chills, possibly indicative of septicemia, is important. Hypotension, not hypertension, is associated with septicemia. Tachycardia, not bradycardia, is most likely to occur. Nausea and diarrhea may occur but are not classic signs of sepsis

A nurse is making initial rounds at the beginning of the shift and notice that the parenteral nutrition (PN) bag of an assigned client is empty. Which of the following solutions readily available on the nursing unit should the nurse hang until another PN solution is mixed and delivered to the nursing unit? A. 10% dextrose in water. B. 5% dextrose in water. C. 5% dextrose in normal saline. D. 5% dextrose in lactated Ringer solution.

A The client is at risk of hypoglycemia. Hence the nurse will hang a solution that has the highest amount of glucose until the new parenteral nutrition solution becomes readily available.

The nurse is caring for a patient following an appendectomy. The patienttakes a deep breath, coughs, and then winces in pain. Which of thefollowing statements, if made by the nurse to the patient, is BEST?" A.) "Take three deep breaths, hold your incision, and then cough." B.) "That was good. Do that again and soon it won't hurt as much." C.) "It won't hurt as much if you hold your incision when you cough." D.) "Take another deep breath, hold it, and then cough deeply.

A most effective way of deep breathing and coughing, dilates airway and expands lung surface area

Which information will the nurse include when teaching a patient how to avoid chronic constipation (select all that apply)? a. Many over-the-counter (OTC) medications can cause constipation. b. Stimulant and saline laxatives can be used regularly. c. Bulk-forming laxatives are an excellent source of fiber. d. Walking or cycling frequently will help bowel motility. e. A good time for a bowel movement may be after breakfast.

A, C, D, E

During a home health visit, you are helping a patient develop a list of foods they should avoid due to GERD. Which items in the patient's pantry should be avoided? SELECT-ALL-THAT-APPLY: A. Hot and Spicy Pork Rinds B. Peppermint Patties C. Green Beans D. Tomato Soup E. Chocolate Fondue F. Almonds G. Oranges

ABDEG

A nurse is planning care for a client who has acute gastritis. Which of the following nursinginterventions should the nurse include in the plan of care? (Select all that apply.) A. Evaluate intake and output. B. Monitor laboratory reports of electrolytes. C. Provide three large meals a day. D. Administer ibuprofen for pain. E. Observe stool characteristics.

ABE

The client with hiatal hernia chronically experiences heartburn following meals. The nurses plans to teach the client to avoid which action because it is contraindicated with a hiatal hernia? 1. Lying recumbent following meals 2. Taking in small, frequent, bland meals 3. Raising the head of bed on 6-inch blocks 4. Taking H2-receptor antagonist medication

Answer 1, Hiatal hernia is caused by a protrusion of a portion of the stomach above the diaphragm where the esophagus usually is positioned. The client usually experiences pain from reflux caused by ingestion of irritating foods, lying flat following meals or at night, and eating large or fatty meals. Relief is obtained with the intake of small, frequent meals, use of H2-receptor antagonists and antacids, and elevation of the thorax following meals and during sleep

5. Which of the following drugs is a histamine blocker and reduces levels of gastric acid?" A. Omeprazole (Prilosec) B. Metoclopramide (Reglan) C. Cimetidine (Tagamet) D. Magnesium Hydroxide (Maalox)

Answer C, Cimetidine bind to H2 in the tissue and decreases the production of gastric acid

Which of the following symptoms is associated with ulcerative colitis? 1. Dumping syndrome 2. Rectal bleeding 3. Soft stools 4. Fistulas

Answer: 2. Rectal bleeding In ulcerative colitis, rectal bleeding is the predominant symptom. Soft stools are more commonly associated with Crohn's disease, in which malabsorption is more of a problem. Dumping syndrome occurs after gastric surgeries. Fistulas are associated with Crohn's disease.

Surgical management of ulcerative colitis may be performed to treat which of the following complications? 1. Gastritis 2. Bowel herniation 3. Bowel outpouching 4. Bowel perforation

Answer: 4. Bowel perforation Perforation, obstruction, hemorrhage, and toxic megacolon are common complications of ulcerative colitis that may require surgery. Herniation and gastritis aren't associated with irritable bowel diseases, and outpouching of the bowel is diverticulosis.

If a client had irritable bowel syndrome, which of the following diagnostic tests would determine if the diagnosis is Crohn's disease or ulcerative colitis? 1. Abdominal computed tomography (CT) scan 2. Abdominal x-ray 3. Barium swallow 4. Colonoscopy with biopsy

Answer: 4. Colonoscopy with biopsy A colonoscopy with biopsy can be performed to determine the state of the colon's mucosal layers, presence of ulcerations, and level of cytologic development. An abdominal x-ray or CT scan wouldn't provide the cytologic information necessary to diagnose which disease it is. A barium swallow doesn't involve the intestine.

Which of the following associated disorders may a client with ulcerative colitis exhibit? 1. Gallstones 2. Hydronephrosis 3. Nephrolithiasis 4. Toxic megacolon

Answer: 4. Toxic megacolon Toxic megacolon is extreme dilation of a segment of the diseased colon caused by paralysis of the colon, resulting in complete obstruction. This disorder is associated with both Crohn's disease and ulcerative colitis. The other disorders are more commonly associated with Crohn's disease.

A 54-year-old critically ill patient with sepsis is frequently incontinent of watery stools. What action by the nurse will prevent complications associated with ongoing incontinence? a. Apply incontinence briefs. b. Use a fecal management system c. Insert a rectal tube with a drainage bag. d. Assist the patient to a commode frequently.

B

A patient has the following signs and symptoms: abdominal cramping which is mainly located in the right lower side, ulcers in the mouth, bleeding anal fissure, and diarrhea. Based on the patient's signs and symptoms, which disease does this describe? a. Ulcerative Colitis b. Crohn's Disease

B

The nurse recognizes that teaching a 44-year-old woman following a laparoscopic cholecystectomy has been effective when the patient states which of the following? a. "I can expect yellow-green drainage from the incision for a few days." b. "I can remove the bandages on my incisions tomorrow and take a shower." c. "I should plan to limit my activities and not return to work for 4 to 6 weeks." d. "I will always need to maintain a low-fat diet since I no longer have a gallbladder."

B

A client arrives at the hospital emergency department complaining of acute right lower quadrant abdominal pain, and appendicitis is suspected. Laboratory tests are performed, and the nurse notes that the client's white blood cell (WBC) count is elevated. On the basis of these findings, the nurse should question which health care provider (HCP) prescription documented in the client's medical record? A. Apply a cold pack to the abdomen. B. Administer 30 mL of milk of magnesia (MOM). C. Maintain nothing by mouth (nil per os [NPO]) status. D. Initiate an intravenous (IV) line for the administration of IV fluids.

B Appendicitis should be suspected in a client with an elevated WBC count complaining of acute right lower abdominal quadrant pain. Laxatives are never prescribed because if appendicitis is present, the effect of the laxative may cause a rupture with resultant peritonitis. Cold packs may be prescribed for comfort. The client would be NPO and given IV fluids in preparation for possible surgery.

Which nursing action will the nurse include in the plan of care for a 35-year-old male patient admitted with an exacerbation of inflammatory bowel disease (IBD)? a. Restrict oral fluid intake. b. Monitor stools for blood. c. Ambulate four times daily. d. Increase dietary fiber intake.

B Because anemia or hemorrhage may occur with IBD, stools should be assessed for the presence of blood

A patient in the emergency department has just been diagnosed with peritonitis caused by a ruptured diverticulum. Which prescribed intervention will the nurse implement first? a. Insert a urinary catheter to drainage. b. Infuse metronidazole (Flagyl) 500 mg IV. c. Send the patient for a computerized tomography scan. d. Place a nasogastric (NG) tube to intermittent low suction.

B Because peritonitis can be fatal if treatment is delayed, the initial action should be to start antibiotic therapy (after any ordered cultures are obtained). The other actions can be done after antibiotic therapy is initiated.

After undergoing Billroth I gastric surgery, the client experiences fatigue and complains of numbness and tingling in the feet and difficulties with balance. On the basis of these symptoms, the nurse suspects which postoperative complication? A. Stroke B. Pernicious anemia C. Bacterial meningitis D. Peripheral arterial disease

B Billroth I surgery involves removing one half to two thirds of the stomach and reanastomosing the remaining segment of the stomach to the duodenum. With the loss of this much of the stomach, development of pernicious anemia is not uncommon. Pernicious anemia is a macrocytic anemia that most commonly is caused by the lack of intrinsic factor. During a Billroth I procedure, a large portion of the parietal cells, which are responsible for producing intrinsic factor (a necessary component for vitamin B12 absorption), are removed. In this anemia, the red blood cell is larger than usual and hence does not last as long in the circulation as normal red blood cells do, causing the client to have anemia with resultant fatigue. Vitamin B12 also is necessary for normal nerve function. Because of the lack of the necessary intrinsic factor, persons with pernicious anemia also experience paresthesias, impaired gait, and impaired balance. Although the symptoms could possibly indicate the other options listed, pernicious anemia is the most logical based on the surgery the client underwent.

The nurse determines that teaching regarding cobalamin injections has been effective when the patient with chronic atrophic gastritis states which of the following? a. "The cobalamin injections will prevent gastric inflammation." b. "The cobalamin injections will prevent me from becoming anemic." c. "These injections will increase the hydrochloric acid in my stomach." d. "These injections will decrease my risk for developing stomach cancer."

B Cobalamin supplementation prevents the development of pernicious anemia. Chronic gastritis may cause achlorhydria, but cobalamin does not correct this. The loss of intrinsic factor secretion with chronic gastritis is permanent, and the patient will need lifelong supplementation with cobalamin. The incidence of stomach cancer is higher in patients with chronic gastritis, but cobalamin does not reduce the risk for stomach cancer.

The nurse is assessing a 31-year-old female patient with abdominal pain. Th nurse,who notes that there is ecchymosis around the area of umbilicus, will document this finding as a. Cullen sign. b. Rovsing sign. c. McBurney sign. d. Grey-Turner's sign.

B Cullen sign is ecchymosis around the umbilicus. Rovsing sign occurs when palpation of the left lower quadrant causes pain in the right lower quadrant. Deep tenderness at McBurney's point (halfway between the umbilicus and the right iliac crest), known as McBurney's sign, is a sign of acute appendicitis.

A client is readmitted to the hospital with dehydration after surgery for creation of an ileostomy. The nurse assesses that the client has lost 3 lb of weight, has poor skin turgor, and has concentrated urine. The nurse interprets the client's clinical picture as correlating most closely with recent intake of which medication, which is contraindicated for the ileostomy client? A. Folate B. Biscodyl C. Ferrous sulfate D. Cyanocobalamin

B The client with an ileostomy is prone to dehydration because of the location of the ostomy in the gastrointestinal tract and should not take laxatives. Laxatives will compound the potential risk for the client. These clients are at risk for deficiencies of folate, iron, and cyanocobalamin and should receive them as supplements if necessary.

The nurse is providing dietary instructions to a client with a diagnosis of irritable bowel syndrome. The nurse determines that education was effective if the client states the need to avoid which food? A. Rice B. Corn C. Broiled chicken D. Cream of wheat

B The client with irritable bowel should take in a diet that consists of 30 to 40 g of fiber daily because dietary fiber will help produce bulky, soft stools and establish regular bowel habits. The client should also drink 8 to 10 glasses of fluid daily and chew food slowly to promote normal bowel function. Foods that are irritating to the intestines need to be avoided. Corn is high in fiber but can be very irritating to the intestines and should be avoided. The food items in the other options are acceptable to eat.

A severely malnourished patient reports that he is Jewish. The nurse's initial action to meet his nutritional needs will be to a. have family members bring in food. b. ask the patient about food preferences. c. teach the patient about nutritious Kosher foods. d. order nutrition supplements that are manufactured Kosher.

B The nurse's first action should be further assessment whether or not the patient follows any specific religious guidelines that impact nutrition. The other actions may also be appropriate, based on the information obtained during the assessment.

A 26-year-old woman is being evaluated for vomiting and abdominal pain. Which question from the nurse will be most useful in determining the cause of the patient's symptoms? a. "What type of foods do you eat?" b. "Is it possible that you are pregnant?" c. "Can you tell me more about the pain?" d. "What is your usual elimination pattern?"

C

A 71-year-old patient had an abdominal-perineal resection for colon cancer. Which nursing action is most important to include in the plan of care for the day after surgery? a. Teach about a low-residue diet. b. Monitor output from the stoma. c. Assess the perineal drainage and incision. d. Encourage acceptance of the colostomy stoma.

C

A 76-year-old patient with obstipation has a fecal impaction and is incontinent of liquid stool. Which action should the nurse take first? a. Administer bulk-forming laxatives. b. Assist the patient to sit on the toilet. c. Manually remove the impacted stool. d. Increase the patient's oral fluid intake.

C

A client with a gastric ulcer is prescribed both magnesium hydroxide and cimetidine twice daily. How should the nurse schedule the medications for administration? A. Drink 8 ounces of water between taking each medication. B. Administer the cimetidine and magnesium hydroxide at the same time twice daily. C. Administer each dose of cimetidine 1 hour prior to the administration of magnesium hydroxide. D. Collaborate with the health care provider (HCP), as the client should not be receiving both medications.

C

A nurse is providing discharge teaching to a client who has a new prescription for aluminum hydroxide(Amphojel). The nurse should advise the client to A. take the medication with food. B. monitor for diarrhea. C. wait 1 to 2 hr before taking other oral medications. D. maintain a low-fiber diet.

C

Your patient is unable to have a cholecystectomy for the treatment of cholecystitis. Therefore, a cholecystostomy tube is placed to help treat the condition. Which statement about a cholecystostomy (C-Tube) is TRUE? A. The C-Tube is placed in the cystic duct of the gallbladder and helps drain infected bile from the gallbladder. B. Gallstones regularly drain out of the C-Tube, therefore, the nurse should flush the tube regularly to ensure patency. C. The C-Tube is placed through the abdominal wall and directly into the gallbladder where it will drain infected bile from the gallbladder. D. The tubing and drainage bag of the C-Tube should always be level with the insertion site to ensure the tube is draining properly.

C

an 18 yr old is admitted with an acute onset of right lower quadrant pain. Appendicitis is suspected. For which clinical indicator should the nurse assess the client to determine if the pain is secondary to appendicitis A) urinary retention B) gastric hyperacidity C) rebound tenderness D) increased lower bowel motility

C

The nurse provides discharge instructions for a 64-year-old woman with ascites and peripheral edema related to cirrhosis. Which statement, if made by the patient, indicates teaching was effective? A. "It is safe to take acetaminophen up to four times a day for pain." B. "Lactulose (Cephulac) should be taken every day to prevent constipation." C. "Herbs and other spices should be used to season my foods instead of salt." D. "I will eat foods high in potassium while taking spironolactone (Aldactone)."

C A low-sodium diet is indicated for the patient with ascites and edema related to cirrhosis. Table salt is a well-known source of sodium and should be avoided. Alternatives to salt to season foods include the use of seasonings such as garlic, parsley, onion, lemon juice, and spices. Pain medications such as acetaminophen, aspirin, and ibuprofen should be avoided as these medications may be toxic to the liver. The patient should avoid potentially hepatotoxic over-the-counter drugs (e.g., acetaminophen) because the diseased liver is unable to metabolize these drugs. Spironolactone is a potassium-sparing diuretic. Lactulose results in the acidification of feces in bowel and trapping of ammonia, causing its elimination in feces.

After abdominal surgery, a patient with protein calorie malnutrition is receiving parenteral nutrition (PN). Which is the best indicator that the patient is receiving adequate nutrition? a. Serum albumin level is 3.5 mg/dL. b. Fluid intake and output are balanced. c. Surgical incision is healing normally. d. Blood glucose is less than 110 mg/dL.

C Because poor wound healing is a possible complication of malnutrition for this patient, normal healing of the incision is an indicator of the effectiveness of the PN in providing adequate nutrition. Blood glucose is monitored to prevent the complications of hyperglycemia and hypoglycemia, but it does not indicate that the patient's nutrition is adequate. The intake and output will be monitored, but do not indicate that the PN is effective. The albumin level is in the low-normal range but does not reflect adequate caloric intake, which is also important for the patient.

The nurse is caring for a client experiencing an exacerbation of Crohn's disease. Which intervention should the nurse anticipate the health care provider prescribing? A. Enteral feedings B. Fluid restrictions C. Oral corticosteroids D. Activity restrictions

C Crohn's disease is a form of inflammatory bowel disease that is a chronic inflammation of the gastrointestinal (GI) tract. It is characterized by periods of remission interspersed with periods of exacerbation. Oral corticosteroids are used to treat the inflammation of Crohn's disease, so option 3 is the correct one. In addition to treating the GI inflammation of Crohn's disease with medications, it is also treated by resting the bowel. Therefore, option 1 is incorrect. Option 2 is incorrect, as clients with Crohn's disease typically have diarrhea and would not be on fluid restrictions. Option 4, activity restrictions, is not indicated. The client can do activities as tolerated but should avoid stress and strain.

Which diet choice by the patient with an acute exacerbation of inflammatory bowel disease (IBD) indicates a need for more teaching? a. Scrambled eggs b. White toast and jam c. Oatmeal with cream d. Pancakes with syrup

C During acute exacerbations of IBD, the patient should avoid high-fiber foods such as whole grains.

When assessing a patient's abdomen, what would be most appropriate for the nurse to do? A) Palpate the abdomen before auscultation. B) Percuss the abdomen before auscultation. C) Auscultate the abdomen before palpation. D) Perform deep palpation before light palpation.

C During examination of the abdomen, auscultation is done before percussion and palpation because these latter procedures may alter the bowel sounds.

A patient's capillary blood glucose level is 120 mg/dL 6 hours after the nurse initiated a parenteral nutrition (PN) infusion. The most appropriate action by the nurse is to a. obtain a venous blood glucose specimen. b. slow the infusion rate of the PN infusion. c. recheck the capillary blood glucose in 4 to 6 hours. d. notify the health care provider of the glucose level.

C Mild hyperglycemia is expected during the first few days after PN is started and requires ongoing monitoring. Because the glucose elevation is small and expected, notification of the health care provider is not necessary. There is no need to obtain a venous specimen for comparison. Slowing the rate of the infusion is beyond the nurse's scope of practice and will decrease the patient's nutritional intake.

A 26-year-old woman has been admitted to the emergency department with nausea and vomiting. Which action could the RN delegate to unlicensed assistive personnel (UAP)? a. Auscultate the bowel sounds. b. Assess for signs of dehydration. c. Assist the patient with oral care. d. Ask the patient about the nausea.

C Oral care is included in UAP education and scope of practice. The other actions are all assessments that require more education and a higher scope of nursing practice.

The nurse is teaching the patient and family about possible causative factors for peptic ulcers. The nurse explains that ulcer formation is: a) caused by stressful lifestyle and other acid producing factors such as H. pylori. b) inherited within families and reinforced by bacterial spread of Staphylococcus aureus in childhood. c) promoted by factors that tend to cause oversecretion of acid, such as excess dietary fats, smoking, and H. pylor. d) promoted by a combination of possible factors that may result in erosion of the gastric mucosa, including certain drugs and alcohol

C Peptic ulcers develop only in the presence of an acidic environment. However, an excess of hydrochloric acid (HCl) may not be necessary for ulcer development. The back-diffusion of HCl into the gastric mucosa results in cellular destruction and inflammation. Histamine is released from the damaged mucosa, resulting in vasodilation and increased capillary permeability and further secretion of acid and pepsin. A variety of agents (i.e., certain infections, medications, and lifestyle factors) can damage the mucosal barrier

The nurse is caring for a hospitalized client with a diagnosis of ulcerative colitis. Which finding, if noted on assessment of the client, should the nurse report to the health care provider (HCP)? A. Hypotension B. Bloody diarrhea C. Rebound tenderness D. A hemoglobin level of 12 mg/dL (120 mmol/L)

C Rebound tenderness may indicate peritonitis. Bloody diarrhea is expected to occur in ulcerative colitis. Because of the blood loss, the client may be hypotensive and the hemoglobin level may be lower than normal. Signs of peritonitis must be reported to the HCP.

Four hours after a bowel resection, a 74-year-old male patient with a nasogastric tube to suction complains of nausea and abdominal distention. The first action by the nurse should be to a. auscultate for hypotonic bowel sounds. b. notify the patient's health care provider. c. reposition the tube and check for placement. d. remove the tube and replace it with a new one.

C Repositioning the tube will frequently facilitate drainage. Because this is a common occurrence, it is not appropriate to notify the health care provider unless other interventions do not resolve the problem. Information about the presence or absence of bowel sounds will not be helpful in improving drainage. Removing the tube and replacing it are unnecessarily traumatic to the patient, so that would only be done if the tube was completely occluded.

A client has surgery for a perforated appendix with localized peritonis. In which position should the nurse place the client? A) Sims position B) trendelenburg C) semi-fowlers D)dorsal recumbant

C Semi-fowlers aids in drainage and prevents spread of infection throughout the abodominal cavity.

A nurse in a clinic is reviewing the laboratory reports of a client who has suspected cholelithiasis. Which of the following is an expected finding? A. Serum albumin 4.1 g/dL B. WBC 9,511/uL C. Direct bilirubin 2.1 mg/dL D. Serum cholesterol 171 mg/dL

C This finding is outside the expected reference range and is increased in the client who has cholelithiasis

The client with a hiatal hernia chronically experiences heartburn following meals. The nurse planc to teach the client to avoid which action because it is contraindicated with hiatal hernia? 1. Lying recumbent following meals 2. Taking in small, frequent, bland meals 3. Raising the head of the bed on 6-inch blocks 4. Taking H2-receptor antagonist medication

Correct answer: 1 Laying recumbant following meals or at night will cause reflux and pain. Relief is usually achieved with the intake of small, bland meals, use of H2 receptor antagonists and antacids, and elevation of the thorax after meals and during sleep

The nurse is conducting discharge teaching for a patient with metastatic lung cancer who was admitted with a bowel impaction. Which instructions would be most helpful to prevent further episodes of constipation? a. Maintain a high intake of fluid and fiber in the diet. b. Reduce intake of medications causing constipation. c. Eat several small meals per day to maintain bowel motility. d. Sit upright during meals to increase bowel motility by gravity.

Correct answer: a Rationale: Increased fluid intake and a high-fiber diet reduce the incidence of constipation caused by immobility, medications, and other factors. Fluid and fiber provide bulk that in turn increases peristalsis and bowel motility. Analgesics taken for lung cancer probably cannot be reduced. Other medications may decrease constipation, but it is best to avoid laxatives. Eating several small meals per day and position do not facilitate bowel motility. Defecation is easiest when the person sits on the commode with the knees higher than the hips.

A 42-year-old male patient has had a herniorrhaphy to repair an incarcerated inguinal hernia. Which patient teaching will the nurse provide before discharge? a. Soak in sitz baths several times each day. b. Cough 5 times each hour for the next 48 hours. c. Avoid use of acetaminophen (Tylenol) for pain. d. Apply a scrotal support and ice to reduce swelling.

D

A 47-year-old female patient is transferred from the recovery room to a surgical unit after a transverse colostomy. The nurse observes the stoma to be deep pink with edema and a small amount of sanguineous drainage. The nurse should a. place ice packs around the stoma. b. notify the surgeon about the stoma. c. monitor the stoma every 30 minutes. d. document stoma assessment findings.

D

A 64-year-old woman who has chronic constipation asks the nurse about the use of psyllium (Metamucil). Which information will the nurse include in the response? a. Absorption of fat-soluble vitamins may be reduced by fiber-containing laxatives. b. Dietary sources of fiber should be eliminated to prevent excessive gas formation. c. Use of this type of laxative to prevent constipation does not cause adverse effects. d. Large amounts of fluid should be taken to prevent impaction or bowel obstruction.

D

A client is discharged home with an enteral feeding tube. What does the home health nurse do to determine the patency of the client's enteral tube? A. Arranges for the client to have an x-ray performed periodically B. Auscultates the client's abdomen for bowel sounds before each feeding C. Instills air into the tube to check for placement and patency before each feeding D. Tests aspirated tube contents for pH level before each feeding

D

A client is receiving nutrition via parenteral nutrition (PN). A nurse assess the client for complications of the therapy and assesses the client for which of the following signs of hyperglycemia? A. High-grade fever, chills, and decreased urination. B. Fatigue, increased sweating, and heat intolerance. C. Coarse dry hair, weakness, and fatigue. D. Thirst, blurred vision, and diuresis

D

The gallbladder is found on the __________ side of the body and is located under the ____________. It stores __________.* A. right; pancreas; bilirubin B. left; liver; bile C. right; thymus' bilirubin D. right; liver; bile

D

The nurse will ask a 64-year-old patient being admitted with acute pancreatitis specifically about a history of a. diabetes mellitus. b. high-protein diet. c. cigarette smoking. d. alcohol consumption.

D

The nurse will plan to teach a patient with Crohn's disease who has megaloblastic anemia about the need for a. oral ferrous sulfate tablets. b. regular blood transfusions. c. iron dextran (Imferon) infusions. d. cobalamin (B12) spray or injections.

D

Which client on a medical-surgical unit does the charge nurse assign to the LPN/LVN? A. 28-year-old with morbid obesity who had bariatric surgery today B. 30-year-old recently admitted with severe diarrhea and Clostridium difficile infection C. 36-year-old whose family needs instruction about how to use a gastric feeding tube D. 39-year-old with a jejunal feeding tube who needs elemental feedings administered

D

Which of the following does NOT play a role in the development of GERD? A. Pregnancy B. Hiatal hernia C. Usage of antihistamines or calcium channel blockers D. All the above play a role in GERD

D

Which client requires immediate nursing intervention? The client who: a) complains of epigastric pain after eating. b) complains of anorexia and periumbilical pain. c) presents with ribbonlike stools. d) presents with a rigid, boardlike abdomen.

D A rigid, boardlike abdomen is a sign of peritonitis, a possibly life-threatening condition. Epigastric pain occurring 90 minutes to 3 hours after eating indicates a duodenal ulcer. Anorexia and periumbilical pain are characteristic of appendicitis. Risk of rupture is minimal within the first 24 hours, but increases significantly after 48 hours. A client with a large-bowel obstruction may have ribbonlike stools.

A client is scheduled for an upper gastrointestinal (GI) endoscopy. Which assessment is essential to include in the plan of care following the procedure? A. Assessing pulses B. Monitoring urine output C. Monitoring for rectal bleeding D. Assessing for the presence of the gag reflex

D Following the procedure, the client remains NPO (nothing by mouth) until the gag reflex returns, which is usually in 1 to 2 hours. The remaining options are not specific assessments related to this procedure.

A 42-year-old woman is admitted to the outpatient testing area for an ultrasound of the gallbladder. Which information obtained by the nurse indicates that the ultrasound may need to be rescheduled? a. The patient took a laxative the previous evening. b. The patient had a high-fat meal the previous evening. c. The patient has a permanent gastrostomy tube in place. d. The patient ate a low-fat bagel 4 hours ago for breakfast.

D Food intake can cause the gallbladder to contract and result in a suboptimal study. The patient should be NPO for 8 to 12 hours before the test. A high-fat meal the previous evening, laxative use, or a gastrostomy tube will not affect the results of the study.

A 51-year-old woman had an incisional cholecystectomy 6 hours ago. The nurse will place the highest priority on assisting the patient to a. choose low-fat foods from the menu. b. perform leg exercises hourly while awake. c. ambulate the evening of the operative day. d. turn, cough, and deep breathe every 2 hours.

D Postoperative nursing care after a cholecystectomy focuses on prevention of respiratory complications because the surgical incision is high in the abdomen and impairs coughing and deep breathing. The other nursing actions are also important to implement but are not as high a priority as ensuring adequate ventilation.

A nurse is conducting a follow-up home visit to a client who has been discharged with a parenteral nutrition(PN). Which of the following should the nurse most closely monitor in this kind of therapy? A. Blood pressure and temperature. B. Blood pressure and pulse rate. C. Height and weight. D. Temperature and weight.

D The client's temperature is monitored to identify signs of infection which is one of the complications of this therapy. While the weight is monitored to detect hypervolemia and to determine the effectiveness of this nutritional therapy.

A nurse is completing the admission assessment of a client who has acute pancreatitis. Which of the following findings is the priority to be reported to the provider? A. History of cholelithiasis B. Serum amylase levels three times greater than the expected value C. Client report of severe pain radiating to the back that is rated at an "8" D. Hand spasms present when blood pressure is checked

D The greatest risk to the client is hypocalcemia due to the risk of cardiac dysrhythmia. Hand spasms when taking a blood pressure is an indication of hypocalcemia and is the priority finding to report to the provider

Which patient should the nurse assess first after receiving change-of-shift report? a. 60-year-old patient whose new ileostomy has drained 800 mL over the previous 8 hours b. 50-year-old patient with familial adenomatous polyposis who has occult blood in the stool c. 40-year-old patient with ulcerative colitis who has had six liquid stools in the previous 4 hours d. 30-year-old patient who has abdominal distention and an apical heart rate of 136 beats/minute

D The patient's abdominal distention and tachycardia suggest hypovolemic shock

A patient reports frequent heartburn twice a week for the past 4 months. What other symptoms reported by the patient may indicate the patient has GERD? SELECT-ALL-THAT-APPLY: A. Bitter taste in mouth B. Dry cough C. Melena D. Difficulty swallowing E. Smooth, red tongue F. Murphy's Sign

ABD

After assisting with a needle biopsy of the liver at a patient's bedside, the nurse should a. put pressure on the biopsy site using a sandbag. b. elevate the head of the bed to facilitate breathing. c. place the patient on the right side with the bed flat. d. check the patient's postbiopsy coagulation studies.

C

Which assessment data support to the nurse the clients diagnosis of gastric ulcer? A. Presence of blood in the clients stool for the past month. B. Reports of a burning sensation moving like a wave. C. Sharp pain in the upper abdomen after eating a heavy meal. D. Complaints of epigastric pain 30-60 minutes after ingesting food.

D. The client diagnosed with a gastric ulcer, pain usually occurs 30 to 60 minutes after eating, but not at night. In contrast,no client with a duodenal ulcer has pain during the night often relieved by eating food. Pain occurs 1-3 hours after meals.

which client problem has priority for the client dx with acute pancreatitis? 1. risk for fluid volume deficit 2. alteration in comfort 3. imbalanced nutrition: less than body requirements 4. knowledge deficit

2 autodigestion of the pancreas results in severe epigastric pain, accompanied by nausea, vomiting, abdominal tenderness and muscle guarding

Which of the following lifestyle modifications should the nurse encourage the client with hiatal hernia to include in ADLs? 1. Daily aerobic exercise 2. eliminating smoking and alcohol use 3. balancing activity and rest 4. avoiding high-stress situation

2. Smoking and alcohol use both reduce esophageal sphincter tone and can result in reflux. They therefore should be avoided by clients with hiatal hernia.

Which assessment finding would the nurse need to report most quickly to the health care provider regarding a patient with acute pancreatitis? a. Nausea and vomiting b. Hypotonic bowel sounds c. Abdominal tenderness and guarding d. Muscle twitching and finger numbness

D Muscle twitching and finger numbness indicate hypocalcemia,

the client dx with liver failure is experiencing pruritus secondary to severe jaundice. which action by the unlicensed assistive personnel warrants intervention by the nurse? 1. the UAP is assisting the client to take a hot soapy shower 2. the UAP applies an emollient to the clients legs and back 3. the UAP puts mittens on both hands of the client 4. the cUAP pats the clients skin dry with a clean towel

4

The nurse is caring for a client diagnosed with rule out peptic ulcer disease. Which test confirms this diagnosis? A. Esophagogastroduodenoscopy B. Magnetic resonance imaging C. Occult blood test D. Gastric acid stimulation.

A. The esophagogastroduodenoscopy (EGD) is an invasive diagnostic test which visualizes the esophagus, stomach, and duodenum to accurately diagnose an ulcer and evaluate the effectiveness of the clients treatment.

Colon cancer is most closely associated with which of the following conditions? 1. Appendicitis 2. Hemorrhoids 3. Hiatal hernia 4. Ulcerative colitis

Answer: 4. Ulcerative colitis Chronic ulcerative colitis, granulomas, and familial polyposis seem to increase a person's chance of developing colon cancer. The other conditions listed have no known effect on colon cancer risk.

Your patient, who is presenting with signs and symptoms of GERD, is scheduled to have a test that assesses the function of the esophagus' ability to squeeze food down into the stomach and the closer of the lower esophageal sphincter. The patient asks you, "What is the name of the test I'm having later today?" You tell the patient the name of the test is: A. Lower Esophageal Gastrointestinal Series B. Transesophageal echocardiogram C. Esophageal manometry D. Esophageal pH monitoring

C

A 73-year-old patient with diverticulosis has a large bowel obstruction. The nurse will monitor for a. referred back pain. b. metabolic alkalosis. c. projectile vomiting. d. abdominal distention.

D

The nurse is preparing to administer A.M. medications to the following clients. Which medication should the nurse question before administering? 1. Pancreatic enzymes to the client who has finished breakfast. 2. The pain medication, morphine, to the client who has a respiratory rate of 20. 3. The loop diuretic to the client who has a serum potassium level of 3.9 mEq/L. 4. The beta blocker to the client who has an apical pulse of 68 bpm.

1 Pancreatic enzymes must be administered with meals to enhance the digestion of starches and fats in the gastrointestinal tract.

after a liver biopsy the nurse should place the client in which position? 1. right side 2. left side 3. high fowlers 4. trendelenburg

1 when the client is placed on the right side, the biopsy site of the liver capsule is compressed against the chest wall, and the escape of blood or bile is impeded

A client with peptic ulcer disease is taking ranitidine (Zantac). What is the expected outcome of this drug? 1. Heal the ulcer. 2. Protect the ulcer surface from acids. 3. Reduce acid concentration. 4. Limit gastric acid secretion.

4. Histamine-2 (H2) receptor antagonists, such as ranitidine, reduce gastric acid secretion. Antisecretory, or proton-pump inhibitors, such as omeprazole (Prilosec), help ulcers heal quickly in 4 to 8 weeks. Cytoprotective drugs, such as sucralfate (Carafate), protect the ulcer surface against acid, bile, and pepsin. Antacids reduce acid concentration and help reduce symptoms.

which is the most common upper GI problem? " 1. peptic ulcer disease 2. Crohns 3. Gerd 4. ulcerative colitis

Answer 3, Gerd is the only upper GI problem

A patient had a colonoscopy which showed a "cobble-stone" appearance of the GI lining. This is found in: a. Ulcerative Colitis b. Crohn's Disease

Answer: B

A patient with a peptic ulcer is suddenly vomiting dark coffee ground emesis. On assessment of the abdomen you find bloating and an epigastric mass in the abdomen. Which complication may this patient be experiencing? A. Obstruction of pylorus B. Upper gastrointestinal bleeding C. Perforation D. Peritonitis

Answer: B Signs and symptoms of a possible GI bleeding with a peptic ulcer include: vomiting coffee ground emesis along with bloating, and abdominal mass.

A patient who is given a bisacodyl (Dulcolax) suppository asks the nurse how long it will take to work. The nurse replies that the patient will probably need to use the bedpan or commode within which time frame after administration? a. 2-5 minutes b. 15-60 minutes c. 2-4 hours d. 6-8 hours

Correct answer: b Rationale: Bisacodyl suppositories usually are effective within 15 to 60 minutes of administration, so the nurse should plan accordingly to assist the patient to use the bedpan or commode.

The client is scheduled to have an upper GI tract series of x-rays. Following the x-rays, the nurse should instruct the client to: 1. Take a laxative 2. follow a clear liquid diet 3. Administer an enema 4. Take an antiemetic

1. The client should take a laxative after an upper GI series to stimulate a bowel movement. This examination involves the administration of barium, which must be promptly eliminated from the body because it may harden and cause an obstruction.

In developing a teaching plan for the client with a hiatal hernia, the nurses assessment of which work-related factors would be most useful? 1. number and length of breaks 2. body mechanics used in lifting 3. temperature in the work area 4. Cleansing solvents used

2. Bending, especially after eating, can cause GERD. Lifting heavy objects increases intra-abdominal pressure. Assessing the client's lifting techniques enables the nurse to evaluate the client's knowledge of factors contributing to hiatal hernia and how to prevent complications.

A client with hiatal hernia chronically experiences heartburn following meals. The nurse should plan to teach the client to avoid which action because it is contraindicated with a hiatal hernia? A. Lying recumbent following meals B. Consuming small, frequent, bland meals C. Taking H2-receptor antagonist medication D. Raising the head of the bed on 6-inch (15 cm) blocks

A Hiatal hernia is caused by a protrusion of a portion of the stomach above the diaphragm where the esophagus usually is positioned. The client usually experiences pain from reflux caused by ingestion of irritating foods, lying flat following meals or at night, and eating large or fatty meals. Relief is obtained with the intake of small, frequent, and bland meals; use of H2-receptor antagonists and antacids; and elevation of the thorax following meals and during sleep.

the nurse identifies the client problem "excess fluid volume" for the client in liver failure. which short term goal would be most appropriate for this problem? 1. the client will not gain more than 2kg a day 2. the client will have no increase in abdominal girth 3. the clients v/s will remain WNL 4. the client will receive a low sodium diet

2

the client dx with acute pancreatitis is in pain. which position should the nurse assist the client to assume to help decrease the pain? 1. recommend lying in the prone position with legs extended 2. maintain a tripod position over the bedside table 3. place in side lying position with knees flexed 4. encourage a supine position with a pillow under the knees

3

After dinner time, during hourly rounding, a patient awakes to report they feel like "food is coming up" in the back of their throat and that there is a bitter taste in their mouth. What nursing intervention will you perform next? A. Perform deep suctioning B. Assist the patient into the Semi-Fowler's position C. Keep the patient NPO D. Instruct the patient to avoid milk products

B

If a gastric acid perforates, which of the following actions should not be included in the immediate management of the client? a. Blood replacement b. Antacid administration c. Nasogastric tube suction d. Fluid and electrolyte replacement

B

The nurse will determine that teaching a 67-year-old man to irrigate his new colostomy has been effective if the patient a. inserts the irrigation tubing 4 to 6 inches into the stoma. b. hangs the irrigating container 18 inches above the stoma. c. stops the irrigation and removes the irrigating cone if cramping occurs. d. fills the irrigating container with 1000 to 2000 mL of lukewarm tap water.

B

Which information obtained by the nurse interviewing a 30-year-old male patient is most important to communicate to the health care provider? a. The patient has a history of constipation. b. The patient has noticed blood in the stools. c. The patient had an appendectomy at age 27. d. The patient smokes a pack/day of cigarettes.

B

The nurse is monitoring a client diagnosed with appendicitis who is scheduled for surgery in 2 hours. The client begins to complain of increased abdominal pain and begns to vomit. On assessment, the nurse notes that the abdomen is distended and bowel sounds are diminished. Which is the appropriate nursing intervention? 1. Notify the Physician 2. Administer the prescribed pain medication 3. Call and ask the operating room team to perform the surgery as soon as possible 4. Reposition the client and apply a heating pad on warm setting to the client's abdomen

1 Based on the assessment information the nurse should suspect peritonitis, a complication that is associated with appendicitis, and notify the physician.

A client with a peptic ulcer reports epigastric pain that frequently awakens her during the night. The nurse should instruct the client to do which activities? Select all that apply. 1. Obtain adequate rest to reduce stimulation. 2. Eat small, frequent meals throughout the day. 3. Take all medications on time as ordered. 4. Sit up for one hour when awakened at night. 5. Stay away from crowded areas.

1, 2, 3, 4. The nurse should encourage the client to reduce stimulation that may enhance gastric secretion. The nurse can also advise the client to utilize health practices that will prevent recurrences of ulcer pain, such as avoiding fatigue and elimination of smoking. Eating small, frequent meals helps to prevent gastric distention if not actively bleeding and decreases distension and release of gastrin. Medications should be administered promptly to maintain optimum levels. After awakening during the night, the client should eat a small snack and return to bed, keeping the head of the bed elevated for an hour after eating. It is not necessary to stay away from crowded areas.

The nurse is obtaining a health history from a client who has a sliding hiatal hernia associated with reflux. The nurse should ask the client about the presence of which of the following symptoms? 1. Heartburn 2. Jaundice 3. Anorexia 4. Stomatitis

1. Heartburn, the most common symptom of a sliding hiatal hernia , results from reflux of gastric secretions into the esophagus. Regurgitation of gastric contents and dysphagia are other common symptoms.

The client diagnosed with appendicitis has undergone an appendectomy. At two hours postoperative, the nurse takes the vital signs and notes T 102.6 F, P 132, R 26, and BP 92/46. Which interventions should the nurse implement? List in order of priority. 1. Increase the IV rate. 2. Notify the health care provider. 3. Elevate the foot of the bed. 4. Check the abdominal dressing. 5. Determine if the IV antibiotics have been administered.

1, 3, 4, 5, 2 1. The nurse should increase the IV rate to maintain the circulatory system function until further orders can be obtained.3. The foot of the bed should be elevated to help treat shock, the symptoms of which include elevated pulse and decreased BP. Those signs and an elevated temperature indicate an infection may be present and the client could be developing septicemia.4. The dressing should be assessed to determine if bleeding is occurring.5. The nurse should administer any IV antibiotics ordered after addressing hypovolemia. The nurse will need this information when reporting to the HCP.2. The HCP should be notified when the nurse has the needed information.

A nurse is assessing a client who has been admitted with a diagnosis of an obstruction in the small intestine. The nurse should assess the client for? Select all that apply. 1. Projectile vomiting. 2. Significant abdominal distention. 3. Copious diarrhea. 4. Rapid onset of dehydration. 5. Increased bowel sounds.

1, 4, 5. Signs and symptoms of intestinal obstructions in the small intestine may include projectile vomiting and rapidly developing dehydration and electrolyte imbalances. The client will also have increased bowel sounds, usually high-pitched and tinkling. The client would not normally have diarrhea and would have minimal abdominal distention. Pain is intermittent, being relieved by vomiting. Intestinal obstructions in the large intestine usually evolve slowly, produce persistent pain, and vomiting is less common. Clients with a large-intestine obstruction may develop obstipation and significant abdominal distention.

The results of a patient's recent endoscopy indicate the presence of peptic ulcer disease (PUD). Which of the following teaching points should the nurse provide to the patient in light of his new diagnosis? A) "You'll need to drink at least two to three glasses of milk daily." B) "It would likely be beneficial for you to eliminate drinking alcohol." C) "Many people find that a minced or pureed diet eases their symptoms of PUD." D) "Your medications should allow you to maintain your present diet while minimizing symptoms

2 Alcohol increases the amount of stomach acid produced so it should be avoided. Milk may exacerbate PUD, so two to three glasses would not be recommended. There is no reason to puree or mince food, and a current diet is likely to be altered to minimize symptoms.

A client who had an appendectomy for a perforated appendix returns from surgery with a drain inserted in the incisional site. The purpose of the drain is to: 1. Provide access for wound irrigation. 2. Promote drainage of wound exudates. 3. Minimize development of scar tissue. 4. Decrease postoperative discomfort.

2 Drains are inserted postoperatively in appendectomies when an abscess was present or the appendix was perforated. The purpose is to promote drainage of exudate from the wound and facilitate healing. A drain is not used for irrigation of the wound. The drain will not minimize scar tissue development or decrease postoperative discomfort.

The nurse is assessing an adolescent who is admitted to the hospital with appendicitis. The nurse should report which of the following to the HCP? 1) change in pain rating of 7 to 8 on a 10 point scale. 2) sudden relief of sharp pain, shifting to diffuse pain. 3)shallow breathing with normal vital signs. 4) decrease of pain rating from 8 to 6 when parents visit.

2 The nurse notifies the HCP if the client has sudden relief of sharp pain and on presence of more diffuse pain. this change in the pain indicates the appendix has ruprured. The diffuse pain is typically accompanied by rigid guarding of the abdomen, progressive abdominal distension, tachycardia, pallor, chills, and irritability. The slight increase pain can be expected; the decrease in pain when parents visit may be attributed to being distracted from the pain. shallow breathing is likely due to the pain and is insignificant when other vital signs are normal

The nurse is completing discharge teaching to the client diagnosed with acute pancreatitis. Which instruction should the nurse discuss with the client? 1. Instruct the client to decrease alcohol intake. 2. Explain the need to avoid all stress. 3. Discuss the importance of stopping smoking. 4. Teach the correct way to take pancreatic enzymes

3 Smoking stimulates the pancreas to release pancreatic enzymes and should be stopped.

A 34-year-old female patient with a new ileostomy asks how much drainage to expect. The nurse explains that after the bowel adjusts to the ileostomy, the usual drainage will be about _____ cups. a. 2 b. 3 c. 4 d. 5

A the average amount of ileostomy drainage is about 500 mL daily. One cup is about 240 mL.

Select ALL of the following that are complications associated with Crohn's Disease: A. Cobble-stone appearance of GI lining B. Lead-pipe sign C. Toxic megacolon D. Fistula E. Abscess F. Anal Fissure

A,D,E,F

A client receives a local anesthetic to suppress the gag reflex for a diagnostic procedure of the upper GI tract. Which of the following nursing interventions is advised for this patient? a) The client should be monitored for any breathing related disorder or discomforts b) The client should not be given any food and fluids until the gag reflex returns, c.) The client should be monitored for cramping or abdominal distention, d) The client's fluid output should be measured for at least 24 hours after the procedure

ANSWER: B For a client receiving a local anesthetic that suppresses the gag reflex, the nurse is advised to withhold food and fluids until the reflex returns

Which physical examination should the nurse implement first when assessing the client diagnosed with peptic ulcer disease? A. Auscultate the clients bowel sounds in all four quadrants B. Palpate the abdominal area for tenderness C. Percuss the abdominal borders to identify organs D. Assess the tender area progressing to nontender

A. Auscultation should be used prior to palpitation or percussion when assessing the abdomen. Manipulation of the abdomen can alter bowel sounds and give false information

A nurse is caring for a client for whom a tap water enema is prescribed, to be repeated until the return is clear. Which of the following actions should the nurse take? A. Clarify the order with the provider. B. Explain the procedure to the client. C. Ensure that the tap water is not too hot. D. Keep the amount per enema to less than 1,000 mL.

A. Clarify the order with the provider. Tap water is a hypotonic solution that can cause water toxicity. It should not be repeated. The nurse should clarify the order with the provider. Explaining the procedure to the client, ensuring that the tap water is not too hot, and keeping the amount to less than 1,000 mL are not pertinent if the enema should not be repeated. If you got this question wrong, I am judging you...just kidding :)

Which oral medication should the nurse question before administering to the client with peptic ulcer disease? A. E-mycin, an antibiotic B. Prilosec, a proton pump inhibitor C. Flagyl, an anti microbial agent D. Tylenol, a nonnarcotic analgesic

A. E-mycinis irritating to stomach, and it's use in a client with peptic ulcer disease should be questioned

Caffeinated beverages and smoking are risk factors to assess for in the development of what condition? A. Duodenal ulcers B. Peptic ulcers C. Helicobacter pylori D. Esophageal reflux

Answer: B PUD risk factors include family history, blood group O, smoking tobacco, and beverages containing caffeine

The client is experiencing bleeding related to peptic ulcer disease (PUD). Which nursing intervention is the highest priority? A. Starting a large-bore intravenous (IV) B. Administering intravenous (IV) pain medication C. Preparing equipment for intubation D. Monitoring the client's anxiety level

A. Starting a large-bore intravenous (IV) A large-bore IV should be placed as requested, so that blood products can be administered.

A nurse is completing nutritional teaching for a client who has pancreatitis. Which of the following statements by the client indicates an understanding of the teaching? (SATA) A. I plan to eat small, frequent meals. B. I will eat easy-to-digest foods with limited spice C. I will use skim milk when cooking D. I plan to drink regular cola E. I will limit alcohol intake to two drinkers per day

ABC

The nurse is caring for a hospitalized client with a diagnosis of ulcerative colitis. Which finding, if noted on assessment of the client, would the nurse report to the physician? 1. Bloody diarrhea 2. Hypotension 3. A hemoglobin of 12 mg/dL 4. Rebound tenderness

Answer: 4. Rebound tenderness Rebound tenderness may indicate peritonitis. Blood diarrhea is expected to occur in ulcerative colitis. Because of the blood loss, the client may be hypotensive and the hemoglobin level may be lower than normal. Signs of peritonitis must be reported to the physician.

A patient has the following symptoms: urgent and frequent bowel movements of diarrhea that contains blood with pus and mucous, low hemoglobin/hematocrit, potassium level of 2.0. Based on the patient's signs and symptoms, which disease does this describe? a. Ulcerative Colitis b. Crohn's Disease

Answer: A

A patient has developed a duodenal ulcer. As the nurse, you know that which of the following plays a role in peptic ulcer formation. Select ALL that apply: A. Spicy foods B. Helicobacter pylori C. NSAIDs D. Milk E. Zollinger-Ellison Syndrome

Answer: B, C, E Helicobacter pylori and NSAIDS are the most common causes for peptic ulcer formation. Zollinger-Ellison Syndrome can cause peptic ulcers but it is not as common as H. pylori or NSAIDS. Foods and stress are no longer thought to cause ulcers. Certain foods and stress can irritate ulcers or prolong healing but there is no evidence to suggest they cause them.

A patient with chronic peptic ulcer disease underwent a gastric resection 1 month ago and is reporting nausea, bloating, and diarrhea 30 minutes after eating. What condition is this patient most likely experiencing? A. Gastroparesis B. Fascia dehiscence C. Dumping Syndrome D. Somogyi effect

Answer: C

Which of the following is NOT a potential complication associated with ulcerative colitis? A. Toxic megacolon B. Anemia C. Stricture D. Peritonitis

Answer: C

Which medications are used in the treatment of Crohn's Disease and ulcerative colitis? SELECT-ALL-THAT-APPLY: A. Guanylate Cyclase-C agonists B. Anticholinergics C. 5-Aminosalicylates D. Antacids E. Corticosteroids F. Immune suppressors

Answer: C, E, F

The physician orders a patient with a duodenal ulcer to take a UREA breath test. Which lab value will the test measure to determine if h. pylori is present? A. Ammonia B. Urea C. Hydrochloric acid D. Carbon dioxide

Answer: D If h. pylori are present, the bacteria will release urease which produces ammonia and carbon dioxide. For the test, the patient will ingest a urea tablet and breath samples will be analyzed for carbon dioxide levels.

In the stomach lining, the parietal cells release _________ and the chief cells release __________ which both play a role in peptic ulcer disease. A. pepsin, hydrochloric acid B. pepsinogen, pepsin C. pepsinogen, gastric acid D. hydrochloric acid, and pepsinogen

Answer: D In the stomach lining, the parietal cells release HYDROCHLORIC ACID and the chief cells release PEPSINOGEN which both plays a role in peptic ulcer disease. Pepsinogen then mixes with the hydrochloric acid and turns into pepsin.

A client receiving parenteral nutrition (PN) complains of a headache. A nurse notes that the client has an increased blood pressure, bounding pulse, jugular distension, and crackles bilaterally. The nurse determines that the client is experiencing which complication of PN therapy? A. Air embolism. B. Hypervolemia. C. Hyperglycemia. D. Sepsis.

B

A 71-year-old male patient tells the nurse that growing old causes constipation so he has been using a suppository for constipation every morning. Which action should the nurse take first? a. Encourage the patient to increase oral fluid intake. b. Assess the patient about risk factors for constipation. c. Suggest that the patient increase intake of high-fiber foods. d. Teach the patient that a daily bowel movement is unnecessary.

B

A 44-year-old man admitted with a peptic ulcer has a nasogastric (NG) tube in place. When the patient develops sudden, severe upper abdominal pain, diaphoresis, and a firm abdomen, which action should the nurse take? a. Irrigate the NG tube. b. Check the vital signs. c. Give the ordered antacid. d. Elevate the foot of the bed.

B The patient's symptoms suggest acute perforation, and the nurse should assess for signs of hypovolemic shock. Irrigation of the NG tube, administration of antacids, or both would be contraindicated because any material in the stomach will increase the spillage into the peritoneal cavity. Elevating the foot of the bed may increase abdominal pressure and discomfort, as well as making it more difficult for the patient to breathe.

Which specific data should the nurse obtain from the client who is suspected of having peptic ulcer disease? A. History of side effects experienced from all medications B. Use of non steroidal anti inflammatory drugs (NSAIDs) C. Any known allergies to drugs and environmental factors D. Medical histories of at lease 3 generations

B. Use of NSAIDs places the client at risk for peptic ulcer and hemorrhage. NSAIDs suppress the production of prostaglandin in the stomach, which is a protective mechanism to prevent damage from hydrochloric acid.

A patient in the emergency room has signs and symptoms associated with cholecystitis. What testing do you anticipate the physician will order to help diagnose cholecystitis? Select all that apply: A. Lower GI series B. Abdominal ultrasound C. HIDA Scan (Hepatobiliary Iminodiacetic AciD scan) D. Colonoscopy

BC

Which statements below are CORRECT regarding the role of bile? Select all that apply: A. Bile is created and stored in the gallbladder. B. Bile aids in digestion of fat soluble vitamins, such as A, D, E, and K. C. Bile is released from the gallbladder into the duodenum. D. Bile contains bilirubin.

BCD

Your patient is diagnosed with acute cholecystitis. The patient is extremely nauseous. A nasogastric tube is inserted with GI decompression. The patient reports a pain rating of 9 on 1-10 scale and states the pain radiates to the shoulder blade. Select all the appropriate nursing interventions for the patient: A. Encourage the patient to consume clear liquids. B. Administered IV fluids per MD order. C. Provide mouth care routinely. D. Keep the patient NPO. E. Administer analgesic as ordered. F. Maintain low intermittent suction to NG tube.

BCDEF

Twenty-four hours after endoscopic retrograde cholangiopancreatography (ERCP), a client develops left upper quadrant abdominal pain and has a temperature of 101° F (38.3° C). What is the nurse's best action? A. Administer acetaminophen for control of fever and pain. B. Document the finding, because it is a normal postprocedure event. C. Notify the health care provider. D. Increase the IV fluid rate.

C The client who has undergone an ERCP may develop complications such as perforation or sepsis manifested by fever and abdominal pain. The nurse should report these symptoms to the health care provider immediately.

The nurse is caring for a client diagnosed with hemorrhage get duodenal ulcer. Which collaborative interventions shoulder nurse implement? *Select all that apply.* A. Perform a complete pain assessment B. Assess the clients vital signs frequently C. Administer a proton pump inhibitor intravenously D. Obtain permission and administer blood products E. Monitor the intake of a soft, bland diet

C. This is a collaborative intervention the nurse should implement. It requires an order from the HCP. D. Administering blood products is collaborative, requiring an order from the HCP.

The nurse is caring for a client with ulcerative colitis. Which finding does the nurse determine is consistent with this diagnosis? A. Hypercalcemia B. Hypernatremia C. Frothy, fatty stools D. Decreased hemoglobin

D Ulcerative colitis is an inflammatory disease of the large colon. Findings associated with ulcerative colitis include diarrhea with up to 10 to 20 liquid bloody stools per day, weight loss, anorexia, fatigue, increased white blood cell count, increased erythrocyte sedimentation rate, dehydration, hyponatremia, and hypokalemia (not hypercalcemia). Because of the loss of blood, clients with ulcerative colitis commonly have decreased hemoglobin and hematocrit levels. Clients with ulcerative colitis have bloody diarrhea, not steatorrhea (fatty, frothy, foul-smelling stools).

Which question from the nurse would help determine if a patient's abdominal pain might indicate irritable bowel syndrome? a. "Have you been passing a lot of gas?" b. "What foods affect your bowel patterns?" c. "Do you have any abdominal distention?" d. "How long have you had abdominal pain?"

D irritable bowel syndrome (IBS) is the presence of abdominal discomfort or pain for at least 3 months.

Which of the nursing interventions should be implemented to manage appendicitis? a. Assess pain b. encourage oral intake of clear fluids. c. provide discharge teaching D. assess for symptoms of peritonitis.

D monitor for peritonitis because if the appendix ruptures, bacteria can enter the peritoneum. Pain will be managed with analgesics, and pt should be NPO for surgery. Discharge is not done at this time

The nurse explains to the patient with gastroesophageal reflux disease that this disorder: A. results in acid erosion and ulceration of the esophagus caused by frequent vomiting, B. will require surgical wrapping or repair of the pyloric sphincter to control the symptoms, C. is the protrusion of a portion of the stomach into to esophagus through an opening in the diaphragm, D. often involves relaxation of the lower esophageal sphincter, allowing stomach contents to back up into the espophagus

D. The acidic contents of the stomach touching the inside of the esophagus are responsible for the physical sensation known as "heart-burn" that is a cardinal symptom of GERD

The nurse teaches the client about an anti-ulcer diet. Which of the following statements by the client indicates to the nurse that dietary teaching was successful? 1. "I must eat bland foods to help my stomach heal." 2. "I can eat most foods, as long as they don't bother my stomach." 3. "I cannot eat fruits and vegetables because they cause too much gas." 4. "I should eat a low-fiber diet to delay gastric emptying -

The answer is 2.

Which of these agents is a major contributing factor in the promotion of peptic ulcer disorder? A) Candida albicans. B) staphyloccus infection. C) streptococcus infection D) Helibacter pylori infection

Answer: D. Rationale: Recurrence of peptic ulcers is related to Helicobacter pylori, use of NSAIDs, smoking, and continued acid hypersecretion

The client with chronic alcoholism has chronic pancreatitis and hypomagnesemia.What should the nurse assess when administering magnesium sulfate to the client? 1. Deep tendon reflexes. 2. Arterial blood gases. 3. Skin turgor. 4. Capillary refill time.

1

the client dx with acute pancreatitis is being discharged home. which statement by the client indicates the teaching has been effective? 1. i should decrease my intake of coffee, tea, and cola 2. i will eat a low fat diet and avoid spicy foods 3. i will check my amylase and lipase levels daily 4. i will return to work tomorrow but take it easy

2 high fat and spicy foods stimulate gastric and pancreatic secretions and may precipitate an acute pancreatic attack

A client who has had ulcerative colitis for the past 5 years is admitted to the hospital with an exacerbation of the disease. Which of the following factors was most likely of greatest significance in causing an exacerbation of ulcerative colitis? 1. A demanding and stressful job. 2. Changing to a modified vegetarian diet. 3. Beginning a weight-training program. 4. Walking 2 miles every day.

1. Stressful and emotional events have been clearly linked to exacerbations of ulcerative colitis, although their role in the etiology of the disease has been disproved. A modified vegetarian diet or an exercise program is an unlikely cause of the exacerbation.

the client with an acute exacerbation of chronic pancreatitis has a nasogastric tube. which interventions should the nurse implement? select all that apply 1. monitor the clients bowel sounds 2. monitor the clients food intake 3. assess the clients intravenous site 4. provide oral and nasal care 5. monitor the clients blood glucose

1345 the return of bowel sounds indicates the return of peristalsis, and the nasogastric suction is usually discontinued within 24 to 48 hrs thereafter. the nurse should assess for signs of infection or infiltration. fasting and the ng tube increase the clients risk for mucous membrane irritation and brk dwn. blood glucose levels are monitored because clients with chronic pancreatitis can develop diabetes mellitus

a client with pancreatitis has a hx of alcohol abuse. the nurse will observe the client for agitation, nausea, vomiting, delirium tremens, and visual, auditory and tactile hallucinations. these are indications of: 1. possible cirrhosis of the liver 2. alcohol withdrawal 3. depression 4. suicidal thoughts

2 the airway is of highest priority in postop clients after general anesthesia

Postoperative nursing care for a client after an appendectomy should include which of the following? 1. Administering sitz baths four times a day. 2. Noting the first bowel movement after surgery. 3. Limiting the client's activity to bathroom privileges. 4. Measuring abdominal girth every 2 hours.

2 Noting the client's first bowel movement after surgery is important because this indicates that normal peristalsis has returned. Sitz baths are used after rectal surgery, not appendectomy. Ambulation is started the day of surgery and is not confined to bathroom privileges. The abdomen should be auscultated for bowel sounds and palpated for softness, but there is no need to measure the girth every 2 hours.

a client has had gallbladder attacks in the past and is considered at risk for gallstone formation. the nurse should check that the clients diet is: 1. high in antacids and calcium 2. low in cholesterol 3. high in protein and carbohydrates 4. free of electrolytes

2 biliary calculi are high in cholesterol, therefore the cholesterol intake of the client should be limited

After change-of-shift report, which patient should the nurse assess first? a. 40-year-old male with celiac disease who has frequent frothy diarrhea b. 30-year-old female with a femoral hernia who has abdominal pain and vomiting c. 30-year-old male with ulcerative colitis who has severe perianal skin breakdown d. 40-year-old female with a colostomy bag that is pulling away from the adhesive wafer

B Pain and vomiting with a femoral hernia suggest possible strangulation

The client with an intestinal obstruction continues to have acute pain even though the nasoenteric tube is patent and draining. Which action by the nurse would be most appropriate? 1. Reassure the client that the nasoenteric tube is functioning. 2. Assess the client for a rigid abdomen. 3. Administer an opioid as ordered. 4. Reposition the client on the left side.

2. The client's pain may be indicative of peritonitis, and the nurse should assess for signs and symptoms, such as a rigid abdomen, elevated temperature, and increasing pain. Reassuring the client is important, but accurate assessment of the client is essential. The full assessment should occur before pain relief measures are employed. Repositioning the client to the left side will not resolve the pain.

the client is admitted to the medical dept. with a dx of r/o acute pancreatitis. which laboratory values should the nurse monitor to confirm this dx? 1. creatinine and BUN 2. troponin and CK-MB 3. serum amylase and lipase 4. serum bilirubin and calcium

3 serum amylase increases within 2 to 12 hrs of the onset of acute pancreatitis to 2 to 3 times normal and returns to normal in 3 to 4 days; lipase elevates and remains elevated for 7 to 14 days

the client is immediate postprocedure endoscopic retrograde cholangiopancreatogram (ERCP). which intervention should the nurse implement? 1. assess for rectal bleeding, 2, increase fluid intake 3. assess gag reflex 4. keep in supine position

3 the gag reflex will be suppressed as a result of the local anesthesia applied to the throat to insert the endoscope into the esophagus; therefore, the gag reflex must be assessed prior to allowing the client to resume eating or drinking

Which menu choice by the patient with diverticulosis is best for preventing diverticulitis? a. Navy bean soup and vegetable salad b. Whole grain pasta with tomato sauce c. Baked potato with low-fat sour cream d. Roast beef sandwich on whole wheat bread

A A diet high in fiber and low in fats and red meat is recommended to prevent diverticulitis

The physician prescribes sulfasalazine (Azulfidine) for the client with ulcerative colitis to continue taking at home. Which instruction should the nurse give the client about taking this medication? 1. Avoid taking it with food. 2. Take the total dose at bedtime. 3. Take it with a full glass (240 mL) of water. 4. Stop taking it if urine turns orange-yellow.

3. Adequate fluid intake of at least 8 glasses a day prevents crystalluria and stone formation during sulfasalazine therapy. Sulfasalazine can cause gastrointestinal distress and is best taken after meals and in equally divided doses. Sulfasalazine gives alkaline urine an orange-yellow color, but it is not necessary to stop the drug when this occurs.

Which of the following nursing interventions would most likely promote self-care behaviors in the client with a hiatal hernia? 1. Introduce the client to other people who are successfully managing their care. 2. Include the client's daughter in the teaching so that she can help implement the plan 3. Ask the client to identify other situation in which the client changed health care habits 4. Provide reassurance that the client will be able to implement all aspects of the plan successfully.

3. self-responsibility is the key to individual health maintenance. Using examples of situation in which the client has demonstrated self-responsibility can be reinforcing and supporting. The client has ultimate responsibility for personal health habits.

a client has been given a dx of acute pancreatitis. the nurse will assess this client for: 1. hyperkalemia 2. metabolic acidosis 3. hypocalcemia 4. hyperglycemia

4

A nurse is providing wound care to a client 1 day after the client underwent an appendectomy. A drain was inserted into the incisional site during surgery. Which action should the nurse perform when providing wound care? 1. Remove the dressing and leave the incision open to air. 2. Remove the drain if wound drainage is minimal. 3. Gently irrigate the drain to remove exudate. 4. Clean the area around the drain moving away from the drain.

4 The nurse should gently clean the area around the drain by moving in a circular motion away from the drain. Doing so prevents the introduction of microorganisms to the wound and drain site. The incision cannot be left open to air as long as the drain is intact. The nurse should note the amount and character of wound drainage, but the surgeon will determine when the drain should be removed. Surgical wound drains are not irrigated.

two days after her cholecystectomy, a client has been experiencing nausea and vomiting. the client has a t tube in place. for what electrolyte imbalance will the nurse monitor? 1. hypernatremia 2. hyperkalemia 3. hypervolemia 4. hypokalemia

4 gastric secretions are high in postassium

Before abdominal surgery for an intestinal obstruction, the nurse monitors the client's urine output and finds that the total output for the past 2 hours was 35 mL. The nurse then assesses the client's total intake and output over the last 24 hours and notes that he had 2,000 mL of I.V. fluid for intake, 500 mL of drainage from the nasogastric tube, and 700 mL of urine for a total output of 1,200 mL. This would indicate which of the following? 1. Decreased renal function. 2. Inadequate pain relief. 3. Extension of the obstruction. 4. Inadequate fluid replacement.

4. Considering that there is usually 1 L of insensible fluid loss, this client's output exceeds his intake (intake, 2,000 mL; output, 2,200 mL), indicating deficient fluid volume. The kidneys are concentrating urine in response to low circulating volume, as evidenced by a urine output of less than 30 mL/ hour. This indicates that increased fluid replacement is needed. Decreasing urine output can be a sign of decreased renal function, but the data provided suggest that the client is dehydrated. Pain does not affect urine output. There are no data to suggest that the obstruction has worsened.

A 50-year-old female patient calls the clinic to report a new onset of severe diarrhea. The nurse anticipates that the patient will need to a. collect a stool specimen. b. prepare for colonoscopy. c. schedule a barium enema. d. have blood cultures drawn.

A

The nurse is planning to teach a client with gastroesophageal reflux disease (GERD) about substances to avoid. Which items should the nurse include on this list? Select all that apply. A. Coffee B. Chocolate C. Peppermint D. Nonfat milk E. Fried chicken F. Scrambled eggs

ABCE Foods that decrease lower esophageal sphincter (LES) pressure and irritate the esophagus will increase reflux and exacerbate the symptoms of GERD and therefore should be avoided. Aggravating substances include coffee, chocolate, peppermint, fried or fatty foods, carbonated beverages, and alcohol. Options 4 and 6 do not promote this effect.

Helicobacter pylori can live in the stomach's acidic conditions because it secretes ___________ which neutralizes the acid. A. ammonia B. urease C. carbon dioxide D. bicarbonate

Answer: B

______________ is most commonly found in the terminal ileum and beginning of the colon. a. Ulcerative Colitis b. Crohn's Disease

Answer: B

The client with peptic ulcer disease (PUD) asks the nurse whether licorice and slippery elm might be useful in managing the disease. What is the nurse's best response? A. "No, they probably won't be useful. You should use only prescription medications in your treatment plan." B. "These herbs could be helpful. However, you should talk with your physician before adding them to your treatment regimen." C. "Yes, these are known to be effective in managing this disease, but make sure you research the herbs thoroughly before taking them." D. "No, herbs are not useful for managing this disease. You can use any type of over-the-counter drugs though. They have been shown to be safe."

B. Although these herbs may be helpful in managing PUD, the client should consult his or her physician before making a change in the treatment regimen.

Which expected outcome should the nurse include for a client diagnosed with peptic ulcer disease? A. The clients pain is controlled with the use of NSAIDs B. The client maintains lifestyle modifications C. The client has no signs and symptoms of hemoptysis D. The client take s antacids with each meal

B. Maintaining lifestyle changes such as following an appropriate diet and reducing stress indicate the client is complying with the medical regimen. Compliance is the goal of treatment to prevent complications.

the nurse is collecting data for a pt who develops jaundice and dark, amber colored urine. the nurse recognizes that which of the following is most likely the cause? a. encephalopathy b. pancreatitis c. bile duct obstruction d. cholecystitis

C

To prepare a 56-year-old male patient with ascites for paracentesis, the nurse a. places the patient on NPO status. b. assists the patient to lie flat in bed. c. asks the patient to empty the bladder. d. positions the patient on the right side.

C The patient should empty the bladder to decrease the risk of bladder perforation during the procedure

A 67-year-old male patient with acute pancreatitis has a nasogastric (NG) tube to suction and is NPO. Which information obtained by the nurse indicates that these therapies have been effective? a. Bowel sounds are present. b. Grey Turner sign resolves. c. Electrolyte levels are normal. d. Abdominal pain is decreased.

D

A 60-year-old man who is hospitalized with an abdominal wound infection has only been eating about 50% of meals and states, "Nothing on the menu sounds good." Which action by the nurse will be most effective in improving the patient's oral intake? a. Order six small meals daily. b. Make a referral to the dietitian. c. Teach the patient about high-calorie foods. d. Have family members bring in favorite foods.

D

After change-of-shift report, which patient should the nurse assess first? a. 42-year-old who has acute gastritis and ongoing epigastric pain b. 70-year-old with a hiatal hernia who experiences frequent heartburn c. 53-year-old who has dumping syndrome after a recent partial gastrectomy d. 60-year-old with nausea and vomiting who has dry oral mucosa and lethargy

D This older patient is at high risk for problems such as aspiration, dehydration, and fluid and electrolyte disturbances. The other patients will also need to be assessed, but the information about them indicates symptoms that are typical for their diagnoses and are not life threatening.

Which of the following would indicate that Bobby's appendix has ruptured? a) diaphoresis b) anorexia c) pain at Mc Burney's point d) relief from pain

D all are normal signs of having appendicits and once you have relief from pain means you could have a rupture

The nurse has administered an antibiotic, a proton pump inhibitor, and Pepto- Bismol for peptic ulcer disease secondary to H. pylori. Which data would indicate to the nurse the medications are effective? A. A decrease in alcohol intake B. Maintaining a bland diet C. A return to previous activities D. A decrease in gastric distress

D. Antibiotics, proton pump inhibitors, and Pepto-Bismol are administered to decrease the irritation of the ulcerative area and cure the ulcer. A decrease in gastric distress indicates the medication is effective

A patient with a history of peptic ulcer disease has presented to the emergency department reporting severe abdominal pain and has a rigid, boardlike abdomen that prompts the health care team to suspect a perforated ulcer. What intervention should the nurse anticipate? A. Providing IV fluids and inserting a nasogastric (NG) tube B. Administering oral bicarbonate and testing the patient's gastric pH level C. Performing a fecal occult blood test and administering IV calcium gluconate D. Starting parenteral nutrition and placing the patient in a high-Fowler's position

A A patient with a history of peptic ulcer disease has presented to the emergency department reporting severe abdominal pain and has a rigid, boardlike abdomen that prompts the health care team to suspect a perforated ulcer. What intervention should the nurse anticipate?A. Providing IV fluids and inserting a nasogastric (NG) tubeB. Administering oral bicarbonate and testing the patient's gastric pH levelC. Performing a fecal occult blood test and administering IV calcium gluconateD. Starting parenteral nutrition and placing the patient in a high-Fowler's position

The nurse is providing discharge teaching for a client with newly diagnosed Crohn's disease about dietary measures to implement during exacerbation episodes. Which statement made by the client indicates a need for further instruction? A. "I should increase the fiber in my diet." B. "I will need to avoid caffeinated beverages." C. "I'm going to learn some stress reduction techniques." D. "I can have exacerbations and remissions with Crohn's disease."

A Crohn's disease is an inflammatory disease that can occur anywhere in the gastrointestinal tract but most often affects the terminal ileum and leads to thickening and scarring, a narrowed lumen, fistulas, ulcerations, and abscesses. It is characterized by exacerbations and remissions. If stress increases the symptoms of the disease, the client is taught stress management techniques and may require additional counseling. The client is taught to avoid gastrointestinal stimulants containing caffeine and to follow a high-calorie and high-protein diet. A low-fiber diet may be prescribed, especially during periods of exacerbation.

The nurse is providing dietary teaching for a client with a diagnosis of chronic gastritis. The nurse instructs the client to include which foods rich in vitamin B12 in the diet? Select all that apply. A. Nuts B. Corn C. Liver D. Apples E. Lentils F. Bananas

ACE Chronic gastritis causes deterioration and atrophy of the lining of the stomach, leading to the loss of function of the parietal cells. The source of intrinsic factor is lost, which results in an inability to absorb vitamin B12, leading to development of pernicious anemia. Clients must increase their intake of vitamin B12 by increasing consumption of foods rich in this vitamin, such as nuts, organ meats, dried beans, citrus fruits, green leafy vegetables, and yeast.

A nurse should plan to implement which interventions for a child admitted with inorganicfailure to thrive? Select all that apply. a. Observation of parent-child interactions b. Assignment of different nurses to care for the child from day to day c. Use of 28 calorie per ounce concentrated formulas d. Administration of daily multivitamin supplements e. Role modeling appropriate adult-child interactions

ADE

A patient has a vagotomy with antrectomy to treat a duodenal ulcer. Postoperatively, the patient develops dumping syndrome. Which of the following statements, if made by the patient, should indicate to the nurse that further dietary teaching is needed? 1. I should eat bread with each meal 2. I should eat smaller meals more frequently. 3. I should lie down after eating. 4. I should avoid drinking fluids with my meals

Answer 1, Patient should decrease intake of carbohydrates

Which of the following drugs is a histamine blocker and reduces levels of gastric acid? A. Omeprazole (Prilosec) B. Metoclopramide (Reglan) C. Cimetidine (Tagamet) D. Magnesium Hydroxide (Maalox)

Answer C, Cimetidine bind to H2 in the tissue and decreases the production of gastric acid

Which of the following best describes the method of action of medications, such as ranitidine (Zantac), which are used in the treatment of peptic ulcer disease? a. Neutralize acid b. Reduce acid secretions c. Stimulate gastrin release d. Protect the mucosal barrier

B

The nurse is caring for a client prescribed enteral feeding via a newly inserted nasogastric (NG) tube. Before initiating the enteral feeding, the nurse should perform which action first? A. Warm the feeding to 103°F (39.4°C). B. Confirm NG placement by x-ray study. C. Make sure the continuous enteral feeding tubing is primed. D. Position the head of the client's bed to 30 degrees or greater.

B Before initiating enteral feedings via a newly inserted NG tube, the placement of the tube is confirmed by x-ray. If the tube is not in the stomach, the client is at risk for aspiration. Formulas are administered at room temperature, not at 103°F. To prevent aspiration while administering a tube feeding, the nurse should place the client in an upright sitting position or elevate the head of the bed at least 30 degrees. Although an important action, it is not the priority. Priming the enteral feeding tube is important prior to initiating the feedings; however, it is not the priority action.

The nurse is providing instructions to a client with a colostomy about measures to reduce the odor from the colostomy. Which client statement indicates that the educational session was effective? A. "I should be sure to eat at least 1 cucumber every day." B. "Beet greens, parsley, or yogurt will help to control the colostomy odor." C. "I will need to increase my egg intake and try to eat ½ to 1 egg per day." D. "Green vegetables such as spinach and broccoli will prevent odor, and I should eat these foods every day."

B The client should be taught to include deodorizing foods in the diet, such as beet greens, parsley, buttermilk, and yogurt. Spinach also may reduce odor, but it is a gas-forming food and should be avoided. Cucumbers, eggs, and broccoli also are gas-forming foods and should be avoided or limited by the client.

A client with gastritis asks the nurse at a screening clinic about analgesics that will not cause epigastric distress. The nurse should tell the client that which medication is unlikely to cause epigastric distress? A. Ibuprofen B. Indomethacin C. Acetaminophen D. Naproxen sodium

C Analgesics, such as acetaminophen, are unlikely to cause epigastric distress. Ibuprofen, indomethacin, and naproxen sodium are nonsteroidal antiinflammatory medications (NSAIDs) and are irritating to the gastrointestinal tract, so they should be avoided in clients with gastritis.

The results of a patient's recent endoscopy indicate the presence of peptic ulcer disease (PUD). Which of the following teaching points should the nurse provide to the patient in light of his new diagnosis? A. You'll need to drink at least two to three glasses of milk daily. B."It would likely be beneficial for you to eliminate drinking alcohol." C. Many people find that a minced or pureed diet eases their symptoms of PUD. D. Your medications should allow you to maintain your present diet while minimizing symptoms

CORRECT ANSWER: B Although there is no specific recommended dietary modification for PUD, most patients find it necessary to make some sort of dietary modifications to minimize symptoms. Milk may exacerbate PUD and alcohol is best avoided because it can delay healing

A client is admitted to the hospital after vomiting bright red blood and is diagnosed with a bleeding duodenal ulcer. The client develops a sudden, sharp pain in the mid epigastric area along with a rigid, board-like abdomen. These clinical manifestations most likely indicate which of the following? a. An intestinal obstruction has developed b. Additional ulcers have developed c. The esophagus has become inflamed d. The ulcer has perforated

D

You're collecting a patient's medication history that has GERD. Which medication below is NOT typically used to treat GERD? A. Colesevelam "Welchol" B. Omeprazole "Prilosec" C. Metoclopramide "Reglan" D. Ranitidine HCL "Zantac"

A

n planning care for the newly admitted pt with acute pancreatitis, which pt outcome should receive highest priority? a. pt expresses satisfaction with pain control b. pt verbalizes understanding of medications for home c. pt increases activity intolerance d. pt maintains normal bowl function

A

Which laboratory data indicate the client's pancreatitis is improving? 1. The amylase and lipase serum levels are decreased. 2. The white blood cell count (WBC) is decreased. 3. The conjugated and unconjugated bilirubin levels are decreased. 4. The blood urea nitrogen (BUN) serum level is decreased.

1

The nurse is monitoring a client diagnosed with appendicitis who is scheduled for surgery in 2 hours. The client begins to complain of increased abdominal pain and begns to vomit. On assessment, the nurse notes that the abdomen is distended and bowel sounds are diminished. Which is the appropriate nursing intervention? 1. Notify the Physician 2. Administer the prescribed pain medication 3. Call and ask the operating room team to perform the surgery as soon as possible 4. Reposition the client and apply a heating pad on warm setting to the client's abdomen

1 Based on the assessment information the nurse should suspect peritonitis, a complication that is associated with appendicitis, and notify the physician.

the nurse is developing a plan of care for a client who has a dx of acute pancreatitis. the rationale for maintaining the client on bed rest is to: 1. reduce pancreatic and gastric secretions 2. minimize the effects of hypoglycemia 3. reduce the risk of DVT 4. decrease the likelihood of orthostatic hypotension

1 bed rest decreases body metabolism and thus reduces pancreatic and gastric secretions in the client with acute pancreatitis

the nurse is administering a pancreatic enzyme to the client dx with chronic pancreatitis. which statement best explains the rationale for administering this med? 1. it is an exogenous source of protease, amylase, and lipase 2. this enzyme increases the number of bowel movements 3. this medication breaks down in the stomach to help with digestion 4. pancreatic enzymes help break down fat in the small intestine

1 pancreatic enzymes enhance the digestion of starches (carbohydrates) in the gastrointestinal tract by supplying an exogenous (outside) source of the pancreatic enzymes protease, amylase and lipase

a nurse is doing preop teaching for a client who will have a cholecystectomy. the nurse teaches that special care must be taken to prevent what postop complication? 1. hypostatic pneumonia 2. thrombosis 3. hemorrhage 4. paralytic ileus

1 the incision is located below the ribcage; therefore the coughing and deep breathing necessary to prevent pneumonia are especially difficult post op

A client with ulcerative colitis is to take sulfasalazine (Azulfidine). Which of the following instructions should the nurse provide for the client about taking this medication at home? Select all that apply. 1. Drink enough fluids to maintain a urine output of at least 1,200- 1,500 mL per day. 2. Discontinue therapy if symptoms of acute intolerance develop and notify the health care provider. 3. Stop taking the medication if the urine turns orange-yellow. 4. Avoid activities that require alertness. 5. If dose is missed, skip and continue with the next dose.

1, 2, 4. Sulfasalazine may cause dizziness and the nurse should caution the client to avoid driving or other activities that require alertness until response to medication is known. If symptoms of acute intolerance (cramping, acute abdominal pain, bloody diarrhea, fever, headache, rash) occur, the client should discontinue therapy and notify the health care provider immediately. Fluid intake should be sufficient to maintain a urine output of at least 1,200- 1,500 mL daily to prevent crystalluria and stone formation. The nurse can also inform the client that this medication may cause orange-yellow discoloration of urine and skin, which is not significant and does not require the client to stop taking the medication. The nurse should instruct the client to take missed doses as soon as remembered unless it is almost time for the next dose.

The nurse is caring for a client who has just had an upper GI endoscopy. The client's vital signs must be taken every 30 minutes for 2 hours after the procedure. The nurse assigns an unlicensed nursing personnel (UAP) to take the vital signs. One hour later, the UAP reports the client, who was previously afebrile, has developed a temperature of 101.8 ° F (38.8 ° C). What should the nurse do in response to this reported assessment data? 1. Promptly assess the client for potential perforation. 2. Tell the assistant to change thermometers and retake the temperature. 3. Plan to give the client acetaminophen (Tylenol) to lower the temperature. 4. Ask the assistant to bathe the client with tepid water.

1. A sudden spike in temperature following an endoscopic procedure may indicate perforation of the GI tract. The nurse should promptly conduct a further assessment of the client, looking for further indicators of perforation, such as a sudden onset of acute upper abdominal pain; a rigid, boardlike abdomen; and developing signs of shock. Telling the assistant to change thermometers is not an appropriate action and only further delays the appropriate action of assessing the client. The nurse would not administer acetaminophen without further assessment of the client or without a physician's order; a suspected perforation would require that the client be placed on nothing-by-mouth status. Asking the assistant to bathe the client before any assessment by the nurse is inappropriate.

The physician orders intestinal decompression with a Cantor tube for a client with an intestinal obstruction. In order to determine effectiveness of intestinal decompression the nurse should evaluate the client to determine if: 1. Fluid and gas have been removed from the intestine. 2. The client has had a bowel movement. 3. The client's urinary output is adequate. 4. The client can sit up without pain.

1. Intestinal decompression is accomplished with a Cantor, Harris, or Miller-Abbott tube. These 6- to 10-foot tubes are passed into the small intestine to the obstruction. They remove accumulated fluid and gas, relieving the pressure. The client will not have an adequate bowel movement until the obstruction is removed. The pressure from the distended intestine should not obstruct urinary output. While the client may be able to more easily sit up, and the pain caused by the intestinal pressure will be less, these are not the primary indicators for successful intestinal decompression.

The nurse instructs the client on health maintenance activities to help control symptoms from a hiatal hernia. Which of the following statements would indicate that the client has understood the instructions? 1. "I'll avoid lying down after a meal." 2. "I can still enjoy my potato chips and cola at bedtime." 3. "I wish I didn't have to give up swimming." 4. "If I wear a girdle, I'll have more support for my stomach."

1. A client with a hiatal hernia should avoid the recumbent position immediately after meals to minimize gastric reflux. Bedtime snacks, as well as high fat foods and carbonated beverages should be avoided.

The client attends two sessions with the dietitian to learn about diet modifications to minimize GERD. The teaching would be considered successful if the client decreases the intake of which of the following foods? 1. fats 2. high-sodium foods 3. Carbohydrates 4. high calcium foods

1. Fats are associated with decreased esophageal sphincter tone.

The physician prescribes metoclopramide hyrochloide (Reglan) for the client with a hiatal hernia. This drug is used in hiatal hernia therapy to accomplish which of the following objectives? 1. Increase tone of the esophageal sphincter 2. Neutralize gastric secretions 3. delay gastric emptying 4. reduce secretion of digestive juices

1. This medicaiton increases esophageal sphincter tone and facilitates gastric emptying; both actions reduce the incidence of reflux.

A nurse is caring for a child who had a laproscopic appendectomy. What interventions should the nurse document on the child's clinical record? Select all that apply. 1) Intake and Output 2) Measurement of Pain 3) Tolerance to low-residue diet 4) Frequency of dressing changes 5) Auscultation of bowel sounds

125

a 32 y/o client had a cholecystectomy 2 hrs ago. the client weighs 250 lbs and had general anesthesia. the client is c/o pain. priority nursing dx at this time: 1. alteration in comfort 2. high risk for airway impairment 3. alteration in fluid volume 4. knowledge deficit

2

A client with peptic ulcer disease reports that he has been nauseated most of the day and is now feeling light-headed and dizzy. Based upon these findings, which nursing actions would be most appropriate for the nurse to take? Select all that apply. 1. Administering an antacid hourly until nausea subsides. 2. Monitoring the client's vital signs. 3. Notifying the physician of the client's symptoms. 4. Initiating oxygen therapy. 5. Reassessing the client in an hour.

2, 3. The symptoms of nausea and dizziness in a client with peptic ulcer disease may be indicative of hemorrhage and should not be ignored. The appropriate nursing actions at this time are for the nurse to monitor the client's vital signs and notify the physician of the client's symptoms. To administer an antacid hourly or to wait 1 hour to reassess the client would be inappropriate; prompt intervention is essential in a client who is potentially experiencing a gastrointestinal hemorrhage. The nurse would notify the physician of assessment findings and then initiate oxygen therapy if ordered by the physician.

The nurse is caring for a client who has had a gastroscopy. Which of the following signs and symptoms may indicate that the client is developing a complication related to the procedure? Select all that apply. 1. The client has a sore throat. 2. The client has a temperature of 100 ° F (37.8 ° C). 3. The client appears drowsy following the procedure. 4. The client has epigastric pain. 5. The client experiences hematemesis.

2, 4, 5. Following a gastroscopy, the nurse should monitor the client for complications, which include perforation and the potential for aspiration. An elevated temperature, complaints of epigastric pain, or the vomiting of blood (hematemesis) are all indications of a possible perforation and should be reported promptly. A sore throat is a common occurrence following a gastroscopy. Clients are usually sedated to decrease anxiety and the nurse would anticipate that the client will be drowsy following the procedure.

When planning care for a client with ulcerative colitis who is experiencing an exacerbation of symptoms, which client care activities can the nurse appropriately delegate to an unlicensed assistant? Select all that apply. 1. Assessing the client's bowel sounds. 2. Providing skin care following bowel movements. 3. Evaluating the client's response to antidiarrheal medications. 4. Maintaining intake and output records. 5. Obtaining the client's weight.

2, 4, 5. The nurse can delegate the following basic care activities to the unlicensed assistant: providing skin care following bowel movements, maintaining intake and output records, and obtaining the client's weight. Assessing the client's bowel sounds and evaluating the client's response to medication are registered nurse activities that cannot be delegated.

The nurse is developing a plan of care for a client with Crohn's disease who is receiving total parenteral nutrition (TPN). Which of the following interventions should the nurse include? Select all that apply. 1. Monitoring vital signs once a shift. 2. Weighing the client daily. 3. Changing the central venous line dressing daily. 4. Monitoring the I.V. infusion rate hourly. 5. Taping all I.V. tubing connections securely.

2, 4, 5. When caring for a client who is receiving TPN, the nurse should plan to weigh the client daily, monitor the I.V. fluid infusion rate hourly (even when using an I.V. fluid pump), and securely tape all I.V. tubing connections to prevent disconnections. Vital signs should be monitored at least every 4 hours to facilitate early detection of complications. It is recommended that the I.V. dressing be changed once or twice per week or when it becomes soiled, loose, or wet.

The nurse is assigning clients for the evening shift. Which of the following clients are appropriate for the nurse to assign to a licensed practical nurse to provide client care? Select all that apply. 1. A client with Crohn's disease who is receiving total parenteral nutrition (TPN). 2. A client who underwent inguinal hernia repair surgery 3 hours ago. 3. A client with an intestinal obstruction who needs a Cantor tube inserted. 4. A client with diverticulitis who needs teaching about his take-home medications. 5. A client who is experiencing an exacerbation of his ulcerative colitis.

2, 5. The nurse should consider client needs and scope of practice when assigning staff to provide care. The client who is recovering from inguinal hernia repair surgery and the client who is experiencing an exacerbation of his ulcerative colitis are appropriate clients to assign to a licensed practical nurse as the care they require fall within the scope of practice for a licensed practical nurse. It is not within the scope of practice for the licensed practical nurse to administer TPN, insert nasoenteric tubes, or provide client teaching related to medications.

Which of the following should be a priority focus of care for a client experiencing an exacerbation of Crohn's disease? 1. Encouraging regular ambulation. 2. Promoting bowel rest. 3. Maintaining current weight. 4. Decreasing episodes of rectal bleeding.

2. A priority goal of care during an acute exacerbation of Crohn's disease is to promote bowel rest. This is accomplished through decreasing activity, encouraging rest, and initially placing client on nothing-by-mouth status while maintaining nutritional needs parenterally. Regular ambulation is important, but the priority is bowel rest. The client will probably lose some weight during the acute phase of the illness. Diarrhea is nonbloody in Crohn's disease, and episodes of rectal bleeding are not expected.

The nurse finds a client who has been diagnosed with a peptic ulcer surrounded by papers from his briefcase and arguing on the telephone with a coworker. The nurse's response to observing these actions should be based on knowledge that: 1. Involvement with his job will keep the client from becoming bored. 2. A relaxed environment will promote ulcer healing. 3. Not keeping up with his job will increase the client's stress level. 4. Setting limits on the client's behavior is an important nursing responsibility.

2. A relaxed environment is an essential component of ulcer healing. Nurses can help clients understand the importance of relaxation and explore with them ways to balance work and family demands to promote healing. Being involved with his work may prevent boredom; however, this client is upset and argumentative. Not keeping up with his job will probably increase the client's stress level, but the nurse's response is best if it is based on the fact that a relaxed environment is an essential component of ulcer healing. Nurses cannot set limits on a client's behavior; clients must make the decision to make lifestyle changes.

Which of the following would be an expected outcome for a client with peptic ulcer disease? The client will: 1. Demonstrate appropriate use of analgesics to control pain. 2. Explain the rationale for eliminating alcohol from the diet. 3. Verbalize the importance of monitoring hemoglobin and hematocrit every 3 months. 4. Eliminate contact sports from his or her lifestyle.

2. Alcohol is a gastric irritant that should be eliminated from the intake of the client with peptic ulcer disease. Analgesics are not used to control ulcer pain; many analgesics are gastric irritants. The client's hemoglobin and hematocrit typically do not need to be monitored every 3 months, unless gastrointestinal bleeding is suspected. The client can maintain an active lifestyle and does not need to eliminate contact sports as long as they are not stress-inducing.

the nurse reinforces teaching for a pt after a cholecystectomy on a low fat diet. the nurse will know that the pt understands the diet if which of the following menu items is selected? a. roast chicken rice gelatin dessert b. cream of chicken soup, milk, gelatin dessert c. meat loaf, mashed potatoes with small amount of gravy, green beans d. turkey and cheese sandwich on whole grain bread, milk

A

A client with peptic ulcer disease tells the nurse that he has black stools, which he has not reported to his physician. Based on this information, which nursing diagnosis would be appropriate for this client? 1. Ineffective coping related to fear of diagnosis of chronic illness. 2. Deficient knowledge related to unfamiliarity with significant signs and symptoms. 3. Constipation related to decreased gastric motility. 4. Imbalanced nutrition: Less than body requirements related to gastric bleeding.

2. Black, tarry stools are an important warning sign of bleeding in peptic ulcer disease. Digested blood in the stool causes it to be black. The odor of the stool is very offensive. Clients with peptic ulcer disease should be instructed to report the incidence of black stools promptly to their primary health care provider. The data do not support the other diagnoses.

Which of the following diets would be most appropriate for the client with ulcerative colitis? 1. High-calorie, low-protein. 2. High-protein, low-residue. 3. Low-fat, high-fiber. 4. Low-sodium, high-carbohydrate.

2. Clients with ulcerative colitis should follow a well-balanced high-protein, high-calorie, low-residue diet, avoiding such high-residue foods as whole-wheat grains, nuts, and raw fruits and vegetables. Clients with ulcerative colitis need more protein for tissue healing and should avoid excess roughage. There is no need for clients with ulcerative colitis to follow low-sodium diets.

Which goal for the client's care should take priority during the first days of hospitalization for an exacerbation of ulcerative colitis? 1. Promoting self-care and independence. 2. Managing diarrhea. 3. Maintaining adequate nutrition. 4. Promoting rest and comfort.

2. Diarrhea is the primary symptom in an exacerbation of ulcerative colitis, and decreasing the frequency of stools is the first goal of treatment. The other goals are ongoing and will be best achieved by halting the exacerbation. The client may receive antidiarrheal agents, antispasmodic agents, bulk hydrophilic agents, or anti-inflammatory drugs.

A client has been placed on long-term sulfasalazine (Azulfidine) therapy for treatment of his ulcerative colitis. The nurse should encourage the client to eat which of the following foods to help avoid the nutrient deficiencies that may develop as a result of this medication? 1. Citrus fruits. 2. Green, leafy vegetables. 3. Eggs. 4. Milk products.

2. In long-term sulfasalazine therapy, the client may develop folic acid deficiency. The client can take folic acid supplements, but the nurse should also encourage the client to increase the intake of folic acid in his diet. Green, leafy vegetables are a good source of folic acid. Citrus fruits, eggs, and milk products are not good sources of folic acid.

A client with Crohn's disease has concentrated urine, decreased urinary output, dry skin with decreased turgor, hypotension, and weak, thready pulses. The nurse should do which of the following first? 1. Encourage the client to drink at least 1,000 mL per day. 2. Provide parenteral rehydration therapy ordered by the physician. 3. Turn and reposition every 2 hours. 4. Monitor vital signs every shift.

2. Initially, the extracellular fluid (ECF) volume with isotonic I.V. fluids until adequate circulating blood volume and renal perfusion are achieved. Vital signs should be monitored as parenteral and oral rehydration are achieved. Oral fluid intake should be greater than 1,000 mL/ day. Turning and repositioning the client at regular intervals aids in the prevention of skin breakdown, but it is first necessary to rehydrate this client.

After insertion of a nasoenteric tube, the nurse should place the client in which position? 1. Supine. 2. Right side-lying. 3. Semi-Fowler's. 4. Upright in a bedside chair.

2. The client is placed in a right side-lying position to facilitate movement of the mercury-weighted tube through the pyloric sphincter. After the tube is in the intestine, the client is turned from side to side or encouraged to ambulate to facilitate tube movement through the intestinal loops. Placing the client in the supine or semi-Fowler's position, or having the client sitting out of bed in a chair will not facilitate tube progression.

Which of the following factors would most likely contribute to the development of a client's hiatal hernia? 1. having a sedentary desk job 2. being 5 feet, 3 inches tall and weighing 190 lbs 3. using laxatives frequently 4. being 40 years old

2. Any factor that increases intra-abdominal pressure, such as obesity, can contribute to the development of hiatal hernia. Other factors include abdominal straining, frequent heavy lifting, and pregnancy. Hiatal hernia is also associated with older age and occurs in women more frequently than in men.

The client asks the nurse if surgery is needed to correct a hiatal hernia. Which reply by the nurse would be the MOST accurate? 1. "Surgery is usually required, although medical treatment is attempted first." 2. "Hiatal hernia symptoms can usually be successfully managed with diet modification, medications, and lifestyle changes." 3. "Surgery is not performed for this type of hernia." 4. "A minor surgical procedure to reduce the size of the diaphragmatic opening will probably be planned."

2. Most clients can be treated successfully with a combination of diet restrictions, medications, weight control, and lifestyle modifications. Surgery is performed only when these attempts fail.

Bethanechol (Urecholine) has been prescribed for a client with GERD. The nurse should assess the client for which of the following adverse effects? 1. Constipation 2. Urinary urgency 3. Hypertension 4. Dry oral mucosa

2. This is a cholinergic drug that may be used in GERD to increase lower esophageal sphincter pressure and facilitate gastric emptying. Cholinergic adverse effects may include urinary urgency, diarrhea, abdominal cramping, hypotension, and increased salivation.

A nurse is monitoring the status of a client's fat emulsion (lipid) infusion and notes that the infusion is 2 hours delay. The nurse should do which of the following actions? A. Adjust the infusion rate to catch up over the next hour. B. Make sure the infusion rate is infusing at the ordered rate. C .Increase the infusion rate to catch up over the next few hours. D. Adjust the infusion rate to full blast until the solution is back on time.

B

The client has had a liver biopsy. which postprocedure intervention should the nurse implement? 1. instruct the client to void immediatly 2. keep the client NPO for 8 hours 3. place the client on the right side 4. monitor BUN and creatinine level

3

The client is being admitted to the outpatient department prior to an endoscopicretrograde cholangiopancreatogram (ERCP) to rule out cancer of the pancreas. Whichpre-procedure instruction should the nurse teach? 1. Prepare to be admitted to the hospital after the procedure for observation. 2. If something happens during the procedure, then emergency surgery will be done. 3. Do not eat or drink anything after midnight the night before the test. 4. If done correctly, this procedure will correct the blockage of the stomach.

3

The nurse is admitting a client with acute appendicitis to the emergency department. The client has abdominal pain of 10 on a pain scale of 1 to 10. The client will be going to surgery as soon as possible. The nurse should: 1. Contact the surgeon to request an order for a narcotic for the pain. 2. Maintain the client in a recumbent position. 3. Place the client on nothing-by-mouth (NPO) status. 4. Apply heat to the abdomen in the area of the pain.

3 The nurse should place the client on NPO status in anticipation of surgery. The nurse can initiate pain relief strategies, such as relaxation techniques, but the surgeon will likely not order narcotic medication prior to surgery. The nurse can place the client in a position that is most comfortable for the client. Heat is contraindicated because it may lead to perforation of the appendix.

the client dx with end stage renal failure and ascites is scheduled for a paracentesis. which client teaching should the nurse discuss with the client? 1. explain the procedure will be done in the operating room 2. instruct the client a foley cath will have to be inserted 3. tell the client v/s will be taken frequently after the procedure 4. provide instructions on holding the breath when the hcp inserts the cath

3 hot water increases pruritus and soap will cause dry skin, which increases pruritus, therefore, the nurse should discuss this with the UAP

the male client dx with chronic pancreatitis calls and reports to the clinic nurse he has been having a lot of gas along with frothy and very foul smelling stools. which intervention should the nurse implement? 1. explain this is common for chronic pancreatitis 2. ask the client to bring in a stool specimen to the clinic 3. arrange an appointment with the hcp for today 4. discuss the need to decrease fat in the diet so this wont happen

3 steatorrhea (fatty, frothy, foul smelling stool) is caused by a decrease in pancreatic enzyme secretion and indicates impaired digestion and possibly an increase in the severity of the pancreatitis. the client should see the hcp

The nurse has been assigned to provide care for four clients at the beginning of the day shift. In what order should the nurse assess these clients? 1. The client awaiting hiatal hernia repair at 11 am. 2. A client with suspected gastric cancer who is on nothing-by-mouth (NPO) status for tests. 3. A client with peptic ulcer disease experiencing sudden onset of acute stomach pain. 4. A client who is requesting pain medication 2 days after surgery to repair a fractured jaw.

3, 4, 2, 1 The client with peptic ulcer disease who is experiencing a sudden onset of acute stomach pain should be assessed first by the nurse. The sudden onset of stomach pain could be indicative of a perforated ulcer, which would require immediate medical attention. It is also important for the nurse to thoroughly assess the nature of the client's pain. The client with the fractured jaw is experiencing pain and should be assessed next. The nurse should then assess the client who is NPO for tests to ensure NPO status and comfort. Last, the nurse can assess the client before surgery.

When obtaining a nursing history on a client with a suspected gastric ulcer, which signs and symptoms should the nurse expect to assess? Select all that apply. 1. Epigastric pain at night. 2. Relief of epigastric pain after eating. 3. Vomiting. 4. Weight loss. 5. Melena.

3, 4, 5. Vomiting and weight loss are common with gastric ulcers. The client may also have blood in the stools (melena) from gastric bleeding. Clients with a gastric ulcer are most likely to complain of a burning epigastric pain that occurs about 1 hour after eating. Eating frequently aggravates the pain. Clients with duodenal ulcers are more likely to complain about pain that occurs during the night and is frequently relieved by eating.

A client's ulcerative colitis signs and symptoms have been present for longer than 1 week. The nurse should assess the client for signs and symptoms of which of the following complications? 1. Heart failure. 2. Deep vein thrombosis. 3. Hypokalemia. 4. Hypocalcemia.

3. Excessive diarrhea causes significant depletion of the body's stores of sodium and potassium as well as fluid. The client should be closely monitored for hypokalemia and hyponatremia. Ulcerative colitis does not place the client at risk for heart failure, deep vein thrombosis, or hypocalcemia.

A client who has ulcerative colitis has persistent diarrhea. He is thin and has lost 12 lb since the exacerbation of his ulcerative colitis. Which of the following will be most effective in helping the client meet his nutritional needs? 1. Continuous enteral feedings. 2. Following a high-calorie, high-protein diet. 3. Total parenteral nutrition (TPN). 4. Eating six small meals a day.

3. Food will be withheld from the client with severe symptoms of ulcerative colitis to rest the bowel. To maintain the client's nutritional status, the client will be started on TPN. Enteral feedings or dividing the diet into six small meals does not allow the bowel to rest. A high-calorie, high-protein diet will worsen the client's symptoms.

A client who has a history of Crohn's disease is admitted to the hospital with fever, diarrhea, cramping, abdominal pain, and weight loss. The nurse should monitor the client for: 1. Hyperalbuminemia. 2. Thrombocytopenia. 3. Hypokalemia. 4. Hypercalcemia.

3. Hypokalemia is the most expected laboratory finding owing to the diarrhea. Hypoalbuminemia can also occur in Crohn's disease; however, the client's potassium level is of greater importance at this time because a low potassium level can cause cardiac arrest. Anemia is an expected development, but thrombocytopenia is not. Calcium levels are not affected.

A client has been taking aluminum hydroxide (Amphojel) 30 mL six times per day at home to treat his peptic ulcer. He tells the nurse that he has been unable to have a bowel movement for 3 days. Based on this information, the nurse would determine that which of the following is the most likely cause of the client's constipation? 1. The client has not been including enough fiber in his diet. 2. The client needs to increase his daily exercise. 3. The client is experiencing an adverse effect of the aluminum hydroxide. 4. The client has developed a gastrointestinal obstruction.

3. It is most likely that the client is experiencing an adverse effect of the antacid. Antacids with aluminum salt products, such as aluminum hydroxide, form insoluble salts in the body. These precipitate and accumulate in the intestines, causing constipation. Increasing dietary fiber intake or daily exercise may be a beneficial lifestyle change for the client but is not likely to relieve the constipation caused by the aluminum hydroxide. Constipation, in isolation from other symptoms, is not a sign of a bowel obstruction.

A client is to take one daily dose of ranitidine (Zantac) at home to treat her peptic ulcer. The client understands proper drug administration of ranitidine when she says that she will take the drug at which of the following times? 1. Before meals. 2. With meals. 3. At bedtime. 4. When pain occurs.

3. Ranitidine blocks secretion of hydrochloric acid. Clients who take only one daily dose of ranitidine are usually advised to take it at bedtime to inhibit nocturnal secretion of acid. Clients who take the drug twice a day are advised to take it in the morning and at bedtime. It is not necessary to take the drug before meals. The client should take the drug regularly, not just when pain occurs.

A client who has been diagnosed with GERD has heartburn. To decrease the heartburn, the nurse should instruct the client to eliminate which of the following items from the diet? 1. Lean beef 2. Air-popped popcorn 3. Hot chocolate 4. Raw vegetables

3. With GERD, eating substances that decrease lower esophageal sphincter pressure causes heartburn. A decrease in the lower esophageal sphincter pressure allows gastric contents to reflux into the lower end of the esophagus. Foods that can cause a decrease in esophageal sphincter pressure include fatty foods, chocolate, caffeinated beverages, peppermint, and alcohol.

A client with acute appendicitis develops a fever, tachycardia, and hypotension. Based on these assessment findings, the nurse should further assess the client for which of the following complications? 1. Deficient fluid volume. 2. Intestinal obstruction. 3. Bowel ischemia. 4. Peritonitis.

4 Complications of acute appendicitis are perforation, peritonitis, and abscess development. Signs of the development of peritonitis include abdominal pain and distention, tachycardia, tachypnea, nausea, vomiting, and fever. Because peritonitis can cause hypovolemic shock, hypotension can develop. Deficient fluid volume would not cause a fever. Intestinal obstruction would cause abdominal distention, diminished or absent bowel sounds, and abdominal pain. Bowel ischemia has signs and symptoms similar to those found with intestinal obstruction.

A client has undergone esophagogastroduodenoscopy. The nurse should place highest priority on which item as part of the client's care plan? 1. Monitoring the temperature 2. Monitoring complaints of heartburn 3. Giving warm gargles for a sore throat 4. Assessing for the return of the gag reflex

4 The nurse places highest priority on assessing for return of the gag reflex. This assessment addresses the client's airway. The nurse also monitors the client's vital signs and for a sudden increase in temperature, which could indicate perforation of the gastrointestinal tract. This complication would be accompanied by other signs as well, such as pain. Monitoring for sore throat and heartburn are also important; however, the client's airway is the priority.

the client is dx with acute pancreatitis. which health care providers admitting order should the nurse question? 1. bedrest with bathroom privleges. 2. initiate iv therapy of D5W at 125 mL/hr 3. weigh client daily 4. low fat, low carb diet

4 the client will be NPO, which will decrease stimulation of the pancreatic enzymes, resulting in decreased autodigestion of the pancreas, therefore decreasing pain

A patient with Crohn's Disease is taking corticosteroids. The patient is complaining of extreme thirst, polyuria, and blurred vision. What is your next nursing action? A. Check the patient's blood glucose B. Give the patient a food containing sugar (ex: orange juice) C. Administer oxygen via nasal cannula D. Assess bowel sounds

A

A client is taking an antacid for treatment of a peptic ulcer. Which of the following statements best indicates that the client understands how to correctly take the antacid? 1. "I should take my antacid before I take my other medications." 2. "I need to decrease my intake of fluids so that I don't dilute the effects of my antacid." 3. "My antacid will be most effective if I take it whenever I experience stomach pains." 4. "It is best for me to take my antacid 1 to 3 hours after meals."

4. Antacids are most effective if taken 1 to 3 hours after meals and at bedtime. When an antacid is taken on an empty stomach, the duration of the drug's action is greatly decreased. Taking antacids 1 to 3 hours after a meal lengthens the duration of action, thus increasing the therapeutic action of the drug. Antacids should be administered about 2 hours after other medications to decrease the chance of drug interactions. It is not necessary to decrease fluid intake when taking antacids. If antacids are taken more frequently than recommended, the likelihood of developing adverse effects increases. Therefore, the client should not take antacids as often as desired to control pain.

A patient, who has recovered from cholecystitis, is being discharged home. What meal options below are best for this patient? A. Baked chicken with steamed carrots and rice B. Broccoli and cheese casserole with gravy and mashed potatoes C. Cheeseburger with fries D. Fried chicken with a baked potato

A

A client who has ulcerative colitis says to the nurse, "I can't take this anymore! I'm constantly in pain, and I can't leave my room because I need to stay by the toilet. I don't know how to deal with this." Based on these comments, an appropriate nursing diagnosis for this client would be: 1. Impaired physical mobility related to fatigue. 2. Disturbed thought processes related to pain. 3. Social isolation related to chronic fatigue. 4. Ineffective coping related to chronic abdominal pain.

4. It is not uncommon for clients with ulcerative colitis to become apprehensive and upset about the frequency of stools and the presence of abdominal cramping. During these acute exacerbations, clients need emotional support and encouragement to verbalize their feelings about their chronic health concerns and assistance in developing effective coping methods. The client has not expressed feelings of fatigue or isolation or demonstrated disturbed thought processes.

A client with a peptic ulcer has been instructed to avoid intense physical activity and stress. Which strategy should the client incorporate into the home care plan? 1. Conduct physical activity in the morning so that he can rest in the afternoon. 2. Have the family agree to perform the necessary yard work at home. 3. Give up jogging and substitute a less demanding hobby. 4. Incorporate periods of physical and mental rest in his daily schedule.

4. It would be most effective for the client to develop a health maintenance plan that incorporates regular periods of physical and mental rest in the daily schedule. Strategies should be identified to deal with the types of physical and mental stressors that the client needs to cope with in the home and work environments. Scheduling physical activity to occur only in the morning would not be restful or practical. There is no need for the client to avoid yard work or jogging if these activities are not stressful.

A client is admitted to the hospital after vomiting bright red blood and is diagnosed with a bleeding duodenal ulcer. The client develops a sudden, sharp pain in the midepigastric region along with a rigid, boardlike abdomen. These clinical manifestations most likely indicate which of the following? 1. An intestinal obstruction has developed. 2. Additional ulcers have developed. 3. The esophagus has become inflamed. 4. The ulcer has perforated.

4. The body reacts to perforation of an ulcer by immobilizing the area as much as possible. This results in boardlike abdominal rigidity, usually with extreme pain. Perforation is a medical emergency requiring immediate surgical intervention because peritonitis develops quickly after perforation. An intestinal obstruction would not cause midepigastric pain. The development of additional ulcers or esophageal inflammation would not cause a rigid, boardlike abdomen.

The client with GERD has a chronic cough. This symptom may be indicative of which of the following? 1. Development of laryngeal cancer 2. Irritation of the esophagus 3. Esophageal scar tissue formation 4. Aspiration of gastric contents

4. Clients with GERD can develop pulmonary symptoms such as coughing, wheezing, and dyspnea, that are caused by the aspiration of gastric contents.

A client is receiving parenteral nutrition (PN) suddenly is having a fever. A nurse notifies the physician and the physician initially prescribes that the solution and tubing be changed. The nurse should do which of the following with the discontinued materials? A. Send them to the laboratory for culture. B. Save them for a return to the manufacturer. C. Return them to the hospital pharmacy. D. Discard them in the unit trash.

A

A patient complains of gas pains and abdominal distention two days after a small bowel resection. Which nursing action is best to take? a. Encourage the patient to ambulate. b. Instill a mineral oil retention enema. c. Administer the ordered IV morphine sulfate. d. Offer the ordered promethazine (Phenergan) suppository.

A

A patient is being transferred to your unit with acute cholecystitis. In report the transferring nurse tells you that the patient has a positive Murphy's Sign. You know that this means: A. The patient stops breathing in when the examiner palpates under the ribs on the right upper side of the abdomen at the midclavicular line. B. The patient stops breathing out when the examiner palpates under the ribs on the right upper side of the abdomen at the midclavicular line. C. The patient verbalizes pain when the lower right quadrant is palpated. D. The patient reports pain when pressure is applied to the right lower quadrant but then reports an increase in pain intensity when the pressure is released.

A

Which patient statement indicates that the nurse's teaching following a gastroduodenostomy has been effective? a. "Vitamin supplements may prevent anemia." b. "Persistent heartburn is common after surgery." c. "I will try to drink more liquids with my meals." d. "I will need to choose high carbohydrate foods."

A Cobalamin deficiency may occur after partial gastrectomy, and the patient may need to receive cobalamin via injections or nasal spray. Although peptic ulcer disease may recur, persistent heartburn is not expected after surgery and the patient should call the health care provider if this occurs. Ingestion of liquids with meals is avoided to prevent dumping syndrome. Foods that have moderate fat and low carbohydrate should be chosen to prevent dumping syndrome.

The nurse has provided dietary instructions to a client with a diagnosis of peptic ulcer disease. Which client statement indicates that education was effective? A. "Baked foods such as chicken or fish are all right to eat." B. "Citrus fruits and raw vegetables need to be included in my daily diet." C. "I can drink beer as long as I consume only a moderate amount each day." D. "I can drink coffee or tea as long as I limit the amount to 2 cups daily."

A Dietary modifications for the client with peptic ulcer disease include eliminating foods that can cause irritation to the gastrointestinal (GI) tract. Items that should be eliminated or avoided include highly spiced foods, alcohol, caffeine, chocolate, and citrus fruits. Other foods may be taken according to the client's level of tolerance for that food.

Diphenoxylate hydrochloride with atropine sulfate is prescribed for a client with ulcerative colitis. The nurse should monitor the client for which therapeutic effect of this medication? A. Decreased diarrhea B. Decreased cramping C. Improved intestinal tone D. Elimination of peristalsis

A Diphenoxylate hydrochloride with atropine sulfate is an antidiarrheal product that decreases the frequency of defecation, usually by reducing the volume of liquid in the stools. The remaining options are not associated therapeutic effects of this medication.

A client experiencing chronic dumping syndrome makes the following comments to the nurse. Which one indicates the need for further teaching? A. "I eat at least 3 large meals each day." B. "I eat while lying in a semirecumbent position." C. "I have eliminated taking liquids with my meals." D. "I eat a high-protein, low- to moderate-carbohydrate diet."

A Dumping syndrome describes a group of symptoms that occur after eating. It is believed to result from rapid dumping of gastric contents into the small intestine, which causes fluid to shift into the intestine. Signs and symptoms of dumping syndrome include diarrhea, abdominal distention, sweating, pallor, palpitations, and syncope. To manage this syndrome, clients are encouraged to decrease the amount of food taken at each sitting, eat in a semirecumbent position, eliminate ingesting fluids with meals, and avoid consumption of high-carbohydrate meals.

The nurse is monitoring a client for the early signs and symptoms of dumping syndrome. Which findings indicate this occurrence? A. Sweating and pallor B. Bradycardia and indigestion C. Double vision and chest pain D. Abdominal cramping and pain

A Early manifestations of dumping syndrome occur 5 to 30 minutes after eating. Symptoms include vertigo, tachycardia, syncope, sweating, pallor, palpitations, and the desire to lie down.

A patient complains of nausea. When administering a dose of metoclopramide (Reglan), the nurse should teach the patient to report which potential adverse effect? A. Tremors B. Constipation C. Double vision D. Numbness in fingers and toes

A Extrapyramidal side effects, including tremors and tardive dyskinesias, may occur as a result of metoclopramide (Reglan) administration. Constipation, double vision, and numbness in fingers and toes are not adverse effects of metoclopramide.

A client who has been advanced to a solid diet after undergoing a subtotal gastrectomy. What is the appropriate nursing intervention in preventing dumping syndrome? A. Remove fluids from the meal tray. B. Give the client 2 large meals per day. C. Ask the client to sit up for 1 hour after eating. D. Provide concentrated, high-carbohydrate foods.

A Factors to minimize dumping syndrome after gastric surgery include having the client lie down for at least 30 minutes after eating; giving small, frequent meals; having the client maintain a low Fowler's position while eating, if possible; avoiding liquids with meals; and avoiding high-carbohydrate food sources. Antispasmodic medications also are prescribed as needed to delay gastric emptying.

The nurse inspects the color of the drainage from a nasogastric tube on a postoperative client approximately 24 hours after gastric surgery. Which finding indicates the need to notify the health care provider (HCP)? A. Dark red drainage B. Dark brown drainage C. Green-tinged drainage D. Light yellowish-brown drainage

A For the first 12 hours after gastric surgery, the nasogastric tube drainage may be dark brown to dark red. Later, the drainage should change to a light yellowish-brown color. The presence of bile may cause a green tinge. The HCP should be notified if dark red drainage, a sign of hemorrhage, is noted 24 hours postoperatively.

The nurse is giving dietary instructions to a client who has a new colostomy. The nurse should encourage the client to eat foods representing which diet for the first 4 to 6 weeks postoperatively? A. Low fiber B. Low calorie C. High protein D. High carbohydrate

A For the first 4 to 6 weeks after colostomy formation, the client should consume a low-fiber diet. After this period, the client should eat a high-carbohydrate, high-protein diet. The client also is instructed to add new foods, including those with fiber, one at a time to determine tolerance to that food.

The nurse provides instructions to a client about measures to treat inflammatory bowel syndrome (IBS). Which statement by the client indicates a need for further teaching? A. "I need to limit my intake of dietary fiber." B. "I need to drink plenty, at least 8 to 10 cups daily." C. "I need to eat regular meals and chew my food well." D. "I will take the prescribed medications because they will regulate my bowel patterns."

A IBS is a functional gastrointestinal disorder that causes chronic or recurrent diarrhea, constipation, and/or abdominal pain and bloating. Dietary fiber and bulk help to produce bulky, soft stools and establish regular bowel elimination habits. Therefore, the client should consume a high-fiber diet. Eating regular meals, drinking 8 to 10 cups of liquid a day, and chewing food slowly help to promote normal bowel function. Medication therapy depends on the main symptoms of IBS. Bulk-forming laxatives or antidiarrheal agents or other agents may be prescribed.

A 50-year-old patient who underwent a gastroduodenostomy (Billroth I) earlier today complains of increasing abdominal pain. The patient has no bowel sounds and 200 mL of bright red nasogastric (NG) drainage in the last hour. The highest priority action by the nurse is to a. contact the surgeon. b. irrigate the NG tube. c. monitor the NG drainage. d. administer the prescribed morphine.

A Increased pain and 200 mL of bright red NG drainage 12 hours after surgery indicate possible postoperative hemorrhage, and immediate actions such as blood transfusion and/or return to surgery are needed. Because the NG is draining, there is no indication that irrigation is needed. Continuing to monitor the NG drainage is not an adequate response. The patient may need morphine, but this is not the highest priority action.

Which action for a patient receiving tube feedings through a percutaneous endoscopic gastrostomy (PEG) may be delegated to a licensed practical/vocational nurse (LPN/LVN)? a. Providing skin care to the area around the tube site b. Teaching the patient how to administer tube feedings c. Determining the need for adding water to the feedings d. Assessing the patient's nutritional status at least weekly

A LPN/LVN education and scope of practice include actions such as dressing changes and wound care. Patient teaching and complex assessments (such as patient nutrition and hydration status) require registered nurse (RN)-level education and scope of practice.

A 48-year-old man who has just been started on tube feedings of full-strength formula at 100 mL/hr has 6 diarrhea stools the first day. Which action should the nurse plan to take? a. Slow the infusion rate of the tube feeding. b. Check gastric residual volumes more frequently. c. Change the enteral feeding system and formula every 8 hours. d. Discontinue administration of water through the feeding tube.

A Loose stools indicate poor absorption of nutrients and indicate a need to slow the feeding rate or decrease the concentration of the feeding. Water should be given when patients receive enteral feedings to prevent dehydration. When a closed enteral feeding system is used, the tubing and formula are changed every 24 hours. High residual volumes do not contribute to diarrhea.

The nurse is monitoring a client admitted to the hospital with a diagnosis of appendicitis who is scheduled for surgery in 2 hours. The client begins to complain of increased abdominal pain and begins to vomit. On assessment, the nurse notes that the abdomen is distended and bowel sounds are diminished. Which is the most appropriate nursing intervention? A. Notify the health care provider (HCP). B. Administer the prescribed pain medication. C. Call and ask the operating room team to perform surgery as soon as possible. D. Reposition the client and apply a heating pad on the warm setting to the client's abdomen.

A On the basis of the signs and symptoms presented in the question, the nurse should suspect peritonitis and notify the HCP. Administering pain medication is not an appropriate intervention. Heat should never be applied to the abdomen of a client with suspected appendicitis because of the risk of rupture. Scheduling surgical time is not within the scope of nursing practice, although the HCP probably would perform the surgery earlier than the prescheduled time.

Sulfasalazine is prescribed for a client with a diagnosis of ulcerative colitis, and the nurse instructs the client about the medication. Which statement made by the client indicates a need for further teaching? A. "The medication will cause constipation." B. "I need to take the medication with meals." C. "I may have increased sensitivity to sunlight." D. "This medication should be taken as prescribed."

A Sulfasalazine is an antiinflammatory sulfonamide. Constipation is not associated with this medication. It can cause photosensitivity, and the client should be instructed to avoid sun and ultraviolet light. It should be administered with meals, if possible, to prolong intestinal passage. The client needs to take the medication as prescribed and continue the full course of treatment even if symptoms are relieved.

What is the nurse's best action for the client who has undergone a laparoscopic cholecystectomy and complains of "free air pain." A. Ambulate the client. B. Instruct the client to breathe deeply and cough. C. Maintain the client on bedrest with his or her legs elevated. D. Insert a rectal tube to facilitate the passage of flatus.

A The client who has undergone a laparoscopic cholecystectomy may complain of free air pain because of the retention of carbon dioxide in the abdomen. The nurse assists the client with early ambulation to promote absorption of the carbon dioxide.

A patient is being scheduled for endoscopic retrograde cholangiopancreatography (ERCP) as soon as possible. Which actions from the agency policy for ERCP should the nurse take first? a. Place the patient on NPO status. b. Administer sedative medications. c. Ensure the consent form is signed. d. Teach the patient about the procedure.

A The patient will need to be NPO for 8 hours before the ERCP is done, so the nurse's initial action should be to place the patient on NPO status. The other actions can be done after the patient is NPO

A patient is receiving continuous enteral nutrition through a small-bore silicone feeding tube. What should the nurse plan for when this patient has a computed tomography (CT) scan ordered? a. Shut the feeding off 30 to 60 minutes before the scan. b. Ask the health care provider to reschedule the CT scan. c. Connect the feeding tube to continuous suction during the scan. d. Send the patient to CT scan with oral suction in case of aspiration.

A The tube feeding should be shut off 30 to 60 minutes before any procedure requiring the patient to lie flat. Because the CT scan is ordered for diagnosis of patient problems, rescheduling is not usually an option. Prevention, rather than treatment, of aspiration is needed. Small-bore feeding tubes are soft and collapse easily with aspiration or suction, making nasogastric suction of gastric contents unreliable.

A female patient is awaiting surgery for acute peritonitis. Which action will the nurse include in the plan of care? a. Position patient with the knees flexed. b. Avoid use of opioids or sedative drugs. c. Offer frequent small sips of clear liquids. d. Assist patient to breathe deeply and cough.

A There is less peritoneal irritation with the knees flexed, which will help decrease pain. Opioids and sedatives are typically given to control pain and anxiety. Preoperative patients with peritonitis are given IV fluids for hydration. Deep breathing and coughing will increase the patient's discomfort.

A nurse is teaching a client who has a new diagnosis of pernicious anemia due to chronic gastritis.Which of the following should be included in the teaching? A. Cells producing gastric acid have been damaged. B. A monthly injection of medication is required. C. Vitamin K supplements will be administered. D. Increased production of intrinsic factor is occurring.

B

A client is being weaned off from parenteral nutrition (PN) and is given a go-signal to take a regular diet. The ongoing solution rate has been 120ml/hr. A nurse expects that which of the following prescriptions regarding the PN solution will accompany the diet order? A. Decrease the PN rate to 60ml/hr. B. Start 0.9% normal saline at 30 ml/hr. C. Maintain the present infusion rate. D. Discontinue the PN.

A When a client begins eating a regular diet after a period of receiving PN, the PN is decreased slowly. PN that is terminated abruptly will cause hypoglycemia. Gradually decreasing the infusion rate allows the client to remain sufficiently nourished during the transition to a normal diet and prevents an episode of hypoglycemia.

The nurse preparing for the annual physical exam of a 50-year-old man will plan to teach the patient about a. endoscopy. b. colonoscopy. c. computerized tomography screening. d. carcinoembryonic antigen (CEA) testing.

B

A client is taking cemetidine (Tagament) to treat a hiatal herna. The nurse should evaluate the client to determine that the drug has been effective in preventing which of the following? 1. esophageal reflux 2. dysphagia 3. esophagitis 4. ulcer formation

A histamine receptor blocker that decreases the quantity of gastric secretions. It may be used in hiatal hernia therapy to prevent or treat the esophagitis and heartburn associated with reflux.

The nurse is caring for an older adult male client who reports stomach pain and heartburn. Which syndrome is most significant in determining whether the client's ulceration is gastric or duodenal in origin? A. Pain occurs 1 1/2 to 3 hours after a meal, usually at night. B. Pain is worsened by the ingestion of food. C. The client has a malnourished appearance. D. The client is a man older than 50 years.

A. Pain occurs 1 1/2 to 3 hours after a meal, usually at night. A key symptom characteristic of duodenal ulcers is that pain usually awakens the client between 1 AM and 2 AM, occurring 1 1/2 to 3 hours after a meal.

Nursing management of the patient with acute pancreatitis includes: (SATA) A. Check for signs of hypocalcemia B. Provide a diet low in carbohydrates C. Giving insulin based on sliding scale D. Observing stools for signs of steatorrhea E. Monitoring for infection, particularly respiratory tract infection

AE During the acute phase, it is important to monitor vital signs. Hemodynamic stability may be compromised by hypotension, fever, and tachypnea. Injection fluids are ordered, and the response to therapy is monitored. Fluid and electrolyte balances are closely monitored. Frequent vomiting, along with gastric suction, may result in decreased levels of chloride, sodium, and potassium. Because hypocalcemia can occur in acute pancreatitis, the nurse should observe for symptoms of tetany, such as jerking, irritability, and muscular twitching. Numbness or tingling around the lips and in the fingers is an early indicator of hypocalcemia. The patient should be assessed for Chvostek's sign or Trousseau's sign. A patient with acute pancreatitis should be observed for fever and other manifestations of infection. Respiratory infections are common because the retroperitoneal fluid raises the diaphragm, which causes the patient to take shallow, guarded abdominal breaths.

The client with hiatal hernia chronically experiences heartburn following meals. The nurse plans to teach the client to avoid which action because it is contraindicated with a hiatal hernia? 1. Lying recumbent following meals. 2. Taking in small, frequent bland meals. 3. Raising the head of the bed on 6-inch block. 4. Taking H2-receptor antagonist medication

Answer 1 Hiatal hernia is caused by a protrusion of a portion of the stomach above the diaphragm where the esophagus is normally positioned. he client usually experiences pain from reflux caused by ingestion of irritating foods, lying flat following meals or at night, and eating large or fatty meals. Option 2-4, and actually elevating the thorax after a meal, provide relief

A patient has a vagotomy with antrectomy to treat a duodenal ulcer. Postoperatively, the patient develops dumping syndrome. Which of the following statements, if made by the patient, should indicate to the nurse that further dietary teaching is needed? 1. I should eat bread with each meal 2. I should eat smaller meals more frequently. 3. I should lie down after eating. 4. I should avoid drinking fluids with my meals

Answer 1, Patient should decrease intake of carbohydrates

What response should a nurse offer to a client who asks why he's having a vagotomy to treat his ulcer? 1. To repair a hole in the stomach 2. to reduce the ability of the stomach to produce acid 3. to prevent the stomach from sliding into the chest 4. to remove a potentially malignant lesion in the stomach

Answer 2: A vagotomy is perfomred to elimniate the acid-secreting stimulus to gastric cells. a perforation would be repaired with a gastric resection. Repair of hiatal hernia (fundoplication) prevents the stomach from sliding through the diaphragm. Removal of a potentially malignant tumor wouldn't reduce the entire acid-producing mechanism

Gastroesophageal reflux disease (GERD) weakens the lower esophageal sphincter, predisposing older persons to risk for impaired swallowing. In managing the symptoms associated with GERD, the nurse should assign the highest priority to which of the following interventions? 1. Decrease daily intake of vegetables and water, and ambulate frequently 2. Drink coffee diluted with milk at each meal, and remain in an upright position for 30 minutes. 3. Eat small, frequent meals, and remain in an upright position for at least 30 minutes after eating 4. Avoid over-the-counter drugs that have antacids in them

Answer 3, Eating small and frequent meals requires less release of hydrochloric acid. Remaining in an upright position for 30 minutes after meals prevents reflux into the esophagus which is often exacerbated when lying down, expecially after a large meal which makes the patient tired

which is the most common upper GI problem? 1. peptic ulcer disease 2. Crohns 3. Gerd 4. ulcerative colitis

Answer 3, Gerd is the only upper GI problem

The nurse is monitoring a client with a diagnosis of peptic ulcer. Which assessment finding would most likely indicate perforation of the ulcer? 1. Bradycardia 2. Numbness in the legs 3. Nausea and vomiting 4. A rigid, board-like abdomen

Answer 4, Perforation of an ulcer is a surgical emergency and is characterized by sudden, sharp, intolerable severe pain beginning in the midepigastric area and spreading over the abdomen, which become rigid and board-like. Nausea and vomiting may occur. Tachycardia may occur as hypovolemic shock develops. Numbness in the legs is not an associated finding

Which of the following nursing interventions should the nurse perform for a female client receiving enteral feedings through a gastrostomy tube? a. Change the tube feeding solutions and tubing at least every 24 hours b. Maintain the head of the bed at a 15-degree elevation continuously. c. Check the gastrostomy tube for position every 2 days. d. Maintain the client on bed rest during the feedings

Answer A Tube feeding solutions and tubing should be changed every 24 hours, or more frequently if the feeding requires it. Doing so prevents contamination and bacterial growth. The head of the bed should be elevated 30 to 45 degrees continuously to prevent aspiration. Checking for gastrostomy tube placement is performed before initiating the feedings and every 4 hours during continuous feedings. Clients may ambulate during feedings

"A patient with a history of peptic ulcer disease has presented to the ED with complaints of severe abdominal pain and a rigid, boardlike abdomen, prompting the health care team to suspect a perforated ulcer. Which of the following actions should the nurse anticiptate? "A: Providing IV fluids and inserting a nasogastric tube B:Administering oral bicarbonate and testing patient's gastric pH level C:Performing a fecal occult blood test and administering IV calcium gluconate D: Starting parenteral nutrition and placing the patient in high-fowler's position

Answer A, A perforated peptic ulcer requires IV replacement of fluid losses and continued gastric aspiration by NG tube. Nothing is given by mouth and gastric pH testing is not a priority. Calcium gluconate is not a medication directly relevent to the patient's suspected diagnosis and parenteral nutrition is not a priority in the short term

"A patient with a history of peptic ulcer disease has presented to the emergency department with complaints of severe abdominal pain and a rigid, boardlike abdomen, prompting the health care team to suspect a perforated ulcer. Which of the following actions should the nurse anticipate? A. Providing IV fluids and inserting a nasogastric tube B. Administering oral bicarbonate and testing the patient's gastric pH level C. Performing a fecal occult blood test and administering IV calcium gluconate D. Starting parenteral nutrition and placing the patient in a high-Fowler's position

Answer A, A perforated peptic ulcer requires IV replacement of fluid losses and continued gastric aspiration by NG tube. Nothing is given by mouth and gastric pH testing is not a priority. Calcium gluconate is not a medication directly relevant to the patient's suspected diagnosis and parenteral nutrition is not a priority in the short term

A patient with a history of peptic ulcer disease has presented to the emergency department with complaints of severe abdominal pain and a rigid, boardlike abdomen, prompting the health care team to suspect a perforated ulcer. Which of the following actions should the nurse anticipate? A. Providing IV fluids and inserting a nasogastric tube B. Administering oral bicarbonate and testing the patient's gastric pH level. C. Performing a fecal occult blood test and administering IV calcium gluconate. D. Starting parenteral nutrition and placing the patient in high-Fowler's position

Answer A, A perforated peptic ulcer requires IV replacement of fluid losses and continued gastric aspiration by NG tube. Nothing is given by mouth and gastric pH testing is not a priority. Calcium gluconate is not a medication directly relevant to the patient's suspected diagnosis and parenteral nutrition is not a priority in the short term

"The nurse has instructed the client who is experiencing diarrhea associated with irritable bowel syndrome on dietary changes to prevent diarrhea. The nurse knows the client understands the dietary changes if the client selects which of the following menu choices? a) Yogurt, crackers and sweet tea b) Salad with chicken, whole wheat crackers c) Bacon, tomato, lettuce with mayonnaise and a soft drink d) Tuna on white bread and coconut cake

Answer B Rationale: Bacon tomato lettuce with mayonnaise and soft drink is high in fat and the soft drink is hyperosmolar both contributing to diarrhea. Salad, whole wheat crackers may decrease diarrhea due to increased fiber. Dairy increases diarrhea. Food high in carbohydrates increase diarrhea. Coconut may increase diarrhea

Which care activity for a patient with a paralytic ileus is appropriate for the registered nurse (RN) to delegate to unlicensed assistive personnel (UAP)? a. Auscultation for bowel sounds b. Nasogastric (NG) tube irrigation c. Applying petroleum jelly to the lips d. Assessment of the nares for irritation

C

The nurse is caring for a female client with active upper GI bleeding. What is the appropriate diet for this client during the first 24 hours after admission? a. regular diet b. skim milk c. nothing by mouth d. clear liquids

Answer C Shock and bleeding must be controlled before oral intake, so the client should receive nothing by mouth. A regular diet is incorrect. When the bleeding is controlled, the diet is gradually increased, starting with ice chips and then clear liquids. Skim milk shouldn't be given because it increases gastric acid production, which could prolong bleeding. A liquid diet is the first diet offered after bleeding and shock are controlled

"The nurse determines that a patient has experienced the beneficial effects of medication therapy with famotidine (Pepcid) when which of the following symptoms is relieved? " A) Nausea B) Belching C) Epigastric pain D) Difficulty swallowing

Answer C, "Famotidine is an H2-receptor antagonist that inhibits parietal cell output of HCl acid and minimizes damage to gastric mucosa related to hyperacidity, thus relieving epigastric pain

The nurse determines that a patient has experienced the beneficial effects of medication therapy with famotidine (Pepcid) when which of the following symptoms is relieved? A) Nausea B) Belching C) Epigastric pain D) Difficulty swallowing

Answer C, "Famotidine is an H2-receptor antagonist that inhibits parietal cell output of HCl acid and minimizes damage to gastric mucosa related to hyperacidity, thus relieving epigastric pain

The nurse is reviewing the medication record of a female client with acute gastritis. Which medication, if noted on the client's record, would the nurse question? a. Digoxin (Lanoxin) b. Furosemide (Lasix) c. Indomethacin (Indocin) d. Propranolol hydrochloride (Inderal)

Answer is C, Indomethacin (Indocin) is a nonsteroidal anti-inflammatory drug and can cause ulceration of the esophagus, stomach, or small intestine. Indomethacin is contraindicated in a client with gastrointestinal disorders. Furosemide (Lasix) is a loop diuretic. Digoxin is a cardiac medication. Propranolol (Inderal) is a β-adrenergic blocker. Furosemide, digoxin, and propranolol are not contraindicated in clients with gastric disorders

Fistulas are most common with which of the following bowel disorders? 1. Crohn's disease 2. Diverticulitis 3. Diverticulosis 4. Ulcerative colitis

Answer: 1. Crohn's disease The lesions of Crohn's disease are transmural; that is, they involve all thickness of the bowel. These lesions may perforate the bowel wall, forming fistulas with adjacent structures. Fistulas don't develop in diverticulitis or diverticulosis. The ulcers that occur in the submucosal and mucosal layers of the intestine in ulcerative colitis usually don't progress to fistula formation as in Crohn's disease.

When planning care for a client with ulcerative colitis who is experiencing symptoms, which client care activities can the nurse appropriately delegate to a unlicensed assistant? Select all that apply. 1. Assessing the client's bowel sounds 2. Providing skin care following bowel movements 3. Evaluating the client's response to antidiarrheal medications 4. Maintaining intake and output records 5. Obtaining the client's weight.

Answer: 2, 4, and 5. The nurse can delegate the following basic care activities to the unlicensed assistant: providing skin care following bowel movements, maintaining intake and output records, and obtaining the client's weight. Assessing the client's bowel sounds and evaluating the client's response to medication are registered nurse activities that cannot be delegated.

Which of the following factors is believed to cause ulcerative colitis? 1. Acidic diet 2. Altered immunity 3. Chronic constipation 4. Emotional stress

Answer: 2. Altered immunity Several theories exist regarding the cause of ulcerative colitis. One suggests altered immunity as the cause based on the extraintestinal characteristics of the disease, such as peripheral arthritis and cholangitis. Diet and constipation have no effect on the development of ulcerative colitis. Emotional stress can exacerbate the attacks but isn't believed to be the primary cause.

Which goal of the client's care should take priority during the first days of hospitalization for an exacerbation of ulcerative colitis? 1. Promoting self-care and independence 2. Managing diarrhea 3. Maintaining adequate nutrition 4. Promoting rest and comfort

Answer: 2. Managing diarrhea Diarrhea is the primary symptom in an exacerbation of ulcerative colitis, and decreasing the frequency of stools is the first goal of treatment. The other goals are ongoing and will be best achieved by halting the exacerbation. The client may receive antidiarrheal medications, antispasmodic agents, bulk hydrophilic agents, or anti-inflammatory drugs.

During the first few days of recovery from ostomy surgery for ulcerative colitis, which of the following aspects should be the first priority of client care? 1. Body image 2. Ostomy care 3. Sexual concerns 4. Skin care

Answer: 2. Ostomy care Although all of these are concerns the nurse should address, being able to safely manage the ostomy is crucial for the client before discharge.

The nurse is planning the care of a client diagnosed with lower esophageal sphincter dysfunction. Which dietary modifications should be included in the plan of care? 1. Allow any of the client's favorite foods as long as the amount is limited. 2. Have the client perform eructation exercises several times a day. 3. Eat four (4) to six (6) small meals a day and limit fluids during mealtimes. 4. Encourage the client to consume a glass of red wine with one (1) meal a day

Answer: 3 1. The client is instructed to avoid spicy and acidic foods and any food producing symptoms. 2. Eructation means belching, which is a symptom of GERD. 3. Clients should eat small, frequent meals and limit fluids with the meals to prevent reflux into the esophagus from a distended stomach.CORRECT 4. Clients are encouraged to forgo all alcoholic beverages because alcohol relaxes the lower esophageal sphincter and increases the risk of reflux

Which finding by the nurse during abdominal auscultation indicates a need for a focused abdominal assessment? a. Loud gurgles b. High-pitched gurgles c. Absent bowel sounds d. Frequent clicking sounds

C

A client with ulcerative colitis has an order to begin salicylate medication to reduce inflammation. The nurse instructs the client to take the medication: 1. 30 minutes before meals 2. On an empty stomach 3. After meals 4. On arising

Answer: 3. After meals Salicylate compounds act by inhibiting prostaglandin synthesis and reducing inflammation. The nurse teaches the client to take the medication with a full glass of water and to increase fluid intake throughout the day. This medication needs to be taken after meals to reduce GI irritation.

A client's ulcerative colitis symptoms have been present for longer than 1 week. The nurse recognizes that the client should be assessed carefully for signs of which of the following complications? 1. Heart failure 2. DVT 3. Hypokalemia 4. Hypocalcemia

Answer: 3. Hypokalemia Excessive diarrhea causes significant depletion of the body's stores of sodium and potassium as well as fluid. The client should be closely monitored for hypokalemia and hyponatremia. Ulcerative colitis does not place the client at risk for heart failure, DVT, or hypocalcemia.

A client who has ulcerative colitis has persistent diarrhea. He is thin and has lost 12 pounds since the exacerbation of his ulcerative colitis. The nurse should anticipate that the physician will order which of the following treatment approaches to help the client meet his nutritional needs? 1. Initiate continuous enteral feedings 2. Encourage a high protein, high-calorie diet 3. Implement total parenteral nutrition 4. Provide six small meals a day.

Answer: 3. Implement total parenteral nutrition Food will be withheld from the client with severe symptoms of ulcerative colitis to rest the bowel. To maintain the client's nutritional status, the client will be started on TPN. Enteral feedings or dividing the diet into 6 small meals does not allow the bowel to rest. A high-calorie, high-protein diet will worsen the client's symptoms.

Which assessment data support the client's diagnosis of gastric ulcer? 1. Presence of blood in the client's stool for the past month. 2.Complaints of a burning sensation that moves like a wave. 3.Sharp pain in the upper abdomen after eating a heavy meal. 4.Comparison of complaints of pain with ingestion of food and sleep

Answer: 4 1. The presence of blood does not specifically indicate diagnosis of an ulcer. The client could have hemorrhoids or cancer that would result in the presence of blood. 2. A wavelike burning sensation is a symptom of gastroesophageal reflus. 3. Sharp pain in the upper abdomen after eating a heavy meal is a symptom of gallbladder disease. 4. (CORRECT) In a client diagosed with a gastric ulcer, pain usually occurs 30-60 minutes after eating, but not at night. In contrast, a client with duodenal ulcer has pain durin ghte night that is often relieved by eating food. Pain occurs 1-3 hours after meals

A patient arrives to the clinic for evaluation of epigastric pain. The patient describes the pain to be relieved by food intake. In addition, the patient reports awaking in the middle of the night with a gnawing pain in the stomach. Based on the patient's description this appears to be what type of peptic ulcer? A. Duodenal B. Gastric C. Esophageal D. Refractory

Answer: A

A physician prescribes a Proton-Pump Inhibitor to a patient with a gastric ulcer. Which medication is considered a PPI? A. Pantoprazole B. Famotidine C. Magnesium Hydroxide D. Metronidazole

Answer: A

Which bowel disease starts in the rectum and migrates in a continuous fashion throughout the colon? a. Ulcerative Colitis b. Crohn's Disease

Answer: A

______________ affects the inner layer of the intestinal lining. a. Ulcerative Colitis b. Crohn's Disease

Answer: A

Which statement is INCORRECT about Histamine-receptor blockers? A. "H2 blockers block histamine which causes the chief cells to decrease the secretion of hydrochloric acid." B. "Ranitidine and Famotidine are two types of histamine-receptor blocker medications." C. "Antacids and H2 blockers should not be given together." D. All the statements are CORRECT.

Answer: A H2 blockers block histamine which causes the PARTIETAL (not chief) cells to decrease the secretion of hydrochloric acid.

A patient is recovering from discomfort from a peptic ulcer. The doctor has ordered to advance the patient's diet to solid foods. The patient's lunch tray arrives. Which food should the patient avoid eating? A. Orange B. Milk C. White rice D. Banana

Answer: A When an ulcer is actively causing signs and symptoms, the patient should avoid acidic foods like tomatoes or citric fruits/juices, chocolate, alcohol, fried foods and caffeine. These foods can irritate the ulcer site. Instead the patient should consume alkalotic or bland foods like milk, white rice or bananas.

Select ALL the options below that are similarities between ulcerative colitis and Crohn's Disease: A. Each cause inflammation B. Both affect the serosa layer C. Each are found from the mouth to anus D. They both increase colon cancer risk E. The cause of both is unknown F. The cure for both diseases includes total colectomy

Answer: A, D, E

Select all the medications a physician may order to treat a H. Pylori infection that is causing a peptic ulcer? A. Proton-Pump Inhibitors B. Antacids C. Anticholinergics D. 5-Aminosalicylates E. Antibiotics F. H2 Blockers G. Bismuth Subsalicylates

Answer: A, E, F, and G All these medications can be used to treat an h. pylori infection that is causing a peptic ulcer.

Which type of bowel disease is most likely to cause severe malnourishment? a. Ulcerative Colitis b. Crohn's Disease

Answer: B

You're precepting a nursing student who is helping you provide T-Tube drain care. You explain to the nursing student that the t-shaped part of the drain is located in what part of the biliary tract? A. Cystic duct B. Common hepatic duct C. Common bile duct D. Pancreatic duct

C

You're educating a group of patients at an outpatient clinic about peptic ulcer formation. Which statement is correct about how peptic ulcers form? A. "An increase in gastric acid is the sole cause of peptic ulcer formation." B. "Peptic ulcers can form when acid penetrates unprotected stomach mucosa. This causes histamine to be released which signals to the parietal cells to release more hydrochloric acid which erodes the stomach lining further." C. "Peptic ulcers form when acid penetrates unprotected stomach mucosa. This causes pepsin to be released which signals to the parietal cells to release more pepsinogen which erodes the stomach lining further." D. "The release of prostaglandins cause the stomach lining to breakdown which allows ulcers to form."

Answer: B

The nurse determines that a patient has experienced the beneficial effects of medication therapy with famotidine (Pepcid) when which of the following symptoms is relieved? 1. Ice tea 2. Dry toast 3. warm broth 4. plain hamburger

Answer: B Dry toast (Dry toast or crackers may alleviate the feeling of nausea and prevent further vomiting. Extremely hot or cold liquids and fatty foods are generally not well tolerated

You are providing discharge teaching to a patient taking Sucralfate (Carafate). Which statement by the patient demonstrates they understand how to take this medication? A. "I will take this medication at the same time I take Ranitidine." B. "I will always take this medication on an empty stomach." C. "It is best to take this medication with antacids." D. "I will take this medication once a week."

Answer: B It should always be taken on an empty stomach without food so it can coat the site of ulceration. This medication should NOT be taken with H2 blockers (Ranitidine) or antacids because these drugs affect the absorption of Carafate.

In a patient with Dumping Syndrome, select ALL the correct statements on how to educate this patient about decreasing their symptoms: A. "It is best to eat 3 large meals a day rather than small frequent meals." B. "After eating a meal lie down for 30 minutes." C. "Eat a diet high in protein, fiber, and low in carbs." D. "Be sure to drink at least 16 oz. of milk with meals."

Answer: B, C

Select-ALL-the complications associated with Crohn's Disease: A. Loss of form to the haustra B. Fistulas C. Strictures D. Hemorrhoids E. Anal Fissure

Answer: B, C, E

A patient with severe ulcerative colitis had a barium enema. Which finding is associated with this disease? A. Cobble-stone appearance of the colon B. Cullen's Sign C. Lead-Pipe Sign D. McBurney's Sign

Answer: C In severe cases of ulcerative colitis, the haustra (which are the pouches of the large intestine that churn food) will lose their form. On a barium enema, the large intestine will appear as a smooth organ (hence a lead pipe) rather than a unique organ with the pouches.

"The nurse is caring for a 68 year old patient admitted with abdominal pain, nausea, and vomiting. The patient has an abdominal mass and a bowel obstruction is suspected. The nurse auscultating the abdomen listens for which of the following types of bowel sounds that is consistent with the patient's clinical picture? A. low pitched and rumbling above the area of obstruction B. High pitched and hypoactive below the area of obstruction C. low pitched and hyperactive below the area of obstruction D. high pitched and hyperactive above the area of obstruction

Answer: D Early in intestinal obstruction, the patient's bowel sounds are hyperactive adn high pitched. This occurs because peristaltic action increases to "push past" the area of obstruction. As the obstruction becomes complete, bowel sounds decrease and finally become absent

Which assessment data support to the the nurse the client's diagnosis of gastric ulcer? A. Presence of blood in the client's stool for the past month? B. Reports of a burning sensation moving like a wave. C. Sharp pain in the upper abdomen after eating a heavy meal. D. Complaints of epigastric pain 30-60 minutes after ingesting food

Answer: D In a client diagnosed with a gastric ulcer, pain usually occurs 30-60 minutes after eating, but not at night. In contrast, a client with a duodenal ulcer has pain during the night often relieved by eating foods. Other answers: the presence of blood does not specifically indicate diagnose of an ulcer. The client could have hemorrhoids or cancer. A waveline burning sensation is a symptom of GERD. Sharp pain in the upper abdomen after eating a heavy meal is a symptom of gallbladder disease

The nurse explains to the patient with gastroesophageal reflux disease that this disorder: A. results in acid erosion and ulceration of the esophagus caused by frequent vomiting, B. will require surgical wrapping or repair of the pyloric sphincter to control the symptoms, C. is the protrusion of a portion of the stomach into to esophagus through an opening in the diaphragm, D. often involves relaxation of the lower esophageal sphincter, allowing stomach contents to back up into the espophagus

Answer: D. The acidic contents of the stomach touching the inside of the esophagus are responsible for the physical sensation known as "heart-burn" that is a cardinal symptom of GERD

A 25-year-old male patient calls the clinic complaining of diarrhea for 24 hours. Which action should the nurse take first? a. Inform the patient that laboratory testing of blood and stools will be necessary. b. Ask the patient to describe the character of the stools and any associated symptoms. c. Suggest that the patient drink clear liquid fluids with electrolytes, such as Gatorade or Pedialyte. d. Advise the patient to use over-the-counter loperamide (Imodium) to slow gastrointestinal (GI) motility.

B

Which information will the nurse include in teaching a patient who had a proctocolectomy and ileostomy for ulcerative colitis? a. Restrict fluid intake to prevent constant liquid drainage from the stoma. b. Use care when eating high-fiber foods to avoid obstruction of the ileum. c. Irrigate the ileostomy daily to avoid having to wear a drainage appliance. d. Change the pouch every day to prevent leakage of contents onto the skin.

B

Which information will the nurse teach a 23-year-old patient with lactose intolerance? a. Ice cream is relatively low in lactose. b. Live-culture yogurt is usually tolerated. c. Heating milk will break down the lactose. d. Nonfat milk is a better choice than whole milk.

B

Which patient statement indicates that the nurse's teaching about sulfasalazine (Azulfidine) for ulcerative colitis has been effective? a. "The medication will be tapered if I need surgery." b. "I will need to use a sunscreen when I am outdoors." c. "I will need to avoid contact with people who are sick." d. "The medication will prevent infections that cause the diarrhea."

B

Your patient is post-op day 3 from a cholecystectomy due to cholecystitis and has a T-Tube. Which finding during your assessment of the T-Tube requires immediate nursing intervention? A. The drainage from the T-Tube is yellowish/green in color. B. There is approximately 750 cc of drainage within the past 24 hours. C. The drainage bag and tubing is at the patient's waist. D. The patient is in the Semi-Fowler's position.

B

When preparing a male client, age 51, for surgery to treat appendicitis, the nurse formulates a nursing diagnosis of Risk for infection related to inflammation, perforation, and surgery. What is the rationale for choosing this nursing diagnosis? a. Obstruction of the appendix may increase venous drainage and cause the appendix to rupture. b. Obstruction of the appendix reduces arterial flow, leading to ischemia, inflammation, and rupture of the appendix. c. The appendix may develop gangrene and rupture, especially in a middle-aged client. d. Infection of the appendix diminishes necrotic arterial blood flow and increases venous drainage.

B A client with appendicitis is at risk for infection related to inflammation, perforation, and surgery because obstruction of the appendix causes mucus fluid to build up, increasing pressure in the appendix and compressing venous outflow drainage. The pressure continues to rise with venous obstruction; arterial blood flow then decreases, leading to ischemia from lack of perfusion. Inflammation and bacterial growth follow, and swelling continues to raise pressure within the appendix, resulting in gangrene and rupture. Geriatric, not middle-aged, clients are especially susceptible to appendix rupture.

The nurse will plan to monitor a patient with an obstructed common bile duct for a. melena. b. steatorrhea. c. decreased serum cholesterol levels. d. increased serum indirect bilirubin levels.

B A common bile duct obstruction will reduce the absorption of fat in the small intestine, leading to fatty stools. Gastrointestinal (GI) bleeding is not caused by common bile duct obstruction. Serum cholesterol levels are increased with biliary obstruction. Direct bilirubin level is increased with biliary obstruction.

A 61-year-old patient with suspected bowel obstruction had a nasogastric tube inserted at 4:00 AM. The nurse shares in the morning report that the day shift staff should check the tube for patency at what times? A) 7:00 AM, 10:00 AM, and 1:00 PM B) 8:00 AM, 12:00 PM, and 4:00 PM C) 9:00 AM and 3:00 PM D) 9:00 AM, 12:00 PM, and 3:00 PM

B A nasogastric tube should be checked for patency routinely at 4-hour intervals. Thus if the tube were inserted at 4:00 AM, it would be due to be checked at 8:00 AM, 12:00 PM, and 4:00 PM.

The nurse is caring for a 55-year-old man patient with acute pancreatitis resulting from gallstones. Which clinical manifestation would the nurse expect the patient to exhibit? A. Hematochezia B. Left upper abdominal pain C. Ascites and peripheral edema D. Temperature over 102o F (38.9o C)

B Abdominal pain (usually in the left upper quadrant) is the predominant manifestation of acute pancreatitis. Other manifestations of acute pancreatitis include nausea and vomiting, low-grade fever, leukocytosis, hypotension, tachycardia, and jaundice. Abdominal tenderness with muscle guarding is common. Bowel sounds may be decreased or absent. Ileus may occur and causes marked abdominal distention. Areas of cyanosis or greenish to yellow-brown discoloration of the abdominal wall may occur. Other areas of ecchymoses are the flanks (Grey Turner's spots or sign, a bluish flank discoloration) and the periumbilical area (Cullen's sign, a bluish periumbilical discoloration).

When evaluating a male client for complications of acute pancreatitis, the nurse would observe for: a. increased intracranial pressure. b. decreased urine output. c. bradycardia. d. hypertension."

B Acute pancreatitis can cause decreased urine output, which results from the renal failure that sometimes accompanies this condition. Intracranial pressure neither increases nor decreases in a client with pancreatitis. Tachycardia, not bradycardia, usually is associated with pulmonary or hypovolemic complications of pancreatitis. Hypotension can be caused by a hypovolemic complication, but hypertension usually isn't related to acute pancreatitis."

A malnourished patient is receiving a parenteral nutrition (PN) infusion containing amino acids and dextrose from a bag that was hung 24 hours ago. The nurse observes that about 50 mL remain in the PN container. Which action is best for the nurse to take? a. Ask the health care provider to clarify the written PN order. b. Add a new container of PN using the current tubing and filter. c. Hang a new container of PN and change the IV tubing and filter. d. Infuse the remaining 50 mL and then hang a new container of PN.

B All PN solutions are changed at 24 hours. PN solutions containing dextrose and amino acids require a change in tubing and filter every 72 hours rather than daily. Infusion of the additional 50 mL will increase patient risk for infection. Changing the IV tubing and filter more frequently than required will unnecessarily increase costs. The nurse (not the health care provider) is responsible for knowing the indicated times for tubing and filter changes.

The nurse is admitting a 67-year-old patient with new-onset steatorrhea. Which question is most important for the nurse to ask? a. "How much milk do you usually drink?" b. "Have you noticed a recent weight loss?" c. "What time of day do your bowels move?" d. "Do you eat meat or other animal products?"

B Although all of the questions provide useful information, it is most important to determine if the patient has an imbalance in nutrition because of the steatorrhea.

A patient being admitted with an acute exacerbation of ulcerative colitis reports crampy abdominal pain and passing 15 or more bloody stools a day. The nurse will plan to a. administer IV metoclopramide (Reglan). b. discontinue the patient's oral food intake. c. administer cobalamin (vitamin B12) injections. d. teach the patient about total colectomy surgery.

B An initial therapy for an acute exacerbation of inflammatory bowel disease (IBD) is to rest the bowel by making the patient NPO

A 49-year-old man has been admitted with hypotension and dehydration after 3 days of nausea and vomiting. Which order from the health care provider will the nurse implement first? a. Insert a nasogastric (NG) tube. b. Infuse normal saline at 250 mL/hr. c. Administer IV ondansetron (Zofran). d. Provide oral care with moistened swabs.

B Because the patient has severe dehydration, rehydration with IV fluids is the priority. The other orders should be accomplished as quickly as possible after the IV fluids are initiated.

For Rico who has chronic pancreatitis, which nursing intervention would be most helpful? A. Allowing liberalized fluid intake B. Counseling to stop alcohol consumption C. Encouraging daily exercise D. Modifying dietary protein

B Chronic pancreatitis typically results from repeated episodes of acute pancreatitis. More than half of chronic pancreatitis cases are associated with alcoholism. Counseling to stop alcohol consumption would be the most helpful for the client. Dietary protein modification is not necessary for chronic pancreatitis. Daily exercise and liberalizing fluid intake would be helpful but not the most beneficial intervention.

Which item should the nurse offer to the patient who is to restart oral intake after being NPO due to nausea and vomiting? a. Glass of orange juice b. Dish of lemon gelatin c. Cup of coffee with cream d. Bowl of hot chicken broth

B Clear cool liquids are usually the first foods started after a patient has been nauseated. Acidic foods such as orange juice, very hot foods, and coffee are poorly tolerated when patients have been nauseated.

After a total proctocolectomy and permanent ileostomy, the patient tells the nurse, "I cannot manage all these changes. I don't want to look at the stoma." What is the best action by the nurse? a. Reassure the patient that ileostomy care will become easier. b. Ask the patient about the concerns with stoma management. c. Develop a detailed written list of ostomy care tasks for the patient. d. Postpone any teaching until the patient adjusts to the ileostomy.

B Encouraging the patient to share concerns assists in helping the patient adjust to the body changes. Acknowledgment of the patient's feelings and concerns is important rather than offering false reassurance. Because the patient indicates that the feelings about the ostomy are the reason for the difficulty with the many changes, development of a detailed ostomy care plan will not improve the patient's ability to manage the ostomy. Although detailed ostomy teaching may be postponed, the nurse should offer teaching about some aspects of living with an ostomy.

A 24-year-old woman with Crohn's disease develops a fever and symptoms of a urinary tract infection (UTI) with tan, fecal-smelling urine. What information will the nurse add to a general teaching plan about UTIs in order to individualize the teaching for this patient? a. Bacteria in the perianal area can enter the urethra. b. Fistulas can form between the bowel and bladder. c. Drink adequate fluids to maintain normal hydration. d. Empty the bladder before and after sexual intercourse.

B Fistulas between the bowel and bladder occur in Crohn's disease and can lead to UTI

The patient with sudden pain in the left upper quadrant radiating to the back and vomiting was diagnosed with acute pancreatitis. What intervention(s) should the nurse expect to include in the patient's plan of care? A. Immediately start enteral feeding to prevent malnutrition. B. Insert an NG and maintain NPO status to allow pancreas to rest. C. Initiate early prophylactic antibiotic therapy to prevent infection. D. Administer acetaminophen (Tylenol) every 4 hours for pain relief.

B Initial treatment with acute pancreatitis will include an NG tube if there is vomiting and being NPO to decrease pancreatic enzyme stimulation and allow the pancreas to rest and heal. Fluid will be administered to treat or prevent shock. The pain will be treated with IV morphine because of the NPO status. Enteral feedings will only be used for the patient with severe acute pancreatitis in whom oral intake is not resumed. Antibiotic therapy is only needed with acute necrotizing pancreatitis and signs of infection.

Which client is most at risk for the development of gallstones? A. 22-year-old woman who is 1 month postpartum B. 65-year-old woman after a liquid protein diet C. 70-year-old man with peptic ulcer disease D. 33-year-old man with type 2 diabetes

B Liquid protein diets increase susceptibility to gallstones by releasing cholesterol from tissues, which is then excreted as crystals in the bile.

A client is receiving parenteral nutrition (PN) in the home setting has a weight gain of 5 lb in 1 week. The nurse next assesses the client to identify the presence of which of the following? A. Hypotension. B. Crackles upon auscultation of the lungs. C. Thirst. D. Polyuria.

B Normally, the weight gain of a client receiving PN is about 1-2 pound a week. A weight gain of 5 pounds over a week indicates a client is experiencing fluid retention that can result to hypervolemia. Signs of hypervolemia includes weight gain more than desired, headache, jugular vein distention, bounding pulse, and crackles on lung auscultation. Option A: Hypertension, not hypotension is expected. Options C and D are associated with hyperglycemia.

The patient with right upper quadrant abdominal pain has an abdominal ultrasound that reveals cholelithiasis. What should the nurse expect to do for this patient? A. Prevent all oral intake. B. Control abdominal pain. C. Provide enteral feedings. D. Avoid dietary cholesterol.

B Patients with cholelithiasis can have severe pain, so controlling pain is important until the problem can be treated. NPO status may be needed if the patient will have surgery but will not be used for all patients with cholelithiasis. Enteral feedings should not be needed, and avoiding dietary cholesterol is not used to treat cholelithiasis

Which prescribed intervention for a 61-year-old female patient with chronic short bowel syndrome will the nurse question? a. Ferrous sulfate (Feosol) 325 mg daily b. Senna (Senokot) 1 tablet every day c. Psyllium (Metamucil) 2.1 grams 3 times daily d. Diphenoxylate with atropine (Lomotil) prn loose stools

B Patients with short bowel syndrome have diarrhea because of decreased nutrient and fluid absorption and would not need stimulant laxatives

A 58-year-old man with blunt abdominal trauma from a motor vehicle crash undergoes peritoneal lavage. If the lavage returns brown fecal drainage, which action will the nurse plan to take next? a. Auscultate the bowel sounds. b. Prepare the patient for surgery. c. Check the patient's oral temperature. d. Obtain information about the accident.

B Return of brown drainage and fecal material suggests perforation of the bowel and the need for immediate surgery

A nurse is completing preoperative teaching for a client who will undergo a laparoscopic cholecystectomy. Which of the following should be included in the teaching? A. "The scope will be passed through your rectum." B. "You may have shoulder pain after surgery." C. "The T-tube will remain in place for 1 to 2 weeks." D. "You should limit how often you walk for 1 to 2 weeks."

B Shoulder pain occurs due to free air that is introduced into the abdomen during laparoscopic surgery.

Which assessment information will be most important for the nurse to report to the health care provider about a patient with acute cholecystitis? a. The patient's urine is bright yellow. b. The patient's stools are tan colored. c. The patient has increased pain after eating. d. The patient complains of chronic heartburn.

B Tan or grey stools indicate biliary obstruction, which requires rapid intervention to resolve. The other data are not unusual for a patient with this diagnosis, although the nurse would also report the other assessment information to the health care provider.

A nurse is preparing to change the parenteral nutrition (PN) solution bag and tubing. The client's central venous line is located in the right subclavian vein. The nurse ask the client to take which essential action during the tube change? A. Turn the head to the right. B. Inhale deeply, hold it, and bear down. C. Breathe normally. D. Exhale slowly and evenly.

B The client should be asked to perform the Valsalva maneuver during tubing changes. This helps avoid air embolism during tube changes. The nurse asks the client to take a deep breath, hold it, and bear down. Option A is incorrect because if the intravenous line is on the right, the client turns his or head to the left. This position increases intrathoracic pressure. Options C and D can cause the potential for an air embolism during the tube change.

A 20-year-old man with extensive facial injuries from a motor vehicle crash is receiving tube feedings through a percutaneous endoscopic gastrostomy (PEG). Which action will the nurse include in the plan of care? a. Keep the patient positioned on the left side. b. Check the gastric residual volume every 4 to 6 hours. c. Avoid giving bolus tube feedings through the PEG tube. d. Obtain a daily abdominal x-ray to verify tube placement.

B The gastric residual volume is assessed every 4 to 6 hours to decrease the risk for aspiration. The patient does not need to be positioned on the left side. Bolus feedings can be administered through a PEG tube. An x-ray is obtained immediately after placement of the PEG tube to check position, but daily x-rays are not needed.

After change-of-shift report, which patient will the nurse assess first? a. A 40-year-old woman whose parenteral nutrition infusion bag has 30 minutes of solution left b. A 40-year-old man with continuous enteral feedings who has developed pulmonary crackles c. A 30-year-old man with 4+ generalized pitting edema and severe protein-calorie malnutrition d. A 30-year-old woman whose gastrostomy tube is plugged after crushed medications were administered.

B The patient data suggest aspiration has occurred and rapid assessment and intervention are needed. The other patients should also be assessed as quickly as possible, but the data about them do not suggest any immediately life-threatening complications

The nurse would increase the comfort of the patient with appendicitis by: a. Having the patient lie prone b. Flexing the patient's right knee c. Sitting the patient upright in a chair d. Turning the patient onto his or her left side

B The patient with appendicitis usually prefers to lie still, often with the right leg flexed to decrease pain.

The nurse is administering IV fluid boluses and nasogastric irrigation to a patient with acute gastrointestinal (GI) bleeding. Which assessment finding is most important for the nurse to communicate to the health care provider? a. The bowel sounds are hyperactive in all four quadrants. b. The patient's lungs have crackles audible to the midchest. c. The nasogastric (NG) suction is returning coffee-ground material. d. The patient's blood pressure (BP) has increased to 142/84 mm Hg.

B The patient's lung sounds indicate that pulmonary edema may be developing as a result of the rapid infusion of IV fluid and that the fluid infusion rate should be slowed. The return of coffee-ground material in an NG tube is expected for a patient with upper GI bleeding. The BP is slightly elevated but would not be an indication to contact the health care provider immediately. Hyperactive bowel sounds are common when a patient has GI bleeding.

A 68-year-old patient with a bleeding duodenal ulcer has a nasogastric (NG) tube in place, and the health care provider orders 30 mL of aluminum hydroxide/magnesium hydroxide (Maalox) to be instilled through the tube every hour. To evaluate the effectiveness of this treatment, the nurse a. monitors arterial blood gas values daily. b. periodically aspirates and tests gastric pH. c. checks each stool for the presence of occult blood. d. measures the volume of residual stomach contents.

B The purpose for antacids is to increase gastric pH. Checking gastric pH is the most direct way of evaluating the effectiveness of the medication. Arterial blood gases may change slightly, but this does not directly reflect the effect of antacids on gastric pH. Because the patient has upper gastrointestinal (GI) bleeding, occult blood in the stools will appear even after the acute bleeding has stopped. The amount of residual stomach contents is not a reflection of resolution of bleeding or of gastric pH.

When caring for a 63-year-old woman with a soft, silicone nasogastric tube in place for enteral feedings, the nurse will a. avoid giving medications through the feeding tube. b. flush the tubing after checking for residual volumes. c. administer continuous feedings using an infusion pump. d. replace the tube every 3 days to avoid mucosal damage.

B The soft silicone feeding tubes are small in diameter and can easily become clogged unless they are flushed after the nurse checks the residual volume. Either intermittent or continuous feedings can be given. The tubes are less likely to cause mucosal damage than the stiffer polyvinyl chloride tubes used for nasogastric suction and do not need to be replaced at certain intervals. Medications can be given through these tubes, but flushing after medication administration is important to avoid clogging.

Spironolactone (Aldactone) is prescribed for a client with chronic cirrhosis and ascites. The nurse should monitor the client for which of the following medication-related side effects? A. Jaundice B. Hyperkalemia C. Tachycardia D. Constipation

B This is a potassium-sparing diuretic so clients should be monitored closely for hyperkalemia. Diarrhea, dizziness, and headaches are other more common side effects. Tachycardia, jaundice, and constipation are not expected side effects of spironolactone (Aldactone).

Which information about an 80-year-old man at the senior center is of most concern to the nurse? a. Decreased appetite b. Unintended weight loss c. Difficulty chewing food d. Complaints of indigestion

B Unintentional weight loss is not a normal finding and may indicate a problem such as cancer or depression. Poor appetite, difficulty in chewing, and complaints of indigestion are common in older patients. These will need to be addressed but are not of as much concern as the weight loss.

Which assessment data indicate to the nurse the clients gastric ulcer has perforated? A. Complaints of sudden, sharp, substernal pain B. Rigid, boardlike abdomen with rebound tenderness C. Frequent, clay-colored, liquid stool D. Complaints of vague abdominal pain in the right upper quadrant

B. A rigid, boardlike abdomen with rebound tenderness is the classic sign/symptom of peritonitis, which is a complication of a perforated gastric ulcer

The client with a history of peptic ulcer disease is admitted into the intensive care unit with frank gastric bleeding. Which priority intervention should the nurse implement? A. Maintain a strict record of intake and output B. Insert a nasogastric tube and begin saline lavage C. Assist the client with keeping a detailed calorie count D. Provide a quiet environment to promote rest

B. Inserting a nasogastric tube and lavaging the stomach with saline is the most important intervention because this directly stops the bleeding

A nurse is providing discharge teaching to a client who is postoperative following open cholecystectomywith T-tube placement. Which of the following instructions should the nurse include in the teaching?(Select all that apply. A. Take baths rather than showers. B. Clamp T-tube for 1 to 2 hr before and after meals. C. Keep the drainage system above the level of the abdomen. D. Expect to have the T-tube removed 3 days postoperatively.

BC

A 72-year-old male patient with dehydration caused by an exacerbation of ulcerative colitis is receiving 5% dextrose in normal saline at 125 mL/hour. Which assessment finding by the nurse is most important to report to the health care provider? a. Patient has not voided for the last 4 hours. b. Skin is dry with poor turgor on all extremities. c. Crackles are heard halfway up the posterior chest. d. Patient has had 5 loose stools over the last 6 hours.

C

A client has a primary problem of inadequate nutrition caused by the effects of chemotherapy. The client is receiving continuous enteral feedings through a nasogastric tube (NG) tube. What does the RN ask the LPN/LVN to do for this client? A. Assess nutritional parameters on the client every 3 days. B. Check the residual volume of the NG tube every 4 hours. C. Monitor the client for signs and symptoms of pneumonia. D. Teach the client about the purpose of enteral feedings.

C

A client receiving total parenteral nutrition (TPN) exhibits symptoms of congestive heart failure (CHF) and pulmonary edema. Which complication of TPN is the client most likely experiencing? A. Calcium imbalance B. Fluid volume deficit C. Fluid volume overload D. Potassium imbalance

C

A nurse is caring for a group of clients on a medical-surgical nursing unit. The nurse recognizes that which of the following clients would be the least likely candidate for parenteral nutrition? A. A 55-year-old with persistent nausea and vomiting from chemotherapy. B. A 44-year old client with ulcerative colitis. C. A 59-year old client who had an appendectomy. D. A 25-year old client with a Hirschprung's Disease.

C

A nurse is changing the central line dressing of a client receiving parenteral nutrition (PN) and notes that the catheter insertion site appears reddened. The nurse next assesses which of the following items? A. Time of last dressing change. B. Tightness of the tuning connections. C. Client's temperature. D. Expiration date on the bag.

C

A nurse is completing teaching to a client who has a new prescription for famotidine (Pepcid). Which ofthe following statements by the client indicates understanding of the teaching? A. "This medicine coats the lining of my stomach." B. "This should stop the pain right away." C. "I will take my pill at meal time." D. "I will monitor for bleeding from my nose."

C

A nurse manager in a long-term care facility plans nutritional assessments of all residents. Which nutritional assessment activity does the nurse delegate to unlicensed assistive personnel (UAP) at the facility? A. Assessing residents' abilities to swallow B. Determining residents' functional status C. Measuring the daily food and fluid intake of residents D. Screening a portion of the residents with the Mini Nutritional Assessme

C

A patient is taking Bethanechol "Urecholine" for treatment of GERD. This is known as what type of drug? A. Proton-pump inhibitor B. Histamine receptor blocker C. Prokinetic D. Mucosal Healing Agent

C

A patient with Crohn's Disease is most likely to have the disease is what part of the GI tract? A. Rectum B. Duodenum of the small intestine C. Terminal Ileum D. Descending colon

C

A physician has prescribed a patient with a severe case of Crohn's Disease to take a drug that works by suppressing the immune system. This medication achieves this by blocking a protein that plays a role the inflammatory process. Which drug does this describe? A. Azathioprine B. Sulfasalazine C. Infliximab D. Prednisone

C

The doctor ordered for a complete blood count. After the test, Nurse Ray received the result from the laboratory. Which laboratory values will confirm the diagnosis of appendicitis? a. RBC 5.5 x 106/mm3 b. Hct 44 % c. WBC 13, 000/mm3 d. Hgb 15 g/dL"

C

The nurse evaluates the effectiveness of a paracentesis in a patient who has ascites. Which measurement is most important for the nurse to note? a. Cardiac output b. Blood pressure c. Abdominal girth d. Intake and output

C

The nurse is providing preoperative teaching for a 61-year-old man scheduled for an abdominal-perineal resection. Which information will the nurse include? a. Another surgery in 8 to 12 weeks will be used to create an ileal-anal reservoir. b. The patient will begin sitting in a chair at the bedside on the first postoperative day. c. The patient will drink polyethylene glycol lavage solution (GoLYTELY) preoperatively. d. IV antibiotics will be started at least 24 hours before surgery to reduce the bowel bacteria.

C

The nurse will teach a patient with chronic pancreatitis to take the prescribed pancrelipase (Viokase) a. at bedtime. b. in the morning. c. with each meal. d. for abdominal pain.

C

The physician orders a patient's T-Tube to be clamped 1 hour before and 1 hour after meals. You clamp the T-Tube as prescribed. While the tube is clamped which finding requires you to notify the physician? A. The T-Tube is not draining. B. The T-Tube tubing is below the patient's waist. C. The patient reports nausea and abdominal pain. D. The patient's stool is brown and formed.

C

When taking the blood pressure (BP) on the right arm of a patient with severe acute pancreatitis, the nurse notices carpal spasms of the patient's right hand. Which action should the nurse take next? a. Ask the patient about any arm pain. b. Retake the patient's blood pressure. c. Check the calcium level in the chart. d. Notify the health care provider immediately.

C

Which activity in the care of a 48-year-old female patient with a new colostomy could the nurse delegate to unlicensed assistive personnel (UAP)? a. Document the appearance of the stoma. b. Place a pouching system over the ostomy. c. Drain and measure the output from the ostomy. d. Check the skin around the stoma for breakdown.

C

The nurse is doing an admission assessment on a client with a history of duodenal ulcer. To determine whether the problem is currently active, the nurse should assess the client for which sign(s)/symptom(s) of duodenal ulcer? A. Weight loss B. Nausea and vomiting C. Pain relieved by food intake D. Pain radiating down the right arm

C A frequent symptom of duodenal ulcer is pain that is relieved by food intake. These clients generally describe the pain as a burning, heavy, sharp, or "hungry" pain that often localizes in the mid-epigastric area. The client with duodenal ulcer usually does not experience weight loss or nausea and vomiting. These symptoms are more typical in the client with a gastric ulcer.

A 58-year-old patient has just been admitted to the emergency department with nausea and vomiting. Which information requires the most rapid intervention by the nurse? a. The patient has been vomiting for 4 days. b. The patient takes antacids 8 to 10 times a day. c. The patient is lethargic and difficult to arouse. d. The patient has undergone a small intestinal resection.

C A lethargic patient is at risk for aspiration, and the nurse will need to position the patient to decrease aspiration risk. The other information is also important to collect, but it does not require as quick action as the risk for aspiration.

A 54-year-old man has just arrived in the recovery area after an upper endoscopy. Which information collected by the nurse is most important to communicate to the health care provider? a. The patient is very drowsy. b. The patient reports a sore throat. c. The oral temperature is 101.6° F. d. The apical pulse is 104 beats/minute.

C A temperature elevation may indicate that a perforation has occurred. The other assessment data are normal immediately after the procedure.

A patient with cholelithiasis needs to have the gallbladder removed. Which patient assessment is a contraindication for a cholecystectomy? A. Low-grade fever of 100° F and dehydration B. Abscess in the right upper quadrant of the abdomen C. Activated partial thromboplastin time (aPTT) of 54 seconds D. Multiple obstructions in the cystic and common bile duct

C An aPTT of 54 seconds is above normal and indicates insufficient clotting ability. If the patient had surgery, significant bleeding complications postoperatively are very likely. Fluids can be given to eliminate the dehydration; the abscess can be assessed, and the obstructions in the cystic and common bile duct would be relieved with the cholecystectomy.

The nurse is assessing a patient who had a total gastrectomy 8 hours ago. What information is most important to report to the health care provider? a. Absent bowel sounds b. Complaints of incisional pain c. Temperature 102.1° F (38.9° C) d. Scant nasogastric (NG) tube drainage

C An elevation in temperature may indicate leakage at the anastomosis, which may require return to surgery or keeping the patient NPO. The other findings are expected in the immediate postoperative period for patients who have this surgery.

A client has an appendectomy. This is an example of what kind of surgery? a. Diagnostic b. palliative c. ablative d. constructive

C Appendectomy is an example of ablative surgery. Diagnostic confirms or establishes a diagnosis, palliative relieves or reduces pain, and constructive restores function or appearance.

A 62-year-old patient has had a hemorrhoidectomy at an outpatient surgical center. Which instructions will the nurse include in discharge teaching? a. Maintain a low-residue diet until the surgical area is healed. b. Use ice packs on the perianal area to relieve pain and swelling. c. Take prescribed pain medications before a bowel movement is expected. d. Delay having a bowel movement for several days until healing has occurred.

C Bowel movements may be very painful, and patients may avoid defecation unless pain medication is taken before the bowel movement

A nurse is reviewing the health record of a client who has pancreatitis. The physical exam report by the provider indicates the presence of Cullen's sign. Which of the following is an appropriate action by the nurse to identify this finding? A. Tap lightly at the costovertebral margin on the client's back. B. Palpate the client's right lower quadrant. C. Inspect the skin around the umbilicus. D. Auscultate the area below the client's scapula.

C Cullen's sign is indicated by a bluish-grey discoloration in the periumbilical area.

The nurse is providing discharge instructions to a client following gastrectomy and should instruct the client to take which measure to assist in preventing dumping syndrome? A. Ambulate following a meal. B. Eat high-carbohydrate foods. C. Limit the fluids taken with meals. D. Sit in a high Fowler's position during meals.

C Dumping syndrome is a term that refers to a constellation of vasomotor symptoms that occurs after eating, especially following a gastrojejunostomy (Billroth II procedure). Early manifestations usually occur within 30 minutes of eating and include vertigo, tachycardia, syncope, sweating, pallor, palpitations, and the desire to lie down. The nurse should instruct the client to decrease the amount of fluid taken at meals and to avoid high-carbohydrate foods, including fluids such as fruit nectars; to assume a low Fowler's position during meals; to lie down for 30 minutes after eating to delay gastric emptying; and to take antispasmodics as prescribed.

The nurse determines that a patient has experienced the beneficial effects of therapy with famotidine (Pepcid) when which symptom is relieved? A. Nausea B. Belching C. Epigastric pain D. Difficulty swallowing

C Famotidine is an H2-receptor antagonist that inhibits parietal cell output of HCl acid and minimizes damage to gastric mucosa related to hyperacidity, thus relieving epigastric pain. Famotidine is not indicated for nausea, belching, and dysphagia.

The nurse is planning care for a patient who is chronically malnourished. Which action is appropriate for the nurse to delegate to unlicensed assistive personnel (UAP)? a. Assist the patient to choose high-nutrition items from the menu. b. Monitor the patient for skin breakdown over the bony prominences. c. Offer the patient the prescribed nutritional supplement between meals. d. Assess the patient's strength while ambulating the patient in the room.

C Feeding the patient and assisting with oral intake are included in UAP education and scope of practice. Assessing the patient and assisting the patient in choosing high-nutrition foods require licensed practical/vocational nurse (LPN/LVN)-or registered nurse (RN)-level education and scope of practice.

Marie, a 51-year-old woman, is diagnosed with cholecystitis. Which diet, when selected by the client, indicates that the nurse's teaching has been successful? A. 4-6 small meals of low-carbohydrate foods daily B. High-fat, high-carbohydrate meals C. Low-fat, high-carbohydrate meals D. High-fat, low protein meals

C For the client with cholecystitis, fat intake should be reduced. The calories from fat should be substituted with carbohydrates. Reducing carbohydrate intake would be contraindicated. Any diet high in fat may lead to another attack of cholecystitis.

The nurse instructs a 50-year-old woman about cholestyramine to reduce pruritis caused by gallbladder disease. Which statement by the patient to the nurse indicates she understands the instructions? A. "This medication will help me digest fats and fat-soluble vitamins." B. "I will apply the medicated lotion sparingly to the areas where I itch." C. "The medication is a powder and needs to be mixed with milk or juice." D. "I should take this medication on an empty stomach at the same time each day."

C For treatment of pruritus, cholestyramine may provide relief. This is a resin that binds bile salts in the intestine, increasing their excretion in the feces. Cholestyramine is in powder form and should be mixed with milk or juice before oral administration.

The nurse is caring for a client following a gastrojejunostomy (Billroth II procedure). Which postoperative prescription should the nurse question and verify? A. Leg exercises B. Early ambulation C. Irrigating the nasogastric tube D. Coughing and deep-breathing exercises

C In a gastrojejunostomy (Billroth II procedure), the proximal remnant of the stomach is anastomosed to the proximal jejunum. Patency of the nasogastric tube is critical for preventing the retention of gastric secretions. The nurse should never irrigate or reposition the gastric tube after gastric surgery, unless specifically prescribed by the health care provider. In this situation, the nurse should clarify the prescription. Options 1, 2, and 4 are appropriate postoperative interventions.

The nurse is reviewing the medication record of a client with acute gastritis. Which medication, if noted on the client's record, should the nurse question? A. Digoxin B. Furosemide C. Indomethacin D. Propranolol hydrochloride

C Indomethacin is a nonsteroidal antiinflammatory drug and can cause ulceration of the esophagus, stomach, or small intestine. Indomethacin is contraindicated in a client with gastrointestinal disorders. Digoxin is a cardiac medication. Furosemide is a loop diuretic. Propranolol hydrochloride is a beta-adrenergic blocking agent. Digoxin, furosemide, and propranolol are not contraindicated in clients with gastric disorders.

A client has been taking aluminum hydroxide 30 mL six times per day at home to treat his peptic ulcer. He tells the nurse that he has been unable to have a bowel movement for 3 days. Based on this information, the nurse would determine that which of the following is the most likely cause of the client's constipation? a. The client has not been including enough fiber in his diet b. The client needs to increase his daily exercise c. The client is experiencing a side effect of the aluminum hydroxide. d. The client has developed a gastrointestinal obstruction

C It is most likely that the client is experiencing a side effect of the antacid. Antacids with aluminum salt products, such as aluminum hydroxide, form insoluble salts in the body. These precipitate and accumulate in the intestines, causing constipation. Increasing dietary fiber intake or daily exercise may be a beneficial lifestyle change for the client but is not likely to relieve constipation caused by the aluminum hydroxide. Constipation, in isolation from other symptoms, is not a sign of bowel obstruction.

Following administration of a dose of metoclopramide (Reglan) to the patient, the nurse determines that the medication has been effective when what is noted? A. Decreased blood pressure B. Absence of muscle tremors C. Relief of nausea and vomiting D. No further episodes of diarrhea

C Metoclopramide is classified as a prokinetic and antiemetic medication. If it is effective, the patient's nausea and vomiting should resolve. Metoclopramide does not affect blood pressure, muscle tremors, or diarrhea.

The nurse is caring for a group of patients. Which patient is at highest risk for pancreatic cancer? A. A 38-year-old Hispanic female who is obese and has hyperinsulinemia B. A 23-year-old who has cystic fibrosis-related pancreatic enzyme insufficiency C. A 72-year-old African American male who has smoked cigarettes for 50 years D. A 19-year-old who has a 5-year history of uncontrolled type 1 diabetes mellitus

C Risk factors for pancreatic cancer include chronic pancreatitis, diabetes mellitus, age, cigarette smoking, family history of pancreatic cancer, high-fat diet, and exposure to chemicals such as benzidine. African Americans have a higher incidence of pancreatic cancer than whites. The most firmly established environmental risk factor is cigarette smoking. Smokers are two or three times more likely to develop pancreatic cancer as compared with nonsmokers. The risk is related to duration and number of cigarettes smoked.

The nurse is caring for a client with gastroesophageal reflux disease (GERD) and provides client education on measures to decrease GERD. Which statement made by the client indicates a need for further teaching? A. "I plan to eat 4 to 6 small meals a day." B. "I should sleep in the right side-lying position." C."I plan to have a snack 1 hour before going to bed." D. "I will stop having a glass of wine each evening with dinner."

C The control of GERD involves lifestyle changes to promote health and control reflux. These include eating 4 to 6 small meals a day; avoiding alcohol and smoking; sleeping in the right side-lying position to promote oxygenation and frequent swallowing to clear the esophagus; and avoiding eating at least 3 hours before going to bed because reflux episodes are most damaging at night.

To prepare a 56-year-old male patient with ascites for paracentesis, the nurse a. places the patient on NPO status. b. assists the patient to lie flat in bed. c. asks the patient to empty the bladder. d. positions the patient on the right side.q

C The patient should empty the bladder to decrease the risk of bladder perforation during the procedure. The patient would be positioned in Fowler's position and would not be able to lie flat without compromising breathing. Because no sedation is required for paracentesis, the patient does not need to be NPO.

A patient receiving parenteral nutrition is administered via the following routes except: A. Subclavian line. B. Central Venous Catheter. C. PICC (Peripherally inserted central catheter) line. D. PEG tube.

D Percutaneous endoscopic gastrostomy (PEG tube) is inserted into a person's stomach through the abdominal wall that is used to provide a means of feeding when oral intake is not adequate. While Parenteral nutrition bypasses the digestive system by the administration to the bloodstream

During change-of-shift report, the nurse learns about the following four patients. Which patient requires assessment first? a. 40-year-old with chronic pancreatitis who has gnawing abdominal pain b. 58-year-old who has compensated cirrhosis and is complaining of anorexia c. 55-year-old with cirrhosis and ascites who has an oral temperature of 102° F (38.8° C) d. 36-year-old recovering from a laparoscopic cholecystectomy who has severe shoulder pain

C This patient's history and fever suggest possible spontaneous bacterial peritonitis, which would require rapid assessment and interventions such as antibiotic therapy. The clinical manifestations for the other patients are consistent with their diagnoses and do not indicate complications are occurring.

The patient with chronic gastritis is being put on a combination of medications to eradicate H. pylori. Which drugs does the nurse know will probably be used for this patient? A. Antibiotic(s), antacid, and corticosteroid B. Antibiotic(s), aspirin, and antiulcer/protectant C. Antibiotic(s), proton pump inhibitor, and bismuth D. Antibiotic(s) and nonsteroidal antiinflammatory drugs (NSAIDs)

C To eradicate H. pylori, a combination of antibiotics, a proton pump inhibitor, and possibly bismuth (for quadruple therapy) will be used. Corticosteroids, aspirin, and NSAIDs are drugs that can cause gastritis and do not affect H. pylori.

A patient's peripheral parenteral nutrition (PN) bag is nearly empty and a new PN bag has not arrived yet from the pharmacy. Which intervention is the priority? a. Monitor the patient's capillary blood glucose until a new PN bag is hung. b. Flush the peripheral line with saline and wait until the new PN bag is available. c. Infuse 5% dextrose in water until the new PN bag is delivered from the pharmacy. d. Decrease the rate of the current PN infusion to 10 mL/hr until the new bag arrives.

C To prevent hypoglycemia, the nurse should infuse a 5% dextrose solution until the next PN bag can be started. Decreasing the rate of the ordered PN infusion is beyond the nurse's scope of practice. Flushing the line and then waiting for the next bag may lead to hypoglycemia. Monitoring the capillary blood glucose is appropriate but is not the priority.

A 22-year-old female patient with an exacerbation of ulcerative colitis is having 15 to 20 stools daily and has excoriated perianal skin. Which patient behavior indicates that teaching regarding maintenance of skin integrity has been effective? a. The patient uses incontinence briefs to contain loose stools. b. The patient asks for antidiarrheal medication after each stool. c. The patient uses witch hazel compresses to decrease irritation. d. The patient cleans the perianal area with soap after each stool.

C Witch hazel compresses are suggested to reduce anal irritation and discomfort. Incontinence briefs may trap diarrhea and increase the incidence of skin breakdown. Antidiarrheal medications are not given 15 to 20 times a day. The perianal area should be washed with plain water after each stool.

Pierre who is diagnosed with acute pancreatitis is under the care of Nurse Bryan. Which intervention should the nurse include in the care plan for the client? A. Administration of vasopressin and insertion of a balloon tamponade B. Preparation for a paracentesis and administration of diuretics C. Maintenance of nothing-by-mouth status and insertion of nasogastric (NG) tube with low intermittent suction D. Dietary plan of a low-fat diet and increased fluid intake to 2,000 ml/day

C With acute pancreatitis, the client is kept on nothing-by-mouth status to inhibit pancreatic stimulation and secretion of pancreatic enzymes. NG intubation with low intermittent suction is used to relieve nausea and vomiting, decrease painful abdominal distention, and remove hydrochloric acid. Vasopressin would be appropriate for a client diagnosed with bleeding esophageal varices. Paracentesis and diuretics would be appropriate for a client diagnosed with portal hypertension and ascites. A low-fat diet and increased fluid intake would further aggravate the pancreatitis.

The nurse has been assigned to care for a client diagnosed with peptic ulcer disease. Which assessment data require further intervention? A. Bowel sour s auscultated 15 times in 1 minute B. Belching after eating a heavy and fatty meal late at night C. A decrease in systolic BP of 20 mm Hg from lying to sitting D. A decreased frequency of distress located in the epigastric region

C. A decrease of 20 mm Hg in blood pressure after changing position from lying, to sitting, to standing is orthostatic hypotension. This could indicate client is bleeding.

Your recent admission has acute cholecystitis. The patient is awaiting a cholecystostomy. What signs and symptoms are associated with this condition? Select all that apply: A. Right lower quadrant pain with rebound tenderness B. Negative Murphy's Sign C. Epigastric pain that radiates to the right scapula D. Pain and fullness that increases after a greasy or spicy meal E. Fever F. Tachycardia G. Nausea

CDEFG

You're providing a community in-service about gastrointestinal disorders. During your teaching about cholecystitis, you discuss how cholelithiasis can lead to this condition. What are the risk factors for cholelithiasis that you will include in your teaching to the participants? Select all that apply: A. Being male B. Underweight C. Being female D. Older age E. Native American F. Caucasian G. Pregnant H. Family History I. Obesity

CDEGHI

The results of a patient's recent endoscopy indicate the presence of peptic ulcer disease (PUD). Which of the following teaching points should the nurse provide to the patient in light of his new diagnosis? A. You'll need to drink at least two to three glasses of milk daily. B."It would likely be beneficial for you to eliminate drinking alcohol." C. Many people find that a minced or pureed diet eases their symptoms of PUD. D. Your medications should allow you to maintain your present diet while minimizing symptoms

CORRECT ANSWER: B Although there is no specific recommended dietary modification for PUD, most patients find it necessary to make some sort of dietary modifications to minimize symptoms. Milk may exacerbate PUD and alcohol is best avoided because it can delay healing

A client with inflammatory bowel disease undergoes an ileostomy. On the first day after surgery, the nurse notes that the client's stoma appears dusky. How should the nurse interpret this finding? a) The ostomy bag should be adjusted. b) Blood supply to the stoma has been interrupted. c) An intestinal obstruction has occurred. d) This is a normal finding 1 day after surgery

Correct Answer: (B), Blood supply to the stoma has been interrupted An ileostomy stoma forms as the ileum is brought through the abdominal wall to the surface skin, creating an artificial opening for waste elimination. The stoma should appear cherry red, indicating adequate arterial perfusion. A dusky stoma suggests decreased perfusion. The nurse should interpret this finding as an indication that the stoma's blood supply is interuppted, which may lead to tissue damage or necrosis. A dusky stoma isn't a normal finding 1 day after surgery. Adjusting the ostomy bag wouldn't affect stoma color, which depends on blood supply to the area. An intestinal obstruction also wouldn't change stoma color

The male client tells the nurse he has been experiencing "heartburn" at night that awakens him. Which assessment question should the nurse ask? A. How much weight have you gained recently? B. What have you done to alleviate the heartburn? C. Do you consume many milk and dairy products? D Have you been around anyone with a stomach virus

Correct answer is B, Most clients with GERD have been self medicating with over-the counter medications prior to seeking advice from a healthcare provider. It is important to know what the client has been using to treat the problem

When assessing the client with the diagnosis of peptic ulcer disease, which physical examination should the nurse implement first? 1. Auscultate the client's bowel sounds in all four quadrants. 2.Palpate the abdominal area for tenderness. 3.Percuss the abdominal borders to identify organs. 4.Assess the tender area progressing to nontender

Correct answer: #1. Auscultation should be used prior to palpa-tion or percussion when assessing the abdomen. If the nurse manipulates the abdomen, the bowel sounds can be altered and give false information

The client with a hiatal hernia chronically experiences heartburn following meals. The nurse plans to teach the client to avoid which action because it is contraindicated with hiatal hernia? 1. Lying recumbent following meals 2. Taking in small, frequent, bland meals 3. Raising the head of the bed on 6-inch blocks 4. Taking H2-receptor antagonist medication

Correct answer: 1 Laying recumbant following meals or at night will cause reflux and pain. Relief is usually achieved with the intake of small, bland meals, use of H2 receptor antagonists and antacids, and elevation of the thorax after meals and during sleep

Which of the following types of gastritis ic associated with Helicobacter pylori and duodenal ulcers? 1. Erosive (hemorrhagic) gastritis 2. Fundic gland gastritis (type A) 3. Antral gland gastritis (type B) 4. Aspiring-induced gastric ulcer

Correct answer: 3. Antral gland gastritis ( type B). Rationale: Antral gland gastritis is the most common form of gastritis and is associated with Helicobacter pylori and duodenal ulcers

What should the nurse instruct the patient to do to best enhance the effectiveness of a daily dose of docusate sodium (Colace)? a. Take a dose of mineral oil at the same time. b. Add extra salt to food on at least one meal tray. c. Ensure dietary intake of 10 g of fiber each day. d. Take each dose with a full glass of water or other liquid.

Correct answer: D Rationale: Docusate lowers the surface tension of stool, permitting water and fats to penetrate and soften the stool for easier passage. The patient should take the dose with a full glass of water and should increase overall fluid intake, if able, to enhance effectiveness of the medication. Dietary fiber intake should be a minimum of 20 g daily to prevent constipation. Mineral oil and extra salt are not recommended.

The nurse is monitoring a client with a diagnosis of peptic ulcer. Which assessment finding would most likely indicate perforation of the ulcer? A. Bradycardia B. Numbness in the legs C. Nausea and vomiting D. A rigid, boardlike abdomen

D Perforation of an ulcer is a surgical emergency and is characterized by sudden, sharp, intolerable severe pain beginning in the mid-epigastric area and spreading over the abdomen, which becomes rigid and boardlike. Nausea and vomiting may occur. Tachycardia may occur as hypovolemic shock develops. Numbness in the legs is not an associated finding.

The appropriate collaborative therapy for the patient with acute diarrhea caused by a viral infection is to a. increase fluid intake. b. administer an antibiotic. c. administer antimotility drugs. d. quarantine the patient to prevent spread of the virus.

Correct answer: a Rationale: Acute diarrhea resulting from infectious causes (e.g., virus) is usually self-limiting. The major concerns are transmission prevention, fluid and electrolyte replacement, and resolution of the diarrhea. Antidiarrheal agents are contraindicated in the treatment of infectious diarrhea because they potentially prolong exposure to the infectious organism. Antibiotics are rarely used to treat acute diarrhea. To prevent transmission of diarrhea caused by a virus, hands should be washed before and after contact with the patient and when body fluids of any kind are handled. Vomitus and stool should be flushed down the toilet, and contaminated clothing should be washed immediately with soap and hot water.

A stroke patient who primarily uses a wheelchair for mobility has diarrhea with fecal incontinence. What should the nurse assess first? a. Fecal impaction b. Perineal hygiene c. Dietary fiber intake d. Antidiarrheal agent use

Correct answer: a Rationale: Patients with limited mobility are at risk for fecal impactions due to constipation that may lead to liquid stool leaking around the hardened impacted feces, so assessing for fecal impaction is the priority. Perineal hygiene can be assessed at the same time. Assessing the dietary fiber and fluid intake and antidiarrheal agent use will be assessed and considered next.

The nurse is preparing to insert a nasogastric (NG) tube into a 68-year-old female patient who is nauseated and vomiting. She has an abdominal mass and suspected small intestinal obstruction. The patient asks the nurse why this procedure is necessary. What response by the nurse is most appropriate? a. "The tube will help to drain the stomach contents and prevent further vomiting." b. "The tube will push past the area that is blocked and thus help to stop the vomiting." c. "The tube is just a standard procedure before many types of surgery to the abdomen." d. "The tube will let us measure your stomach contents so that we can plan what type of IV fluid replacement would be best."

Correct answer: a Rationale: The NG tube is used to decompress the stomach by draining stomach contents and thereby prevent further vomiting. The NG tube will not push past the blocked area. Potential surgery is not currently indicated. The location of the obstruction will determine the type of fluid to use, not measure the amount of stomach contents.

Two days following a colectomy for an abdominal mass, a patient reports gas pains and abdominal distention. The nurse plans care for the patient based on the knowledge that the symptoms are occurring as a result of a. impaired peristalsis. b. irritation of the bowel. c. nasogastric suctioning. d. inflammation of the incision site.

Correct answer: a Rationale: Until peristalsis returns to normal following anesthesia, the patient may experience slowed gastrointestinal motility leading to gas pains and abdominal distention. Irritation of the bowel, nasogastric suctioning, and inflammation of the surgical site do not cause gas pains or abdominal distention.

Assessment findings suggestive of peritonitis include a. rebound abdominal pain. b. a soft, distended abdomen. c. dull, continuous abdominal pain. d. observing that the patient is restless.

Correct answer: a Rationale: With peritoneal irritation, the abdomen is hard, like a board, and the patient has severe abdominal pain that is worse with any sudden movement. The patient lies very still. Palpating the abdomen and releasing the hands suddenly causes sudden movement within the abdomen and severe pain. This is called rebound tenderness.

The nurse is planning care for a 68-year-old patient with an abdominal mass and suspected bowel obstruction. Which factor in the patient's history increases the patient's risk for colorectal cancer? a. Osteoarthritis b. History of colorectal polyps c. History of lactose intolerance d. Use of herbs as dietary supplements

Correct answer: b Rationale: A history of colorectal polyps places this patient at risk for colorectal cancer. *This tissue can degenerate over time and become malignant*. Osteoarthritis, lactose intolerance, and the use of herbs do not pose additional risk to the patient.

What information would have the highest priority to be included in preoperative teaching for a 68-year-old patient scheduled for a colectomy? a. How to care for the wound b. How to deep breathe and cough c. The location and care of drains after surgery d. Which medications will be used during surgery

Correct answer: b Rationale: Because anesthesia, an abdominal incision, and pain can impair the patient's respiratory status in the postoperative period, it is of high priority to teach the patient to cough and deep breathe. Otherwise, the patient could develop atelectasis and pneumonia, which would delay early recovery from surgery and hospital discharge. Care for the wound and location and care of the drains will be briefly discussed preoperatively, but done again with higher priority after surgery. Knowing which drugs will be used during surgery may not be meaningful to the patient and should be reviewed with the patient by the anesthesiologist.

The nurse should administer an as-needed dose of magnesium hydroxide (MOM) after noting what information while reviewing a patient's medical record? a. Abdominal pain and bloating b. No bowel movement for 3 days c. A decrease in appetite by 50% over 24 hours d. Muscle tremors and other signs of hypomagnesemia

Correct answer: b Rationale: MOM is an osmotic laxative that produces a soft, semisolid stool usually within 15 minutes to 3 hours. This medication would benefit the patient who has not had a bowel movement for 3 days. MOM would not be given for abdominal pain and bloating, decreased appetite, or signs of hypomagnesemia.

A colectomy is scheduled for a 38-year-old woman with ulcerative colitis. The nurse should plan to include what prescribed measure in the preoperative preparation of this patient? a. Instruction on irrigating a colostomy b. Administration of a cleansing enema c. A high-fiber diet the day before surgery d. Administration of IV antibiotics for bowel preparation

Correct answer: b Rationale: Preoperative preparation for bowel surgery typically includes *bowel cleansing with antibiotics, such as oral neomycin and cleansing enemas, including Fleet enemas*. Instructions to irrigate the colostomy will be done postoperatively. Oral antibiotics are given preoperatively, and an IV antibiotic may be used in the OR. A clear liquid diet will be used the day before surgery with the bowel cleansing.

True or False: Crohn's Disease and ulcerative colitis are two forms of IBS (irritable bowel syndrome).

F

In planning care for the patient with Crohn's disease, the nurse recognizes that a major difference between ulcerative colitis and Crohn's disease is that Crohn's disease a. frequently results in toxic megacolon. b. causes fewer nutritional deficiencies than ulcerative colitis. c. often recurs after surgery, whereas ulcerative colitis is curable with a colectomy. d. is manifested by rectal bleeding and anemia more frequently than is ulcerative colitis.

Correct answer: c Rationale: Ulcerative colitis affects only the colon and rectum; it can cause megacolon and rectal bleeding, but not nutrient malabsorption. Surgical removal of the colon and rectum cures it. Crohn's disease usually involves the ileum, where bile salts and vitamin cobalamin are absorbed. After surgical treatment, disease recurrence at the site is common.

The nurse is preparing to administer a dose of bisacodyl (Dulcolax). In explaining the medication to the patient, the nurse would explain that it acts in what way? a. Increases bulk in the stool b.Lubricates the intestinal tract to soften feces c. Increases fluid retention in the intestinal tract d. Increases peristalsis by stimulating nerves in the colon wall

Correct answer: d Rationale: Bisacodyl is a stimulant laxative that aids in producing a bowel movement by irritating the colon wall and stimulating enteric nerves. It is available in oral and suppository forms. Fiber and bulk forming drugs increase bulk in the stool; water and stool softeners soften feces, and saline and osmotic solutions cause fluid retention in the intestinal tract.

The nurse is caring for a 68-year-old patient admitted with abdominal pain, nausea, and vomiting. The patient has an abdominal mass, and a bowel obstruction is suspected. The nurse auscultating the abdomen listens for which type of bowel sounds that are consistent with the patient's clinical picture? a. Low-pitched and rumbling above the area of obstruction b. High-pitched and hypoactive below the area of obstruction c. Low-pitched and hyperactive below the area of obstruction d. High-pitched and hyperactive above the area of obstruction

Correct answer: d Rationale: Early in intestinal obstruction, the patient's bowel sounds are hyperactive and high-pitched, sometimes referred to as "tinkling" above the level of the obstruction. *This occurs because peristaltic action increases to "push past" the area of obstruction*. As the obstruction becomes complete, bowel sounds decrease and finally become absent.

The nurse would question the use of which cathartic agent in a patient with renal insufficiency? a. Bisacodyl (Dulcolax) b. Lubiprostone (Amitiza) c. Cascara sagrada (Senekot) d. Magnesium hydroxide (Milk of Magnesia)

Correct answer: d Rationale: Milk of Magnesia may cause hypermagnesemia in patients with renal insufficiency. The nurse should question this order with the health care provider. Bisacodyl, lubiprostone, and cascara sagrada are safe to use in patients with renal insufficiency as long as the patient is not currently dehydrated.

The nurse is preparing to administer a scheduled dose of docusate sodium (Colace) when the patient reports an episode of loose stool and does not want to take the medication. What is the appropriate action by the nurse? a. Write an incident report about this untoward event. b. Attempt to have the family convince the patient to take the ordered dose. c. Withhold the medication at this time and try to administer it later in the day. d. Chart the dose as not given on the medical record and explain in the nursing progress notes.

Correct answer: d Rationale: Whenever a patient refuses medication, the dose should be charted as not given with an explanation of the reason documented in the nursing progress notes. In this instance, the refusal indicates good judgment by the patient, and the patient should not be encouraged to take it today.

The nurse performs a detailed assessment of the abdomen of a patient with a possible bowel obstruction, knowing that manifestations of an obstruction in the large intestine are (select all that apply) a. persistent abdominal pain. b. marked abdominal distention. c. diarrhea that is loose or liquid. d. colicky, severe, intermittent pain. e. profuse vomiting that relieves abdominal pain.

Correct answers: a, b Rationale: With lower intestinal obstructions, abdominal distention is markedly increased and pain is persistent. Onset of a large intestine obstruction is gradual, vomiting is rare, and there is usually absolute constipation, not diarrhea.

When a 35-year-old female patient is admitted to the emergency department with acute abdominal pain, which possible diagnosis should you consider that may be the cause of her pain (select all that apply)? a. Gastroenteritis b. Ectopic pregnancy c. Gastrointestinal bleeding d. Irritable bowel syndrome e. Inflammatory bowel disease

Correct answers: a, b, c, d, e Rationale: All these conditions could cause acute abdominal pain.

The teaching plan for the patient being discharged following an acute episode of upper GI bleeding will concern information concerning the importance of (select all that apply) a. only taking aspirin with milk or bread products b. avoiding taking aspirin and drugs containing aspirin c. taking only drugs prescribed by the health care provider d. taking all drugs 1 hour before mealtime to prevent further bleeding e. reading all OTC drug labels to avoid those containing stearic acid and calcium

Correct answers: b, c Rationale: Before discharge, the patient with upper gastrointestinal (GI) bleeding and the caregiver should be taught how to avoid future bleeding episodes. Ulcer disease, drug or alcohol abuse, and liver and respiratory diseases can cause upper GI bleeding. Help make the patient and caregiver aware of the consequences of noncompliance with drug therapy. Emphasize that no drugs (especially aspirin and nonsteroidal antiinflammatory drugs [NSAIDs]) other than those prescribed by the health care provider should be taken. Smoking and alcohol should be eliminated because they are sources of irritation and interfere with tissue repair.

Which clinical manifestations of inflammatory bowel disease are common to both patients with ulcerative colitis (UC) and Crohn's disease (select all that apply)? a. Restricted to rectum b. Strictures are common. c. Bloody, diarrhea stools d. Cramping abdominal pain e. Lesions penetrate intestine.

Correct answers: c, d Rationale: Clinical manifestations of UC and Crohn's disease include bloody diarrhea, cramping abdominal pain, and nutritional disorders. Intestinal lesions associated with UC are usually restricted to the rectum before moving into the colon. Lesions that penetrate the intestine or cause strictures are characteristic of Crohn's disease.

A nurse is making a home health visit and finds the client experiencing right lower quadrant abdominal pain, which has decreased in intensity over the last day. The client also has a rigid abdomen and a temperature of 103.6 F. The nurse should intervene by: a) administer Tylenol (acetaminophen) for the elevated temperature b) advising the client to increase oral fluids c) asking the client when she last had a bowel movement d) notifying the physician

D

A nurse is preparing to hang the initial bag of the parenteral nutrition (PN) solution via the central line of a malnourished client. The nurse ensure the availability of which medical equipment before hanging the solution? A. Glucometer. B. Dressing tray. C. Nebulizer. D. Infusion Pump

D

A nurse is reviewing a new prescription for ursodiol (Ursodeoxycholic Acid) with a client who has cholelithiasis. Which of the following should be included in the teaching? A. This medication reduces biliary spasms. B. This medication reduces inflammation in the biliary tract. C. This medication dilates the bile duct to promote passage of bile. D. This medication dissolves gall stones.

D

A patient experiencing a flare-up with Crohn's Disease is ordered complete bowel rest by the physician. You are administering TPN (total parental nutrition) per physician order. When developing the patient's nursing plan of care, which nursing diagnosis is MOST important to include in the care plan? A. Risk for allergy response B. Risk for unstable blood glucose level C. Risk for imbalance nutrition: more than body requirements D. Risk for imbalanced nutrition: less than body requirements

D

After providing education to a patient with GERD. You ask the patient to list 4 things they can do to prevent or alleviate signs and symptoms of GERD. Which statement is INCORRECT? A. "It is best to try to consume small meals throughout the day than eat 3 large ones." B. "I'm disappointed that I will have to limit my intake of peppermint and spearmint because I love eating those types of hard candies." C. "It is important I avoid eating right before bedtime." D. "I will try to lie down after eating a meal to help decrease pressure on the lower esophageal sphincter."

D

After several days of antibiotic therapy, an older hospitalized patient develops watery diarrhea. Which action should the nurse take first? a. Notify the health care provider. b. Obtain a stool specimen for analysis. c. Teach the patient about handwashing. d. Place the patient on contact precautions.

D

The ED nurse has inspected, auscultated, and palpated the abdomen with no obvious abnormalities, except pain. When the nurse palpates the abdomen for rebound tenderness, there is severe pain. The nurse should know that this could indicate what problem? A) Hepatic cirrhosis B) Hypersplenomegaly C) Gall bladder distention D) Peritoneal inflammation

D

The client with peptic ulcer disease is scheduled for a pyloroplasty. The client asks the nurse about the procedure. The nurse plans to respond knowing that a pyloroplasty involves: a. Cutting the vagus nerve b. Removing the distal portion of the stomach c. Removal of the ulcer and a large portion of the cells that produce hydrochloric acid d. An incision and resuturing of the pylorus to relax the muscle and enlarge the opening from the stomach to the duodenum

D

You're providing teaching to a patient who has been newly diagnosed with Crohn's Disease. Which statement by the patient's spouse requires re-education? A. "Crohn's Disease can be scattered throughout the GI tract in patches with some areas appearing healthy while others are diseased." B. "There is no cure for Crohn's Disease." C. "Strictures are a common complication with Crohn's Disease." D. "Crohn's Disease can cause the haustra of the large intestine to lose its form."

D

a pt with acute pacreatitis is NPO and has been receiving on IV hydration. which laboratory result indicates the need to consult the dietitian for nutritional support? a. potassium 4.2 mEq/L b. sodium 130 mEq/L c. fasting glucose 82 mg/dL d. serum albumin 2.9g/dL

D

the nurse is caring for a pt with chronic pancreatitis, the nurse would expect an elevation in which of the following lab tests? a. serum bilirubin b. serum calcium c. serum albumin d. serum amylase

D

A nurse is completing an admission assessment of a client who has pancreatitis. Which of the following is an expected finding? A. Pain in right upper quadrant radiating to right shoulder B. Report of pain being worse when sitting upright C. Pain relieved with defecation D. Epigastric pain radiating to left shoulder

D A client who has pancreatitis will report severe, boring epigastric pain that radiates to the back, left flank, or left shoulder.

A client has just had surgery to create an ileostomy. The nurse assesses the client in the immediate postoperative period for which most frequent complication of this type of surgery? A. Folate deficiency B. Malabsorption of fat C. Intestinal obstruction D. Fluid and electrolyte imbalance

D A frequent complication that occurs following ileostomy is fluid and electrolyte imbalance. The client requires constant monitoring of intake and output to prevent this from occurring. Losses require replacement by intravenous infusion until the client can tolerate a diet orally. Intestinal obstruction is a less frequent complication. Fat malabsorption and folate deficiency are complications that could occur later in the postoperative period.

Which assessment finding is of most concern for a 46-year-old woman with acute pancreatitis? a. Absent bowel sounds b. Abdominal tenderness c. Left upper quadrant pain d. Palpable abdominal mass

D A palpable abdominal mass may indicate the presence of a pancreatic abscess, which will require rapid surgical drainage to prevent sepsis. Absent bowel sounds, abdominal tenderness, and left upper quadrant pain are common in acute pancreatitis and do not require rapid action to prevent further complications.

Which clinical manifestation would the nurse expect a client diagnosed with acute cholecystitis to exhibit? A. Jaundice, dark urine, and steatorrhea B. Acute right lower quadrant (RLQ) pain, diarrhea, and dehydration C. Ecchymosis petechiae, and coffee-ground emesis D. Nausea, vomiting, and anorexia

D Acute cholecystitis is an acute inflammation of the gallbladder commonly manifested by the following: anorexia, nausea, and vomiting; biliary colic; tenderness and rigidity the right upper quadrant (RUQ) elicited on palpation (e.g., Murphy's sign); fever; fat intolerance; and signs and symptoms of jaundice. Ecchymosis, petechiae, and coffee-ground emesis are clinical manifestations of esophageal bleeding. The coffee-ground appearance indicates old bleeding. Jaundice, dark urine, and steatorrhea are clinical manifestations of the icteric phase of hepatitis.

A nurse is caring a client who disconnected the tubing of the parenteral nutrition from the central line catheter. A nurse suspects an occurrence of an air embolism. Which of the following is an appropriate position for the client in this kind of situation? A. On the right side, with head higher than the feet. B. On the right side, with head lower than the feet. C. On the left side, with the head higher than the feet. D. On the left side, with head lower than the feet.

D Air embolism happens when air enters the catheter system when the IV tubing disconnects. If it is suspected, the client should be placed in a left-side-lying position. The head should be lower than the feet. This position will lessen the effect of the air traveling as a bolus to the lungs by trapping it on the right side of the heart.

A client with an exacerbation of ulcerative colitis has been placed on total parenteral nutrition (TPN). The client asks why nutrition is being supplied in this manner and not by mouth. What is the nurse's best response? A. "TPN contains a high percentage of glucose that is more readily absorbed into the bloodstream than into the ulcerated colon." B. "TPN will be given in addition to your meals to help you gain any weight that you may have lost through diarrhea." C. "TPN is considered an elemental formula and, as such, is easier to digest." D. "TPN will be given during this period to allow your bowel to rest."

D Bowel rest during severe exacerbations of ulcerative colitis is part of the nonsurgical management of the disease.

After a colonoscopy, a client reports that he is experiencing abdominal fullness and cramping. What is the nurse's best action? A. Insert a rectal tube to assist in passing of flatus. B. Notify the physician immediately because a bowel perforation is suspected. C. Have the client consume only liquids for 4 to 6 hours after the test. D. Explain to the client that this feeling can be expected for several hours after the test.

D Feelings of abdominal fullness, cramping, and the passing of flatus are normally expected after a colonoscopy.

A 52-year-old man was referred to the clinic due to increased abdominal girth. He is diagnosed with ascites by the presence of a fluid thrill and shifting dullness on percussion. After administering diuretic therapy, which nursing action would be most effective in ensuring safe care? A. Measuring serum potassium for hyperkalemia B. Assessing the client for hypervolemia C. Measuring the client's weight weekly D. Documenting precise intake and output

D For the client with ascites receiving diuretic therapy, careful intake and output measurement is essential for safe diuretic therapy. Diuretics lead to fluid losses, which if not monitored closely and documented, could place the client at risk for serious fluid and electrolyte imbalances. Hypokalemia, not hyperkalemia, commonly occurs with diuretic therapy. Because urine output increases, a client should be assessed for hypovolemia, not hypervolemia. Weights are also an accurate indicator of fluid balance. However, for this client, weights should be obtained daily, not weekly.

The nurse receives the following information about a 51-year-old woman who is scheduled for a colonoscopy. Which information should be communicated to the health care provider before sending the patient for the procedure? a. The patient has a permanent pacemaker to prevent bradycardia. b. The patient is worried about discomfort during the examination. c. The patient has had an allergic reaction to shellfish and iodine in the past. d. The patient refused to drink the ordered polyethylene glycol (GoLYTELY).

D If the patient has had inadequate bowel preparation, the colon cannot be visualized and the procedure should be rescheduled. Because contrast solution is not used during colonoscopy, the iodine allergy is not pertinent. A pacemaker is a contraindication to magnetic resonance imaging (MRI), but not to colonoscopy. The nurse should instruct the patient about the sedation used during the examination to decrease the patient's anxiety about discomfort

The nurse would question the use of which cathartic agent in a patient with renal insufficiency? A. Bisacodyl (Dulcolax) B. Lubiprostone (Amitiza) C. Cascara sagrada (Senekot) D. Magnesium hydroxide (Milk of Magnesia)

D Milk of Magnesia may cause hypermagnesemia in patients with renal insufficiency. The nurse should question this order with the health care provider. Bisacodyl, lubiprostone, and cascara sagrada are safe to use in patients with renal insufficiency as long as the patient is not currently dehydrated.

A client with Crohn's disease is experiencing acute pain, and the nurse provides information about measures to alleviate the pain. Which statement by the client indicates the need for further teaching? A. "I know I can massage my abdomen." B. "I will continue using antispasmodic medication." C. "One of the best things I can do is use relaxation techniques." D. "The best position for me is to lie supine with my legs straight."

D Pain associated with Crohn's disease is alleviated by the use of analgesics and antispasmodics and also by practicing relaxation techniques, applying local cold or heat to the abdomen, massaging the abdomen, and lying with the legs flexed. Lying with the legs extended is not useful because it increases the muscle tension in the abdomen, which could aggravate inflamed intestinal tissues as the abdominal muscles are stretched.

Bobby, a 13 year old is being seen in the emergency room for possible appendicitis. An important nursing action to perform when preparing Bobby for an appendectomy is to:" a) administer saline enemas to cleanse the bowels b) apply heat to reduce pain c) measure abdominal girth d) continuously monitor pain

D Pain is closely monitored in appendicitis. In most cases, pain medication is not given until prior to surgery or until the diagnosis is confirmed to be able to closely monitor the progression of the disease. A sudden change in the character of pain may indicate rupture or bowel perforation. Administering an enema or applying heat may cause perforation and abdominal girth may not change with appendicitis.

The nurse obtains an admission history for a client with suspected peptic ulcer disease (PUD). Which client factor documented by the nurse would increase the risk for PUD? A. Recently retired from a job B. Significant other has a gastric ulcer C. Occasionally drinks 1 cup of coffee in the morning D. Takes nonsteroidal antiinflammatory drugs (NSAIDs) for osteoarthritis

D Risk factors for PUD include Helicobacter pylori infection, smoking (nicotine), chewing tobacco, corticosteroids, aspirin, NSAIDs, caffeine, alcohol, and stress. When an NSAID is taken as often as is typical for osteoarthritis, it will cause problems with the stomach. Certain medical conditions such as Crohn's disease, Zollinger-Ellison syndrome, and hepatic and biliary disease also can increase the risk for PUD by changing the amount of gastric and biliary acids produced. Recent retirement should decrease stress levels rather than increase them. Ulcer disease in a first-degree relative also is associated with increased risk for an ulcer. A significant other is not a first-degree relative; therefore, no genetic connection is noted in this relationship. Although caffeinated drinks are a known risk factor for PUD, the option states that the client drinks 1 cup of coffee occasionally.

A client who is receiving total enteral nutrition (TEN) exhibits acute confusion and shallow breathing and says, "I feel weak." As the client begins to have a generalized seizure, how does the nurse interpret this client's signs and symptoms? A. The enteral tube is misplaced or dislodged. B. Abdominal distention is present. C. A fluid and electrolyte imbalance is present. D. This is refeeding syndrome.

D Symptoms of refeeding syndrome include shallow respirations, weakness, acute confusion, seizures, and increased bleeding tendency.

A client has a large, deep duodenal ulcer diagnosed by endoscopy. Which sign or symptom indicative of a complication should the nurse look for during the client's postprocedure assessment? A. Bradycardia B. Nausea and vomiting C. Numbness in the legs D. A rigid, boardlike abdomen

D The client with a large, deep duodenal ulcer is at risk for perforation of the ulcer. If this occurs, the client will experience sudden, sharp, intolerable severe pain beginning in the midepigastric area and spreading over the abdomen, which then becomes rigid and boardlike. Tachycardia, not bradycardia, may occur as hypovolemic shock develops. Nausea and vomiting may not occur if the pyloric sphincter is intact. Numbness in the legs is not an associated finding.

A client with acute ulcerative colitis requests a snack. Which is the most appropriate snack for this client? A. Carrots and ranch dip B. Whole-grain cereal and milk C. A cup of popcorn and a cola drink D. Applesauce and a graham cracker

D The diet for the client with ulcerative colitis should be low fiber (low residue). The nurse should avoid providing foods such as whole-wheat grains, nuts, and fresh fruits or vegetables. Typically, lactose-containing foods also are poorly tolerated. The client also should avoid caffeine, pepper, and alcohol.

After vertical banded gastroplasty, a 42-year-old male patient returns to the surgical nursing unit with a nasogastric tube to low, intermittent suction and a patient-controlled analgesia (PCA) machine for pain control. Which nursing action should be included in the postoperative plan of care? a. Offer sips of fruit juices at frequent intervals. b. Irrigate the nasogastric (NG) tube frequently. c. Remind the patient that PCA use may slow the return of bowel function. d. Support the surgical incision during patient coughing and turning in bed.

D The incision should be protected from strain to decrease the risk for wound dehiscence. The patient should be encouraged to use the PCA because pain control will improve the cough effort and patient mobility. NG irrigation may damage the suture line or overfill the stomach pouch. Sugar-free clear liquids are offered during the immediate postoperative time to decrease the risk for dumping syndrome.

Which assessment should the nurse perform first for a patient who just vomited bright red blood? a. Measuring the quantity of emesis b. Palpating the abdomen for distention c. Auscultating the chest for breath sounds d. Taking the blood pressure (BP) and pulse

D The nurse is concerned about blood loss and possible hypovolemic shock in a patient with acute gastrointestinal (GI) bleeding. BP and pulse are the best indicators of these complications. The other information is important to obtain, but BP and pulse rate are the best indicators for assessing intravascular volume

A client with complaints of right lower quadrant pain is admitted to the emergency department. Blood specimens are drawn and sent to the laboratory. Which laboratory finding should be reported to the physician immediately? a) Hematocrit 42% b) Serum potassium 4.2 mEq/L c) Serum sodium 135 mEq/L d) White blood cell (WBC) count 22.8/mm3.

D The nurse should report the elevated WBC count. This finding, which is a sign of infection, indicates that the client's appendix might have ruptured. Hematocrit of 42%, serum potassium of 4.2 mEq/L, and serum sodium of 135 mEq/L are within normal limits. Alterations in these levels don't indicate appendicitis.

A client is admitted with a diagnosis of acute appendicitis. When assessing the abdomen, the nurse would expect to find rebound tenderness at which location? a) Left lower quadrant b) Left upper quadrant c) Right upper quadrant d) Right lower quadrant

D The pain of acute appendicitis localizes in the right lower quadrant (RLQ) at McBurney's point, an area midway between the umbilicus and the right iliac crest. Often, the pain is worse when manual pressure near the region is suddenly released, a condition called rebound tenderness.

When caring for a patient with a history of a total gastrectomy, the nurse will monitor for a. constipation. b. dehydration. c. elevated total serum cholesterol. d. cobalamin (vitamin B12) deficiency.

D The patient with a total gastrectomy does not secrete intrinsic factor, which is needed for cobalamin (vitamin B12) absorption. Because the stomach absorbs only small amounts of water and nutrients, the patient is not at higher risk for dehydration, elevated cholesterol, or constipation.

A 27-year-old female patient is admitted to the hospital for evaluation of right lower quadrant abdominal pain with nausea and vomiting. Which action should the nurse take? a. Encourage the patient to sip clear liquids. b. Assess the abdomen for rebound tenderness. c. Assist the patient to cough and deep breathe. d. Apply an ice pack to the right lower quadrant.

D The patient's clinical manifestations are consistent with appendicitis, and application of an ice pack will decrease inflammation at the area. Checking for rebound tenderness frequently is unnecessary and uncomfortable for the patient. The patient should be NPO in case immediate surgery is needed. The patient will need to know how to cough and deep breathe postoperatively, but coughing will increase pain at this time.

A 45-year-old patient is admitted to the emergency department with severe abdominal pain and rebound tenderness. Vital signs include temperature 102° F (38.3° C), pulse 120, respirations 32, and blood pressure (BP) 82/54. Which prescribed intervention should the nurse implement first? a. Administer IV ketorolac (Toradol) 15 mg. b. Draw blood for a complete blood count (CBC). c. Obtain a computed tomography (CT) scan of the abdomen. d. Infuse 1 liter of lactated Ringer's solution over 30 minutes.

D The priority for this patient is to treat the patient's hypovolemic shock with fluid infusion. The other actions should be implemented after starting the fluid infusion.

During the assessment of a patient with acute abdominal pain, the nurse should: a. perform deep palpation before auscultation b. obtain blood pressure and pulse rate to determine hypervolemic changes c. auscultate bowel sounds because hyperactive bowel sounds suggest paralytic ileus d. measure body temperature because an elevated temperature may indicate an inflammatory or infectious process.

D for the patient complaining of acute abdominal pain, nurse should take vital signs immediately. Increased pulse and decreasing blood pressure are indicative of hypovolemia. An elevated temperature suggests an inflammatory infectious process. Intake and output measurements provide essential information about the adequate of vascular volume. Inspect abdomen first and then auscultate bowel sounds. Palpation is performed next and should be gentle.

Which problems should the nurse include in the plan of care for the client diagnosed with peptic ulcer disease to observe for physiological complications? A. Alteration in bowel elimination patterns B. Knowledge deficit in the causes of ulcers C. Inability to cope with changing family roles D. Potential for alteration in gastric emptying

D. Potential for alteration in gastric emptying is caused by edema or scarring associated with an ulcer, which may cause a feeling of "fullness", vomiting of undigested food or abdominal distention

A nurse is caring for a client who will perform fecal occult blood testing at home. Which of the following should the nurse include when explaining the procedure to the client? A. Eating more protein is recommended prior to testing. B. One stool specimen is sufficient for testing. C. A red color change indicates a positive test. D. The specimen cannot be contaminated with urine.

D. The specimen cannot be contaminated with urine. For fecal occult blood testing at home, the stool specimens cannot be contaminated with water or urine; three specimens from three different bowel movements are required; some proteins such as red meat, fish, and poultry can alter the test results; and a blue color indicates a positive guaiac or presence of fecal occult blood not red. Chapter 45; Page 1046

The nurse is caring for a 47-year-old female patient who is comatose and is receiving continuous enteral nutrition through a soft nasogastric tube. The nurse notes the presence of new crackles in the patient's lungs. In which order will the nurse take action? a. Check the patient's oxygen saturation. b. Notify the patient's health care provider. c. Measure the tube feeding residual volume. d. Stop administering the continuous feeding.

DACB The assessment data indicate that aspiration may have occurred, and the nurse's first action should be to turn off the tube feeding to avoid further aspiration. The next action should be to check the oxygen saturation because this may indicate the need for immediate respiratory suctioning or oxygen administration. The residual volume should be obtained because it provides data about possible causes of aspiration. Finally, the health care provider should be notified and informed of all the assessment data the nurse has just obtained.


संबंधित स्टडी सेट्स

Chapter 6: Organizational Strategy

View Set

Sentences that would be useful at a restaurant

View Set